You are on page 1of 131

The VietNam Inequality Mathematic Forum The VietNam Inequality Mathematic Forum The VietNam Inequality Mathematic Forum

The VietNam Inequality Mathematic Forum


http://www.ddbdt.tk





DISCOVERY INEQUALITIES
VIMF VIMF VIMF VIMF
The Fifth Version
Bn Quyn 2010 Thuc DIN N BT NG THC VIT NAM


Li Ni u
This file was downloaded from VietNam Inequality Mathematic Olympiad Resource Page http://www.ddbdt.tk
LI NI U.
Khng cn nghi ng g na, bt ng thc thc s chim c ch ng ring cho mnh trong nn
ton hc hin i. Nu cc bn thng xuyn theo di cc chng trnh 360
0
th thao hay 24/7 th cc bn c
th thy nhng chng trnh phn ln u bnh lun nhiu v mn th thao vua bng . Bt ng thc cng
vy, ngy nay i khp cc din n ton ta u thy vn nng bng nht, thi s nht vn l bt ng thc.
Vi mong mun c th gp cht t cng sc vo cng cuc i mi dy v hc, chng ti nhng thnh vin
ca Din n Bt ng Thc Vit Nam VIMF mun to ra mt sn chi tht s b ch cho nhng bn
ang v s yu thch b mn bt ng thc ny. V s ra i ca tp san ny l i din cho nhng g m chng
ti mong mun. Tp san bao gm nhng chuyn mc chnh:
- Bt ng thc t nhng cuc thi: tuyn chn nhng bt ng thc v nhng li gii hay t cc cuc thi
nh NMO (thi ton ca cc quc gia), TST (chn i tuyn thi ton quc t), thi tuyn sinh i hc, cc
cuc thi bt ng thc,
- Bt ng thc su tm v sng to: tuyn chn nhng bt ng thc hay m chng ti t sng to hoc
su tm c t cc bi ton trn cc din n, c bit l Din n Bt ng Thc Vit Nam.
- Gii ton nh th no?: tm kim nhng bi vit mi hay v cc vn bt ng thc c in hin i,
nhng sng to mi, nhng ng dng ca mt bt ng thc c bit no gip gii quyt c c mt lp
cc bi ton bt ng thc, nhng tm ti m rng cho mt bt ng thc hay no ,
Tp ti liu khng ch l ngi bn ng hnh vi chng ta m ti cn mong mun rng n cn gip cc
bn t trao i, tch ly kinh nghim ln nhau thng qua vic mi thnh vin hy ng gp mt cht sc mnh
vo tp san n ngy cng hon thin hn. Cc bn c th tham gia ng gp cc chuyn mc: Bt ng
thc t nhng cuc thi v Gii ton nh th no?. Ring chuyn mc bt ng thc t nhng cuc thi ku
gi s ng gp t tt c cc thnh vin thng qua vic post cc thi hay t cc nm ln forum tng hp v
tm li gii mi (nu c).


Thay li kt.

DISCOVERY INEQUALITY (DI) tri qua 4 ln chnh sa b sung c th ni l kh phong ph c v bi
ton ln li gii cho cc bi ton bt ng thc. V vi DI Vol 5 cui cng ny cha th l y trong th gii
bt ng thc mun mu nhng cng tp hp c khng it cc bi ton hay : su tm, sng tc, hay l cc bi
ton qua cc cuc thi,
Trong thi gian bin son chun b cho DI vol cui cng ny ti nhn c s ng gp qu bu ca
bn Nguyn Vn Huyn. Xin chn thnh cm n bn gi cc thi v li gii ca cc bt ng thc thi
Olympic cng vi cc bi ton m bn su tm c, l cho DI thm phong ph hn.

















Copyright 2010 by VIMF VIMF VIMF VIMF.
Tp san ny cng vi file i km c to ra v mc ch gio dc. khng c s dng ebook ny di
bt k mc ch thng mi no, tr khi c s ng ca tc gi. Mi chi tit xin vui lng lin h
http://www.ddbdt.tk
This file was downloaded from VietNam Inequality Mathematic Olympiad Resource Page http://www.ddbdt.tk

Cc bi ton Problem Of The Day

y l cc bi ton mi l v hay do cc thnh vin post trn DBTVN v c chng ti chn ng
trn trang ch mi ngi cng tho lun.

Bi ton 1. Cho cc s thc khng m sao cho khng c 2 s no cng bng 0. Chng minh rng

2
2

2
2

2
2


22
Li gii 1 (VIMF). D thy nu c mt s bng 0 th bt ng thc ng. V th ta ch cn chng minh bt
ng thc khi , , 0.
Khng mt tnh tng qut, gi s min, , . t

th 1 v
min , , nn 1. Bt ng thc c vit li thnh

2
2 1

2
2 1

2
2 1
22
p dng bt ng thc Cauchy ta c

2
2 1

2
2 1

2
2 1


2
2 1

2
2 1

1
2

2
2 1

1
2

2
2 1
4

1
4

2
2 1
.
2
2 1
.
2
2 1



Do ta ch cn chng minh
2
2 1
.
2
2 1
.
2
2 1

1
Khai trin v sau khi rt gn (vi iu kin 1), ta c bt ng thc sau
1 2 2 2

1 21

0
Bt ng thc ny ng theo gi thit 1 trn. Php chng minh hon tt. ng thc xy ra khi v ch
khi , 0 v cc hon v.

Li gii 2.
2 2 2
2 2 2
2 2 2 2 2 2
2
2 2 2
2 2 2 2 2 2
2 2 2
2 1 2 1 2
2 0
2 2 2 2 2
3 3
2 ( 2 4 2 ) 2 ( 2 4 2 )
3
2 ( 2 4 2 )
2 ( 2 4 2 ) 2 ( 2 4 2 )
2 ( 2 4 2
a bc b ca c ab
BDT
a bc b ca c ab
bc ac
a bc a bc a bc b ac b ac b ac
c
c ab c ab c ab
b a
a bc a bc a bc b ac b ac b ac
c
c ab c ab c a
+ + +
+ +
+ + +
+
+ + + + + + + +

+ + + +
+
+ + + + + + + +

+ + + +
(*)
) b

Bt ng thc s c chng minh nu ta chng minh c
2 2 2 2 2 2
2 2 2 2 2 2
2 ( 2 4 2 ) 2 ( 2 4 2 )
(2 ) (2 4 )(2 ) (2 ) (2 4 )(2 )(1)
a c
b ac b ac b ac c ab c ab c ab
a c ab a c ab c ab c b ac c b ac b ac

+ + + + + + + +
+ + + + + + + +

Chng minh bt ng thc ny quy v chng minh 2 bt ng thc
1/
2 2 2 2 2 2
(2 ) 2 2 a c ab b c ac ac a b b c + + +
Bt ng thc ny ng nh p dng bt ng thc AM-GM cho VT kt hp iu kin a b c
This file was downloaded from VietNam Inequality Mathematic Olympiad Resource Page http://www.ddbdt.tk

2/
2 2 2 2
(2 4 )(2 ) (2 4 )(2 ) a c ab c ab c b ac b ac + + + +
Bt ng thc ny chng minh khng kh, bnh phng ri khai trin, kt hp iu kin a b c
Nh vy (1) ng tc l (*) ng. Bt ng thc c chng minh, du = xy ra khi
0, , dng bt k v cc hon v.


Bi ton 2. Cho cc s thc khng m , , . Chng minh rng
( )( ) ( )( ) ( )( )
2 2 2 2 2 2 2 2 2 2 2 2
1
bc ca ab
a ab b a ac c c cb b a ab b a ac c b bc c




Li gii (VIMF). bt ng thc tng ng vi
( )( )( )
2 2 2 2 2 2 2 2 2 2 2 2
bc b bc c ca c ca a ab a ab b a ab b b bc c c ca a
( ) ( ) ( )
2 2 2 2 2 2 2 2 2 2 2 2
a b a ab b b c b bc c c a c ca a
( )( ) ( )( ) ( )( )
2 2 2 2 2 2 2 2 2 2 2 2 2 2 2
2 2 2 a bc a ab b a ac c ab c c cb b a ab b abc a ac c b bc c
( )( )( )
2 2 2 2 2 2
a ab b b bc c c ca a
Theo bt ng thc Bunhiacopxki ta c
( )( )
2 2
2
2 2 2 2 2 2
3 3 3
.
2 2 2 2 2 2 4 2 2
a a a a ab ac
a ab b a ac c b a c a b c a a bc
1 1
1 1 1 1


= =


( ) ( ) ( )( )
( ) ( )

Tng t ta cng c
( )( )
2 2 2 2 2
,
2 2
ab bc
c cb b a ab b b ca
( )( )
2 2 2 2 2
2 2
bc ca
a ac c b bc c c ab
Do
( ) ( ) ( )
2 2 2 2 2 2 2 2 2 2 2 2
a b a ab b b c b bc c c a c ca a
( )( ) ( )( ) ( )( )
2 2 2 2 2 2 2 2 2 2 2 2 2 2 2
2 2 2 a bc a ab b a ac c ab c c cb b a ab b abc a ac c b bc c
( ) ( ) ( )
2 2 2 2 2 2 2 2 2 2 2 2
a b a ab b b c b bc c c a c ca a
2 2 2 2 2 2
2 2 2
2 2 2 2 2 2
ab ca ab bc bc ca
a bc a bc ab c b ca abc c ab
1 1 1




( ) ( ) ( )
( )
4 2 2 4 4 2 2 4 4 2 2 4 3 3 3 3 3 3 3 3 3
2 a b a b b c b c c a c a a b b c c a abc a b c =
( )
2 2 2 2 2 2 2 2 2
6 abc a b b c c a ab bc ca a b c
Mt khc,
( )( )( )
2 2 2 2 2 2 4 2 2 4 4 2 2 4 4 2 2 4 3 3 3 3 3 3
a ab b b bc c c ca a a b a b b c b c c a c a a b b c c a =
( ) ( )
3 3 3 2 2 2 2 2 2 2 2 2
2 3 abc a b c abc a b b c c a ab bc ca a b c
Do ta ch cn chng minh
( ) ( )
3 3 3 2 2 2 2 2 2 2 2 2
3 abc a b c a b c abc a b b c c a ab bc ca
Hay
3 3 3 2 2 2 2 2 2
3 a b c abc a b b c c a ab bc ca
y chnh l bt ng thc Schur. Php chng minh hon tt. ng thc xy ra khi v ch khi a b c = =
hoc , 0 a b c = = v cc hon v.


Bi ton 3. Cho cc s dng , , . Chng minh

1
9

TRN QUC ANH
This file was downloaded from VietNam Inequality Mathematic Olympiad Resource Page http://www.ddbdt.tk

Li gii (VIMF). t

th 1 v bt ng thc tr thnh
1
2


1
2


1
2


1
9

Theo nguyn l Dirichlet th 2 trong 3 s 1, 1, 1 cng du. Khng mt tnh tng qut, gi
s 1 1 0 1. Theo bt ng thc Cauchy ta c
( ) ( )
( )( ) ( )
3
2
3 3
3
2 2
2
3 3
1 1 1 1 1 1
2 2 2 2
3
2 2 4 2 3 6
2 2
6
x y xy x y xy
x y
z
| |
| | | | |
| |
+ = =
| | | |
| |
+ + + + + +
+ + \
| \ \
+
\

Do ta ch cn chng minh
( )
3
2
3
1 1 1
2
3
9
2
6
z
z
| |
|
+
|
+
|
+
\

Xt ( )
( )
3
2
3
1 1
2
3
2
6
f z
z
z
| |
|
= +
|
+
|
+
\
, ta c
( )
( )
'
5 4
2
2
9 3
2
1
3
6
f z
z
z
z
=
+
| |
|
|
|
+
\

Vic ta cn tm l gii phng trnh 0, y l mt iu kh khn bi n kh phc tp v khng th
bit c phng trnh ny c bao nhiu nghim. Nhng tht may mn l phng trnh ny ch mt nghim
duy nht. Tht vy,

0 2

272 1


Ta c
2

272 1

17

129

577

833

465

97

0
R rng phng trnh ny ch c mt nghim duy nht l

1.
Lp bng bin thin ta thy hm s i du t sang khi i qua

. Do


Php chng minh hon tt. ng thc xy ra khi v ch khi 1 hay .

Bi ton 4. Cho cc s thc dng , , . Chng minh rng


Li gii. ta c


Theo bt ng thc AM-GM ta c

2
1


Do ta ch cn chng minh


Theo bt ng thc AM-GM th


Mt khc, theo bt ng thc AM-GM th


Suy ra
This file was downloaded from VietNam Inequality Mathematic Olympiad Resource Page http://www.ddbdt.tk


Do


Php chng minh hon tt

Bi ton 5. Cho cc s thc khng m , , sao cho khng c hai s no ng thi bng 0. Chng minh
1


33



Li gii (VIMF). bt ng thc cho tng ng vi

33

33

2
4




1
4

2

1
1
4


3
2
3

3
4
1
3

1
8


3
2

3
4

2 3
0

1
8

3
8

. 3
0
Ta chng minh

0. Ly i din


Ta c


1
8


3
8

. 3

1
8


9
16

1
8

7 9

16
2

. 7 9

0
Coi y l a thc bc 2 theo

. Ta c
2

4.9.2

49

47

47

98 0
Do

0. Tng t ta cng c

0. Php chng minh hon tt.



Bi ton 6. Cho cc s dng , , sao cho 1. Chng minh
1
1



1
1

1
1



3
8

TRN QUC ANH
Li gii 1 (VIMF). Ta xt 2 trng hp
This file was downloaded from VietNam Inequality Mathematic Olympiad Resource Page http://www.ddbdt.tk

Trng hp 1. Nu
Theo bt ng thc AM-GM ta c 1 2. 2 4
Tng t 1 4; 1 4. Do ta ch cn chng minh
1
1

1
1

1
1

3
2

Bt ng thc ny ng theo gi thit trn

Trng hp 2. Nu
Theo bt ng thc AM-GM ta c 1

, nn

. Tng t

. Do ta ch cn chng minh









3
2

3
2

3
2

3
2


3
2

V nn

;
1
3
.

;
1
6

1
6
. 9
3
2

Cng cc bt ng thc trn v theo v ta c iu phi chng minh.

Li gii 2 (V QUC B CN). t
2 2 2
, , , a x b y c z = = = vi , , x y z l cc s thc dng. Bt ng thc
tr thnh
2 2 2 2 2 2 2 2 2 2 2 2
1 1 1 3
.
( 1) ( ) ( 1) ( ) ( 1) ( ) 8 x y z y z x z x y
+ +
+ + + + + +

Theo bt ng thc Cauchy-Schwarz ta c
2
2 2 2
1 1
( 1)( ) ,
z
x y z xy z z
z z
+
+ + + = + = v
2
2 2 2
1 1
(1 )( ) .
y
x y z y xz y
y y
+
+ + + = + =
Suy ra
2 2
2 2 2
( 1)( 1)
( 1)( ) ,
y z
x y z
yz
+ +
+ + v cc bt ng thc tng t. T ta c
2 2 2 2 2 2 2
1
.
( 1) ( ) ( 1)( 1)( 1)
yz
x y z x y z

+ + + + +


Do ta ch cn chng minh rng
2 2 2
8
( 1)( 1)( 1) ( ).
3
x y z xy yz zx + + + + +
Theo bt ng thc Cauchy-Schwarz ta c
2 2 2 2 2 2
( 1)(1 ) , ( 1)(1 ) , ( 1)(1 ) x y x y y z y z z x z x + + + + + + + + +
V 2 bt ng thc tng t, vi ch
8
( )( )( ) ( )( ),
9
x y y z z x x y z xy yz zx + + + + + + + v 3, x y z + +
Ta suy ra
2 2 2
8 8
( 1)( 1)( 1) ( )( )( ) ( )( ) ( )
9 9
x y z x y y z z x x y z xy yz zx xy yz zx + + + + + + + + + + + +
Php chng minh hon tt.

Li gii 3. t
3 3 3
, , a x b y c z = = = , ta c
3 3 3 3 3
2 3 2 3 3 2 2 3 3
1
.
( 1) ( ) ( ) ( ) ( ) ( )
x y z xy z
a b c x xyz y z x yz y z
= =
+ + + + + +

Theo bt ng thc AM-GM ta c
2 2 2 2 2 2 2 2 2
( )( ) ( ) ( ) 2 ( )( ). x yz y z y x z z x y yz x y x z + + = + + + + +

Suy ra
This file was downloaded from VietNam Inequality Mathematic Olympiad Resource Page http://www.ddbdt.tk

2 2 2 2
2 2
4 ( )( )
( ) ( ) .
yz x y x z
x yz y z
y z
+ +
+ +
+

V ch rng
2 2 2 2
2( ) y yz z y z + + , khi
2 2 2 2
2 2 2 2 2 2 2 2 2 2 2 2 2
1 ( ) ( )
.
( 1) ( ) 4( )( )( ) 2( )( )( )

xy z y z xy z y z
a b c x y x z y yz z x y x z y z
+ +

+ + + + + + + +

Do ta cn chng minh rng
2 2 2 2 2 2 2 2
4 ( ) 3( )( )( ) 3 4 ,
x y y z z x x y
xy z y z x y y z z x
y x z y x z y x
| || | | |
| |
+ + + + + + + +
| | | |
\
\ \ \


Hay
2 2 2 2
( ) ( ) ( ) ( )
3 2 2 2 24 4 .
x y y z z x x y
xy yz zx xy
( ( (
+ + + +
( ( (



T bt ng thc quen thuc (2 )(2 )(2 ) 8 4( ), , , 0, u v w u v w u v w + + + + + + ta c
2 2 2 2 2
( ) ( ) ( ) ( ) ( )
3 2 2 2 24 12 24 4 ,
x y y z z x x y x y
xy yz zx xy xy
( ( (
+ + + + +
( ( (



Php chng minh hon tt.

Li gii 4. t , ,
y x z
a b c
x z y
= = = . V bt ng thc tr thnh
2 2 2
2 2 2 2 2 2
3
.
( ) ( ) ( ) ( ) ( ) ( ) 8
x yz y zx z xy
x y xy z y z yz x z x zx y
+ +
+ + + + + +

Theo bt ng thc AM-GM ta c
2
2 , xy z z xy + v
2 2 2
( ) 2 2 ( ). x y xy x y + +
Do
2
2 2
2 2
.
( ) ( )
4 2( )
x yz x
x y xy z
x y

+ +
+

Nn ta ch cn chng minh rng
2 2 2 2 2 2
3
,
2
x y z
x y y z z x
+ +
+ + +

y l bi ton quen thuc. phn chng minh rng xin dnh cho bn c.

Bi ton 7. Cho cc s dng , , . Chng minh rng
1

2

1

2

1



Li gii (VIMF). bt ng thc tng ng vi
( )
( )( )( )
( ) ( )( )
( )( )( )
( )
( )( )( )
2 2 3 3 2
2 2 2 2 2 2 2 2 2
2
2 2 2
2 2 2 2 2 2 2 2 2
2 2 2
2 2 2 2 2 2 2 2 2
4 2
2 4
2 2 2
2
2 2 2
2
2 2 2
2
b c a b a c a bc
ab bc ca
a b b c c a a bc b ca c ab
ab bc ca ab bc ca a b c
ab bc ca
a b b c c a a bc b ca c ab
ab bc ca a b c
ab bc ca
a b b c c a a bc b ca c ab
a b
+ + +
+ +

+ + + + +
+ + + + + + +
+ +

+ + + + +
+ + + + +
+ +

+ + + + +
+

( ) ( )
( ) ( )( ) ( ) ( )( )
( )( )( ) ( )( )( ) ( )( )( )
4 2 3 2 2 3 2 2 2 3 3 4
2 2 2 2
4 2 2
2 2 2
3 4
1
2
2
0
a c a b c ab c a b c a b a bc
a b c a ab ac bc b c a ab ac ab ac bc ab ac
a b a c ab ac b c b a ba bc c a c b ca cb
+ + + +
+ +
+ +



Khng mt tnh tng qut, gi s . t

th
. V bt ng thc trn vit di dng
This file was downloaded from VietNam Inequality Mathematic Olympiad Resource Page http://www.ddbdt.tk

0
.


.


.


.



Theo bt ng thc ta c
.


.


2


Php chng minh hon tt. ng thc xy ra khi v ch khi .

Li gii 2. Khng mt tnh tng qut, gi s max{ , , } a a b c = . Theo bt ng thc Cauchy-Schwarz ta c
2 2 2 2
1 1 4
.
2 2 2 2 b ca c ab b c ab ac



Do ta ch cn chng minh
2 2 2 2 2 2 2 2 2
2 2 2 2 2 2 2 2 2 2 2
2 2 2
2 2 2 2 2 2 2 2 2 2 2
2 2 2
1 4
2 2 2
4[ ( ) ]
( )
2 2 ( )
( ) ( )( ) 4 2 ( )
.
2 2 ( )
ab bc ca
a bc b c ab ac a b b c c a
a b b c c a a b c b c
bc a b c
a bc b c a b c
a b bc c b c a b c b c b c a b c
a bc b c a b c










Ta c
2 2 2 2 2 2 2 2 2 2
4 2 ( ) 2 ( ) 2( )( ) 2 ( ), b c a b c bc b c bc a b c bc b c =

V
2 2 2 2 2
2 2 2 2 2
( ) ( )
( ) ,
2
a b c bc b c
a b bc c b c


Suy ra
2 2 2 2 2 2 2 2 2 2
2 2 2 2 2 2
( ) ( ) ( )( ) 2 ( ) ( ) 2
.
2( 2 ) 2 ( ) 2( 2 ) 2 ( )
a b c bc b c a b c b c bc b c a bc a b c bc
a bc b c a b c a bc b c a b c




Theo bt ng thc AM-GM ta c
2 2
2 ( ) 2 2 ( ) b c a b c bc a b c . Do ta cn chng minh
2
2 2 2
( ) 2 3 3
1 1
2 2 2
a bc a b c bc bc bc bc bc
a bc ab ac bc a bc ab ac bc ab ac bc a bc




Bt ng thc ny ng v
2
2 ( ) ( )( ) 0 a bc ab ac bc a b a c = .
Php chng minh hon tt. ng thc xy ra khi v ch khi . a b c = =


Bi ton 8. Cho cc s 0. Chng minh rng



NGUYN C TON
Li gii (VIMF, V QUC B CN). v 0 nn
2 2 2 2 2 2 2 2 2 2 2 2 2 2 2
2 2 2 2 2 2
1 1
2 2 2
2 2
x y y z z x x y y z z x xy yz zx x y y z z x x y y z z x
x y z x y z
z x y z x y z x y z x y z x y
| | | |
+ + = + + + + + + + +
| |
\ \

( )( )( )( ) ( ) ( ) ( ) ( ) ( )( )( )
2 2 2
2 2 2
( ) 1 1 1
2 2 2 ( )
xy yz zx x y y z x z x y z y z x z x y xy yz zx x y z x y y z x z
xyz yz zx xy xyz x y z
| |
+ + + + + +
| = + + + =
|
+ +
\

( ) ( ) ( ) ( )( )( )( )( )( )
( )
( )
2 2 2 2
2 2 2 2 2 2 2 2 2 2 2 2
2 2 2 2 2 2
1 1 1
2 2 2
x y x z y z y x z x z y x y y z z x xy yz zx
x y y z z x x y y z x z
x yz y zx z xy xyz x y z x yz y zx z xy
| |
+ +
+ + +
|
+ + + +
| + + + +
\

( ) ( ) ( )
( )
( ) ( ) ( )
( )
( ) ( ) ( )
( )
2 2 2 2 2 2 2 2 2
1 1
2 2
x y y z z x x y y z z x x y y z z x
xyz x y z xyz x y z xyz x y z
( ( (


+ =
+ + + + + +

Php chng minh hon tt chng minh hon tt.

Bi ton 9. Cho cc s thc dng , , sao cho
2 2 2 2 2 2
1 1 1 2
.
4 4 4 3 a b b c c a



This file was downloaded from VietNam Inequality Mathematic Olympiad Resource Page http://www.ddbdt.tk

Chng minh rng
3
.
4
ab bc ca
NGUYN VN HUYN

Li gii. Ta c
2
2 2
( )
2
a b
a b

, nn t gi thit ta suy ra
2
2 2
1 1 1 ( )
.
( ) 8 3 3 ( ) 8
a b
a b a b





Theo bt ng thc Cauchy-Schwarz tra c
2 2
2 2 2 2
( ) 4( )
.
( ) 8 ( ) 8 ( ) 8 ( ) 8
a b a b c
a b a b b c c a


Suy ra
2 2 2 2 2 2 2
( ) ( ) ( ) 24 12( ) 12 5( ) 11( ) 16( ) a b b c c a a b c a b c ab bc ca ab bc ca
T ta c iu phi chng minh. ng thc xy ra khi v ch khi
1
.
2
a b c = = =


Bi ton 10. Cho , , l cc s thc dng sao cho 1. Chng minh rng
4 4 4
1
( ) ( ) ( ) .
12
a b c b c a c a b

DNG C LM

Li gii 1. Khng mt tnh tng qut, gi s min{ , , } c a b c = . Ta c
4 3
( ) ( ) a b c ab b c v
4 3
( ) ( ) , b a c ab a c
Do
4 4 3 3 2 2 2
( ) ( ) [( ) ( ) ] ( 2 )( ) a b c b c a ab a c b c ab a b c a b c ab bc ca =
2 2 2
( )( ) ab a b c a b c ab bc ca
Ta cng c
4 2 2 2 2 2 2
( ) ( ) [( ) ( )( )] ( ) ( ) c a b c a b a b a c b c c a b a b c ab bc ca =
2 2 2
( )( )( ). c a b a b c a b c ab bc ca
Do
4 4 4 2 2 2
( ) ( ) ( ) ( )( ), a b c b c a c a b ab bc ca a b c ab bc ca
V ta ch cn chng minh rng
2 2 2
12( )( ) 1, ab bc ca a b c ab bc ca
Theo bt ng thc AM-GM ta c
2 2 2 2 2 2 2
12( )( ) [3( ) ( )] 1. ab bc ca a b c ab bc ca ab bc ca a b c ab bc ca =

Php chng minh hon tt.

Li gii 2 (VIMF). Ta c

2 2


Mt khc, ta li c


1
3

. 3

1
3
.
1
4

1
12


1
12

Vy, gi tr ln nht ca l

. ng thc xy ra khi v ch khi xy ra khi , ,

, 0 v
cc hon v

This file was downloaded from VietNam Inequality Mathematic Olympiad Resource Page http://www.ddbdt.tk

Bi ton 11. Cho , , l cc s dng. Chng minh rng
2 2 2
2 2 2 2 2 2
3
.
( ) ( ) ( )
a b c
b a ab b c b bc c a c ca a a b c



Li gii. p dng bt ng thc Cauchy-Schwarz ta c
2
2 2 2
2 2
3 .
( )
a a ab b a a
b a ab b b b b
l 1
1

l


l ( ) ( )
l


Do ta cn chng minh
( )
2 2
,
a a ab b
a
b b
1



( )

hay .
bc ca ab
a b c
a b c

Bt ng thc ny ng theo bt ng thc AM-GM. Php chng minh hon tt. ng thc xy ra khi v
ch khi .

Bi ton 12. Cho , , a b c l cc s thc dng tha 1 ab bc ca + + = . Chng minh rng
2 2 2 2 2 2
1 1 1 1
2 .
3
b bc c c ca a a ab b



V QUC B CN

Li gii (V QUC B CN). Khng mt tnh tng qut, gi s , 0 x y z , khi ta c cc bt ng
thc ng sau
2 2 2
2 2
,
xy yz zx x z xy yz zx y yz z xz z
y y
y yz z y z x z y z x z y z




Bt ng thc ny ng v ( ) ( ) ( ) ( ) 0. x y z z x z x y z z x z =

2 2 2 2
2 2 2
2 2 2 2
( )( )
( ) ( )( ) ( )
( ) ( )( ) ( ) ( )( ) 3 ( )
1 1 ,
xy yz zx x z y z
x xy y x z x z y z y z
x z x z y z y z x z y z z x y z z
x xy y xy yz zx x xy y xy yz zx







Bt ng thc ny ng v
2 2 2
( )( ) ( )( ) ( ) ( ). xy yz zx x y z z x y x y z z x y z x xy y
Tr li bi ton, gi s min{ , , } c a b c = . Theo 2 bt ng thc trn ta c

2 2
2 2
1
,
ab bc ca a c
b bc c b c
b bc c

=



2 2
2 2
1
,
ab bc ca b c
a ac c a c
a ac c

=



V
2 2 2 2
2 2
1 ( )( )
.
( ) ( )( ) ( )
ab bc ca a c b c
a ab b a c a c b c b c
a ab b

=



Do ta ch cn chng minh
2 2
( )( ) 1
2 .
( ) ( )( ) ( ) 3
a c b c a c b c
b c a c a c a c b c b c




t 2
a c b c
u
b c a c

=

, bt ng thc tr thnh
2
1 1
2 .
3
1
u
u


Ta c
2
2 2
2

1 1 1 1 3 1
2 2 2
1 1 3 3 3
1
u
u u u
u u
u


2 2 2
2 2 2
4 2( 4) ( 2)(3 2 7)
( 2) ( 2) 0.
3(

1) 3( 1) 3( 1)
u u u u u
u u
u u u

= =


Bt ng thc cui cng ng do ta c iu phi chng minh. T gi nh min{ , , } c x y z = , ta c ngy
ng thc xy ra khi 1 a b = = v 0. c =


This file was downloaded from VietNam Inequality Mathematic Olympiad Resource Page http://www.ddbdt.tk

Bi ton 13. Cho cc s dng , , . Chng minh rng
2 2 2
3
.
2
4 4 4
a b c
b ca ab c ab bc a bc ca



TRN QUC ANH

Li gii 1 (V QUC B CN). Theo bt ng thc Holder ta c
( )
2
3
2 3
2
( )(4 ) .
4
a
a a c b ca ab a a c
b ca ab
1

l

l
l

( )


Do ta chng minh
( )
3
2 3
3
( )(4 ),
2
a a c a a c b ca ab

hay
( )
3
2 2 2 2 2 3
3
( ) 5 5 .
2
a a c ab a b a b a bc
l

l
l


Khng mt tnh tng qut, gi s 3 a b c = . Theo bt ng thc AM-GM v bt ng thc bit
1,
4
a
b

ta c
( )
3
3 3 3
2
3
3 3
3

( ) ( ) 3 2 (3 )
2 3 2
2( ) 2 2 4
2( )
3 2 3 2
1 3 2.
2 4 2

a a c a a c a b
a a c
a c b
a c
a
a a
b

= =

1

= =


( )



Do ta cn chng minh
2 2 2 2 2
36 ( ) 5 5 , ab a b a b a bc

hay
( )
4
2 2 2 2 2
4 9 ( ) 5 5 . a ab a b a b a bc
l

l
l


Sau khi khai trin ta cn chng minh
4 2 2 2 2 2
4 3 7 ( ) 21 0, a a bc ab a b a b


Hay
( )
2 2 4 2 2
3 ( )( ) 10 ( ) 0, a a b a c ab a b a a b


Bt ng thc trn ng theo bt ng thc Schur v bt ng thc AM-GM.
Php chng minh hon tt. ng thc xy ra khi v ch khi . a b c = =


Li gii 2 (VIMF). Theo bt ng thc Holder ta c


Do ta ch cn chng minh


3
2
2


Ta c
2

12

32

44

12

144

15

20

60

30

90


Th 2 ng thc trn vo bt ng thc (*) ta c bt ng thc tng ng l


9
2

12

21

126

0
Theo bt ng thc ta c
This file was downloaded from VietNam Inequality Mathematic Olympiad Resource Page http://www.ddbdt.tk

;
6


(chia 2 v cho

)
Bt ng thc ny ng v


1
3


V
9
2

12

21


9
2
. 6

12.3

21. 3
126


Cng cc bt ng thc trn ta suy ra iu phi chng minh.
Php chng minh hon tt. ng thc xy ra khi v ch khi .

Bi ton 14. Cho cc s dng a,b,c tha mn 3. Chng minh
1 1 1 3
2 2 2 4 a b b c c a
+ +
+ + + + + +

VIMF
Li gii. t 1 ; 1 ; 1
( )( ) ( ) ( ) ( )( ) ( ) 1 1 1 1 1 1 3 2 x y y z z x xy yz zx x y z + + = + + = + +
Do , bt ng thc cho chuyn thnh
( ) ( ) ( )
( )
1 1 1 3
2
3
2
4
x y z
x y y z z x xy yz zx
z x y xy x y z yz y z x zx
x y z
x y y z z x
xy yz zx x y z
x y y z z x
+ +
+ +
+ + + + +
+ + + + + +
+ + + +
+ + +
+ +
+ +
+ + +



























Bt ng Thc T Nhng Cuc Thi
This file was downloaded from VietNam Inequality Mathematic Olympiad Resource Page http://www.ddbdt.tk

BT NG THC T NHNG CUC THI
Bt ng Thc T Cc Cuc Thi Cc Quc Gia V Vng
Lnh Th

LTG. Trong bi ny chng ti s cp mt s bi ton Olympiad gip cc bn n tp trong cc k thi
Olympic.

BI O 1. Cho cc s thc dng , , sao cho 1. Chng minh rng
3
1


3
1


3
1

3
UK TST 2005
LI GII 1 (VIMF). Do 1 nn t , ,
y z x
a b c
x y z
= = = . Bt ng thc c vit li nh sau
2 2
2
2 2 2
2 2 2
3 3 3 3
3 1 1 1
( ) ( ) ( ) 4
x xy y yz z zx x y y z z x
x y y z z x x y y z z x
(
| | | | + + + | |
+ + + + + + + (
| | |
+ + + + + +
\ ( \ \



2 2 2 2 2 2
2 2 2
1 ( ) ( ) ( ) ( ) ( ) ( )
3
4 ( ) ( ) ( )
x y x y y z y z z x z x
x y y z z x
( + + +
+ + +
(
+ + +


Hay
2 2
2
3( )( )( )
3
( )( )( )
x y y z z x x y y z z x x y y z z x
x y y z z x x y y z z x x y y z z x
| | | | | | | |
+ + + + =
| | | |
+ + + + + + + + +
\
\ \ \

+ Nu ( )( )( ) 0 x y y z z x th bt trn hin nhin ng.
+ Nu ( )( )( ) 0 x y y z z x th theo bt ng thc ta c
2 2 2
2
3
( )( )( )
3
( )( )( )
x y y z z x x y y z z x
x y y z z x x y y z z x
| | | | | | | |
+ +
| | | |
+ + + + + +
\
\ \ \

Nn ta ch cn chng minh
( )
2 2 2
( )( )( )
1 2 0
( )( )( )
x y y z z x
x y y z z x
x y y z z x



(lun ng).
Vy ta c iu phi chng minh. ng thc xy ra khi v ch khi 1.

LI GII 2 (VIMF). Trc tin ta chng minh 2 b sau
B 1.
1 1 1 2
1
1 1 1 1 a b c a b c



B 2.
( ) ( ) ( )
2 2 2
1 1 1 1
1
1
1 1 1
a b c
a b c




Chng minh B 1. Bt ng thc tng ng vi
3 2
2

3
1

1
M theo bt ng thc th

3
Vy B 1 c chng minh.
Chng minh B 2. Theo nguyn l irichlet th 2 trong 3 s 1, 1, 1 cng du, khng
mt tnh tng qut gi s 1 1 0

1 . Ta c
1
1


1
1


1
1
1

0 ng
nn
( ) ( )
2 2
1 1 1
1 1
1 1
c
ab c
a b
=



Do
( ) ( ) ( ) ( )
2 2 2 2
1 1 1 1 1 1
1
1
1 1
1 1 1 1
1
c
c
a b c c
a b c c
c
c
=





Bt ng Thc T Cc Cuc Thi Cc Quc Gia V Vng Lnh Th
This file was downloaded from VietNam Inequality Mathematic Olympiad Resource Page http://www.ddbdt.tk

Vy B 2 c chng minh.
Tr li bi ton. Bt ng thc cho tng ng vi
1
1

1
1

1
1

2
1


2
1


2
1

3
M theo b trn ta c
1
1

1
1

1
1

2
1


2
1


2
1

2
1


2
1


2
1


2
1
1 2 1 3
Vy ta c iu phi chng minh. ng thc xy ra khi v ch khi 1

LI GII 3 (trungdeptrai). t

th 1. Bt ng thc cn chng minh tr thnh


3

3
2

1
1

1
1

1
1


1
1

1
1

1
1

Nh vy ta ch cn chng minh


1
1


1

0
Bt ng thc trn ng v

2 2

0
Tng t i 2 bt ng thc cn li ri cng v theo v ta c iu phi chng minh.

BI O 2. Cho cc s thc dng , , v , , sao cho 1. Chng minh

1

3
Russia 2002
LI GII 1. Ta c

1

6
Theo bt ng thc 2 s th 2, 2, 2
a z b x c y
z a x b y c

Cng cc bt ng thc ny v theo v ta suy ra iu phi chng minh.

LI GII 2. T iu kin ta c

1
Hay

th 1

, 1

, 1


V bt ng thc cn chng minh c vit li thnh

3 1
1
1

1
1

1
1
3

1
1
1
1
1
1
1
1
1 6

1
1

1
1

1
1

1
1

1
1

1
1
6
D dng chng minh c bt ng thc trn bng bt ng thc 6 s

BI O 3. Cho cc s thc dng , , sao cho 1. Chng minh rng

3
4

Romania 2005, Cezar Lupu
Bt ng Thc T Cc Cuc Thi Cc Quc Gia V Vng Lnh Th
This file was downloaded from VietNam Inequality Mathematic Olympiad Resource Page http://www.ddbdt.tk

LI GII 1. T bt ng thc quen thuc

v gi thit
suy ra


Hn na ta cng c 3
Nn 3


T y ta d dng suy ra iu phi chng minh.

LI GII 2 (VIMF). t , , th 1 v


V bt ng thc cn chng minh c vit li thnh

2
.

2


2
.

2


2
.

2

3
4

Hay

3 2
V 1 nn bt ng thc ny c th vit li thnh

2 1 2
y l mt bt ng thc kh quen thuc. Php chng minh hon tt.

BI O 4. Cho , , l cc s thc khng m sao cho 1. Chng minh rng
3 3 3
2 a a b b c c a b c + + + + + + +
Iran TST 2008
LI GII 1 (Albanian Eagle). Bt ng thc cn chng minh c vit li thnh



2



p dng bt ng thc i vi hm

ta c



Do ta ch cn chng minh

2
4


Khng mt tnh tng qut, gi s , , . Khi bt ng thc trn c vit li thnh

0
Bt ng thc ny hin nhin ng do , , . Php chng minh hon tt.

LI GII 2 (VIMF). Theo bt ng thc ta c
3 3 3 3 3 3
( ) ( ) ( ) a a b b c c a a ab bc ca b b ab bc ca c c ab bc ca + + + + + = + + + + + + + + + + +

( ) ( )
2 2
2 2 2
( ) ( ) ( ) 3 a a b c abc b a b c abc c a b c abc a a b c abc = + + + + + + + + + + + + + +


( )
3
9 a b c abc = + + + .
Do ta ch cn chng minh
( ) ( )( )
3
3 2
9 2 3 ( ) a b c abc a b c ab bc ca a abc a b c + + + + + + + + +

.
y chnh l bt ng thc . Vy bi ton c chng minh .
ng thc xy ra khi v ch khi , ,
1 1 1
; ;
3 3 3
1



( )

hoc 1; 1; 0 v cc hon v.

LI GII 3 (conan236).
D nhin bt ng thc trn ng khi c 2 hoc 3 s bng 0
By gi ta ch cn xt 2 trng hp sau :
Trng hp 1. Tn ti ng 1 s bng 0. Khng mt tnh tng qut l .
Khi gi thit tr thnh 1 v bt thc c vit di dng
2
1 2 a a + (d nhin ng)
Trong trng hp ny ta c du bng xy ra khi 1 v 0.
Trng hp 2. Khng c s no bng 0, tc l , , 0.
Bt ng Thc T Cc Cuc Thi Cc Quc Gia V Vng Lnh Th
This file was downloaded from VietNam Inequality Mathematic Olympiad Resource Page http://www.ddbdt.tk

T iu kin 1 v , , 0 ta c th t , , , trong , , l
ba gc ca 1 tam gic nhn .
Khi ta c
3
2
.
2
a a cotA
sin A
+ = ,
3
2
.
2
b b cotB
sin B
+ = ,
3
2
.
2
c c cotC
sin C
+ =
Vy ta c
2
3
2
.
2 2
.
2
cot A
a a cotA
sin A sin A
cotA
+ = =

. p dng bt ng thc ta c
2
2
( )
2 2
. .
2 2
cotA
cot A
sin A sin A
cotA cotA


Mt khc li p dng bt ng thc ta c
2 1 2
. . .
2 3 2
sin A sin A
cotA cotA


M d thy
4
2 3. 3
2 2
sin A

. Do
2
4
2
2 3
.
2
cotA
cot A
sin A
cotA



Mt khc 3 cotA

. Vy nn
4
2 2
3
cotA
cotA


. Suy ra
2
2
2
.
2
cot A
cotA
sin A
cotA



Php chng minh hon tt. Trong trng hp ny th ng thc xy ra khi

BI O 5. Cho cc s thc dng , , . Chng minh rng

2 9
APMO 2004
LI GII 1. t

. Khi
( )( ) ( ) ( )
2 2
2 2 2 2 2 2 2 2 2 2
3
2 2 1 2 2 3 2 2 2 3 3 3
2
b c b c b c bc b c b c b c b c = =
V do ta ch cn chng minh

2
3
2

3 9

22

6
Theo bt ng thc thi

22

2 2

6
Php chng minh hon tt.

LI GII 2 (VIMF). Ta s chng minh bt ng thc cht hn

2 3


vi mi s thc dng , ,
CHNG MINH. Bt ng thc trn tng ng vi
2

2 7 9
Theo bt ng thc AM GM th 2

2 2

2 2

2 4 4 4
V 3

3 3 3
Nn ta ch cn chng minh

2 1 2
Theo nguyn l Dirichlet th 2 trong 3 s 1, 1, 1 cng du. Khng mt tnh tng qut, gi s
1 1 0 th 2 1 1 0 2 2 2 2 . Do ta ch cn chng minh

1 2 2

0
Bt ng thc trn lun ng. Php chng minh hon tt. ng thc xy ra khi v ch khi 1.

LI GII 3 (VIMF). Ta s chng minh bt ng thc cht hn bi ton ban u nhng trong phm vi rng hn
vi bi ton trong li gii 2.

2 3


vi mi s thc , ,
CHNG MINH. Bt ng thc cho tng ng vi
Bt ng Thc T Cc Cuc Thi Cc Quc Gia V Vng Lnh Th
This file was downloaded from VietNam Inequality Mathematic Olympiad Resource Page http://www.ddbdt.tk

2

8 6
Theo bt ng thc th 2

1 2

1 2

1 4
Nn ta ch cn chng minh

2 2
Theo nguyn l Dirichlet th 2 trong 3 s

1,

1,

1 cng du. Khng mt tnh tng qut, gi


s

1 0

1 0, suy ra

. Do

1 2
Php chng minh hon tt. ng thc xy ra khi v ch khi 1

BI O 6. Cho cc s thc , , sao cho . Tm gi tr nh nht ca
2 2 2
1 1 1
x y z
P
x y z
=


Brazilian Math Olympiads
LI GII 1 (V QUC B CN). Cho 1 v 1 th

=
1
2
P , v ta s chng minh gi tr nh nht
ca l
1
2

, tc l chng minh
2 2 2
2 2 2 2 2 2
1 ( 1) ( 1) ( 1)
1 1 1 2 1 1 1
x y z x y z
x y z x y z




T iu kin ta c ( 1) z x y x y xy = .
Ch rng ta khng th c 1 x y = , bi v nu ngc l 1 x y = , th 0 x y xy = hay 1, m
2
( ) 1
4 4
x y
xy

= (iu ny v l) nn 1 x y v ta c
1
x y xy
z
x y

=


Nn ta ch cn chng minh
2 2 2
2 2 2 2
( 1) ( 1) ( 1)
1 1 ( 1) ( )
x y xy
x y x y x y xy




Theo bt ng thc th
[ [
2
2 2 2
2 2 2 2 2 2 2 2 2 2
(1 )(1 ) (1 )(1 )
( 1) ( 1) 4( 1)
1 1 (1 )(1 ) (1 )(1 ) (1 )(1 ) (1 )(1 )
x y y x
x y xy
x y x y y x x y y x


=


Nn ta ch cn chng minh
2 2 2 2 2 2
4( 1) 4( ) (1 )(1 ) (1 )(1 ) x y x y xy x y y x
2 2 2 2
( ) ( 3 3) (3 8 3) 3 3 1 0 f x y y x y y x y y =

M ta c
2 2 2 2 2 2
(3 8 3) 4( 3 3)(3 3 1) 3( 1) 0
f
y y y y y y y = = nn ( ) 0 f x
Php chng minh hon tt.


LI GII 2 (VIMF). t , , , t gi thit ta c
Cng nh d on trong li gii 1 ta s chng minh gi tr nh nht ca l

v ta chng minh
2 2 2 2 2 2
1 3 1
1 1 1 2 2 2 1 2
x y z qr pq r p
x y z r p q q pr




2

2 2 1 0

6 4

1 0
T bt ng thc ng

0, ta thu c

2 2 6 2

6 2 2
V do
( ) ( )
2 2
2 2 2 2 2 2 2
6 4 1 6 2 1 2 6 2 1 6 2 2 1 0 r r p r r p p r r p r pr p r p p = =
Php chng minh hon tt.

BI O 7. Cho , , l cc s thc dng. Chng minh bt ng thc
( )
2 2 2
3
ab bc ca
a b c
c a b
+ + + +

Php 2005
LI GII 1.Theo bt ng thc ta c
( )
2
2 2 2
3 3
ab bc ca ab bc bc ca ca ab
a b c
c a b c a a b b c
| | | |
+ + + + = + +
| |
\ \

Bt ng Thc T Cc Cuc Thi Cc Quc Gia V Vng Lnh Th
This file was downloaded from VietNam Inequality Mathematic Olympiad Resource Page http://www.ddbdt.tk

Nn t y ta suy ra ngay iu phi chng minh.

LI GII 2 (VIMF). Ta s chng minh bt ng thc cht hn l
( )
2 2 2 2 2 2
2 2 2
3
ab bc ca a b b c c a
a b c
c a b a b b c c a
+ + +
+ + + + + +
+ + +

Trc tin ta chng minh
( )
2 2 2 2 2 2
2 2 2
3
a b b c c a
a b c
a b b c c a
+ + +
+ + + +
+ + +

Bt ng thc trn tng ng vi
( )
2 2
2 2 2
3 ( )
2
a b a b
a b c a b c
a b
| | + +
+ + + +
|
+
\


Ta a v dng . . vi
( )
2 2 2
1 1
;
2( )
3
a
S
b c
a b c a b c
=
+
+ + + + +

( )
2 2 2
1 1
;
2( )
3
b
S
a c
a b c a b c
=
+
+ + + + +
( )
2 2 2
1 1
2( )
3
c
S
b a
a b c a b c
=
+
+ + + + +
.

D thy cc , ,
a b c
S S S

u dng nn bt ng thc trn ng.
Tip theo ta chng minh
2 2 2 2 2 2
ab bc ca a b b c c a
c a b a b b c c a
+ + +
+ + + +
+ + +

Bt ng thc trn tng ng vi
2 2
2 2
ab a b a b a b
c a b
| | + + + | |

| |
+
\
\


Ta cng a v dng . . vi
1 1 1
; ;
a b c
a b c
S S S
bc b c ac a c ab b a
= = =
+ + +
.
Khng mt tnh tng qut, gi s , a b c theo bt ng thc ta c
1 1 1 1 2 1 1
2. . 0
( ) ( )
a c
a c a c a c
S S
bc ab b a b c bc ab b a b c b b a b c b b a b b c
+ = + = = + >
+ + + + + + + +
Tng
t
1 ( )
0; 0
( )
b c b
b a b c bc
S S S
ac a c ac a c
+
+ > = =
+ +
. Php chng minh hon tt.

BI O 8. Cho cc s thc dng , , . Chng minh rng


USA TST 2009
LI GII 1 (VIMF). Bt ng thc cho tng ng vi

0
Hay


0
Ta a v dng . . :

0
Trong


Khng mt tnh tng qut, gi s . Khi d thy

0
Ta c

0,

0
Do

0
Vy ta c iu phi chng minh. ng thc xy ra khi v ch khi .

LI GII 2 (V QUC B CN).
Bt ng Thc T Cc Cuc Thi Cc Quc Gia V Vng Lnh Th
This file was downloaded from VietNam Inequality Mathematic Olympiad Resource Page http://www.ddbdt.tk

t

th bt ng thc tr thnh


Hay


Ch rng


Do bt ng thc c vit li thnh

0
Hay

0
D dng chng minh c bt ng thc ny ng theo bt ng thc .
Php chng minh hon tt.

NHN XT. V QUC B CN lu rng bt ng thc trn cn th vit li thnh bt ng thc ng sau

0

BI O 9. Cho cc s thc , , 1. Chng minh rng
1

2
JBMO 2003
LI GII 1. Theo bt ng thc th ta c bt ng thc ng

, v tng t cho 2 bt
ng thc cn li ta suy ra
1

2

1

2

1

2

t 1

, 1

, 1

, , , 0 th bt ng thc trn c vit li thnh

2


2


2
1
Theo bt ng thc th
( )
( )
2
1
2 2 2 3
a b c
a b c
b c c a a b ab bc ca




Php chng minh hon tt.

LI GII 2 (VIMF). Theo bt ng thc th
( )
( )( ) ( )( ) ( )( )
2
2 2 2
2 2 2
2 2 2 2 2 2 2 2 2
1 1 1
1 1 1
1 1 1 1 1 1 1 1 1
x y z
x y z
y z z x x y x y z y z x z x y





Nn ta ch cn chng minh
3

3 2

2
Hay

3 2 2


Theo bt ng thc th

1 2 ,

.
Cng 2 bt ng thc trn v theo v ta c iu phi chng minh. Php chng minh hon tt.

BI O 10. Cho cc s thc dng , , sao cho 3. Chng minh rng
1


Romania TST 2006
LI GII 1 (VIMF). Bt ng thc cho tng ng vi
1

2 9
Bt ng Thc T Cc Cuc Thi Cc Quc Gia V Vng Lnh Th
This file was downloaded from VietNam Inequality Mathematic Olympiad Resource Page http://www.ddbdt.tk

Theo bt ng thc ta c
1

2
1

23

23
3
1

9
Php chng minh hon tt.

LI GII 2 (VIMF). Ta c
1


9


27


Do ta ch cn chng minh
27

27
Theo bt ng thc 3 s ta c

27
Php chng minh hon tt.

BI O11. Cho cc s thc dng , , sao cho 1. Chng minh rng
1

1


1

1


1

1


27
31

Serbian National Olympiad 2008
LI GII 1 (V QUC B CN). t

th bt ng thc trn tng ng vi

27

27

27

3
31

Theo bt ng thc th 3 4 9. Do ta ch cn chng minh bt ng thc sau

3
9

4 72

3
31
1
31
9

4 72
0

7 8 10 7 8 10

.
8

15 10


0
vi
( ) 4 72
9
ab bc ca
s

= .
Php chng minh hon tt.

LI GII 2 (VIMF). Bt ng thc cho tng ng vi

1

27
31

Khng mt tnh tng qut, gi s . Khi

1. T gi
thit ny ta cng c
2 2
1 1
x y
xyz x xyz y



Tht vy, bt ng thc ny tng ng vi
1

1


1

1

1 0
Ch rng t iu kin 1 2 1 nn

1 0 v theo gi thit trn


. Do bt ng thc trn l ng. Tng t ta cng suy ra c

1

Do theo bt ng thc cho 2 dy cng chiu ta c
Bt ng Thc T Cc Cuc Thi Cc Quc Gia V Vng Lnh Th
This file was downloaded from VietNam Inequality Mathematic Olympiad Resource Page http://www.ddbdt.tk

1
3 3
Suy ra
2 2 2 2 2 2
3
1 1 1 3 3
x y z
xyz x xyz y xyz z xyz x y z



Mt khc, t bt ng thc

suy ra
3
1
3


2
3


Nn
( ) ( )
2
2 2 2 2 2 2
1 1 10 1 31
3 3 3
3 3 3 9 9
xyz x y z x y z x y z =
Do
2 2 2 2 2 2
3 27
1 1 1 3 3 31
x y z
xyz x xyz y xyz z xyz x y z



Php chng minh hon tt.

BI O 12. Cho cc s thc dng

, ,

sao cho

1. Chng minh rng



1

China TST 2007
LI GII (NGUYN ANH TUN). Ta xt 2 trng hp
Trng hp 1. Nu

. p dng bt ng thc , ta c

1

Nhn cc v cc bt ng thc trn ta suy ra iu phi chng minh.
Trng hp 2. Nu

th theo bt ng thc , ta c
( )
2
1 2 1 2
1 2 2 3 1 2 2 2
2 2 3 3 1 2 3 1
1 1 1 1
n n
n
a
a a a a a a
a a a a a a
a a a a a a a a
1
1





( )
( )

Mt khc, theo bt ng thc th
( ) ( ) ( ) ( )
2
3
1 2
1 2 2 3 1 1 2
2 3 1
1 1 1 1
1 1 1
n
n n
a a a
a a a a a a a a a
a a a
1

l
=

l

( )

T suy ra
2
1 2
2 3 1
1
.
1
1 1 1 1
1
n
a a a n
n a a a
n
1



( )


Php chng minh hon tt.


BI O 13. Cho cc s thc dng , , sao cho 4. Chng minh rng


China TST 2007
LI GII (VIMF). t

th , , , v iu kin
4 tr thnh 4.
V bt ng thc cn chng minh c vit li thnh
T iu kin 4 ta c th t

v bt ng thc trn c vit


li thnh
2


2


2


2

.
2


2

.
2


2

.
2


Hay

3 .
y chnh l bt ng thc quen thuc. Php chng minh hon tt.

BI O 14. Cho cc s thc dng , , . Chng minh rng
Bt ng Thc T Cc Cuc Thi Cc Quc Gia V Vng Lnh Th
This file was downloaded from VietNam Inequality Mathematic Olympiad Resource Page http://www.ddbdt.tk


China TST 2007
LI GII (VIMF). t

, , 0 th , , v do bt ng
thc cn chng minh c vit li thnh
3 3 3 3 3 3 3
2 a b c a b b c c a
Hay

3 6 8


Theo bt ng thc vi ch 3 0 th

3 6 4
T y p dng th 2


Php chng minh hon tt.

BI O 15. Cho cc s thc dng , , sao cho 1. Chng minh rng
1
2

2

1
2

2

1
2

2

1


UK TST 2007
LI GII 1 (VIMF, BI C ANH). T iu kin bi v theo bt ng thc ta c

1
2

1
2

1
2

1
2 2

1
2 2

1
2 2


2 2


2 2


2 2

2
2

2
2

2
2


1


Php chng minh hon tt.

LI GII 2 (V QUC B CN). Cng tng t nh li gii trn ta vit bt ng thc ny di dang
1
2 2

1
2 2

1
2 2

1


t

. Khi bt ng thc ny c vit li di dng

2 2


2 2


2 2

1


Theo bt ng thc , ta c

2 2


2 2


2 2

2 2 2 2 2 2

V vy, ta ch cn chng minh c

2 2 2 2 2 2
Khai trin ra, ta thy bt ng thc ny tng ng vi 6
Bt ng thc ny hin nhin ng theo bt ng thc . Php chng minh hon tt.

LI GII 3 ( BI C ANH). Cng nh cc li gii trn ta a bt ng thc v dng ng bc l

2 2


2 2


2 2
1

2 2
22 2

2 2
22 2

2 2
22 2
1


2


2


2


2


2


2
2
Theo bt ng thc th

2


2


2

2 2 2

1,
Bt ng Thc T Cc Cuc Thi Cc Quc Gia V Vng Lnh Th
This file was downloaded from VietNam Inequality Mathematic Olympiad Resource Page http://www.ddbdt.tk

2


2


2

2 2 2

1.
Cng 2 bt ng thc ny vi nhau ta thu c iu phi chng minh.

BI O 16. Cho cc s thc khng m , , . Chng minh rng
1


4


VietNam MO 2008
LI GII 1 (V THNH VN). Khng mt tnh tng qut, gi s , , .
Ta c

2 .
Nn theo bt ng thc ta c
1

2


4


4


Php chng minh hon tt.

LI GII 2 (VIMF). Khng mt tnh tng qut, gi s , 0. Khi

1

2 2 2 4
Php chng minh hon tt.

BI O 17. Cho , , l cc s thc dng. Chng minh rng
3 3
2
y x z
x y z
y z z x x y
| |
+ + + + |
|
+ + +
\

Olympic cc vng duyn hi Bc B 2009
LI GII (NGUYN ANH TUN). Theo bt ng thc th
( ) ( )
( ) ( )
2
6
2 2
2
( )
| |
+ +
+ + + |
|
+
\

x x y z
x x y z y z x
y z

Ta s chng minh
( )
( ) ( )
6
2
2
2
27
4


x
x x y z
( ) ( )
4
2
2 4 3 2 2
4 27 4 16 ( ) 30 6 ( ) x x y z x x y z y z xyz x y z + + + + + +


Bt ng thc ny ng v
3 2 2
15 ( ) 30 +

x y z y z v
4 3 2 2
4 ( ) 6 6 ( ) + + + +

x x y z y z xyz x y z
Php chng minh hon tt.
NHN XT. Bt ng thc ny vit dng tng ng l


33
2


Chun ha cho

1 th ta c bi ton
Chng minh rng vi

1 th


33
2
"
y chnh l bi trong mc Bt ng Thc Su Tm V Sng To.

BI O 18. Cho , , l cc s thc dng tha mn 1. Chng minh
2
2
xy yz zx
xy yz yz zx zx xy
+ +
+ + +

Bt ng Thc T Cc Cuc Thi Cc Quc Gia V Vng Lnh Th
This file was downloaded from VietNam Inequality Mathematic Olympiad Resource Page http://www.ddbdt.tk

CHINA TST 2006

LI GII (NGUYN ANH TUN). Bt ng thc s c chng minh nu ta c
1
(1)
2
xy yz zx
xy yz yz zx zx xy
+ +
+ + +

t
2 2 2
, , xy c yz a zx b = = = th (1) tr thnh
2 2 2
1
(2)
2
a b c
a b b c c a
+ +
+ + +

Ta c 1 suy ra 1
ab bc ca
c a b
+ + =
(2) tng ng vi
2 2 2
1
2
a b c ab bc ca
a b b c c a c a b
| |
+ + + +
|
+ + +
\

Ch rng
2 2 2 2 2 2
a b c a b c
a b b c c a a c b a c b
+ + = + +
+ + + + + +

Do ta cn chng minh
2 2
a b ab
a b c
+

+


Bt ng thc ny c chng minh bi O7. Php chng minh hon tt.

BI O 19. Cho cc s thc bt k , , , sao cho 0. Chng minh

12 6
International Zhautykov Olympiad 2009
LI GII (VIMF). Bt ng thc cho tng ng vi

12 6

12 6

12 6
Theo bt ng thc ta c

12
1
4

12

9
4

12

3
4


3
4


3
4

12 4

27
64

. 12

6| |
6
Php chng minh hon tt.

BI O 20. Cho cc s dng

, ,

(vi 3) sao cho

1. Chng minh rng


1
1


1
1


1
1

1
Russia 2004
LI GII. Do

1 nn t

, ,

, v bt ng thc c vit li thnh


1
1

1
1

1
1

1
Bt ng thc ny hin nhin ng v

1
Php chng minh hon tt.
Bt ng Thc T Cc Cuc Thi Cc Quc Gia V Vng Lnh Th
This file was downloaded from VietNam Inequality Mathematic Olympiad Resource Page http://www.ddbdt.tk


BI O 21. Cho cc s thc bt k , , . Chng minh rng


Russia All-Russian Olympiad 2009
LI GII 1 (VIMF). Khai trin bt ng thc ny ta c bt ng thc tng ng l

0
Hay

0
Php chng minh hon tt.

LI GII 2 (VIMF). Ta xt 4 trng hp
Trng hp 1. Nu , , 0. Khi


T y suy ra iu phi chng minh.
Trng hp 2. Nu , , 0 th lm tng nh nh trng hp 1

0,

0,

0
Trng hp 3. Nu c 2 s m v mt s dng, gi s , 0, 0. Khi

0,

3 0,

3 0
Trng hp 4. Nu c mt s m v 2 s dng, gi s 0, , 0. Khi

3 0,

0,

3 0
Php chng minh hon tt.

BI O 22. Cho , , 0 a b c tha mn
2 2 2
1 a b c abc + + + = . Chng minh rng
2 abc ab bc ca abc + + +
USA MO 2001
LI GII (minhhoang).
V th 2 kh n gin, ta i chng minh v th nht
Theo nguyn l Dirichlet th c 2 trong 3 s , , cng ln hn 1 hon cng nh hn1, gi s l , . Khi
( 1)( 1) 0 a b
T gi thit c
2 2 2 2 2 2 2 2
2 2 2 2
(4 )(4 ) (4 )(4 )
4 ( )
4 4 2 4 4
a b ab a b a b ab a b
c abc a b c c

+ + = + + = =
Bt ng thc cn chng minh
2 2 2 2
2 2 2 2 2 2
2 2 2 2
( (4 )(4 ) ) ( )( (4 )(4 ) )
2
2 2
4 (4 )(4 ) ( ) (4 )(4 ) ( ) 2
(4 )(4 )( 1) ( 1) 4 (4 )(4 ) 0
ab a b ab a b a b ab
ab
ab a b a b a b a b ab a b ab
a b ab a b ab ab a b ab a b
+
+ +
+ + + +
+ + +

2
2 2
2 2
2 2 2
2 2 2 2
4( )
( 1)( 1)( (4 )(4 ) ) 0
4 (4 )(4 )
4( 1)( 1)(4 ) 4( )
0
(4 )(4 ) 4 (4 )(4 )
a b
a b a b ab
ab a b
a b a b a b
a b ab ab a b

+
+

+
+ +

(ng do
2 2 2
4 0 a b c abc = + ). Php chng minh hon tt,

BI O 23. Cho , , a b c l cc s thc dng. Chng ming rng.
2 2 2
1
8 8 8
a b c
a bc b ca c ab
+ +
+ + +

IMO 2001
LI GII 1. Ta cn chng minh
Bt ng Thc T Cc Cuc Thi Cc Quc Gia V Vng Lnh Th
This file was downloaded from VietNam Inequality Mathematic Olympiad Resource Page http://www.ddbdt.tk

3 4
3 3 3 2 4 4 4
8
a a
a bc a b c

+ + +
( )
( )
2
3 3 3 3 4 4 4 2 2
8 a b c a a bc + + +
Tht vy, p dng bt ng thc ta c
( ) ( ) ( )( )
2 2
3 3 3 3 3 3 3 3 3 3 4 4 4 4 4 4 4 4 4 4
a b c a a b c a b c + + = + + + +
3 3 3 2 2 2 2
4 .2 8 . a bc b c a bc =
Vy
( ) ( )
( )
2 2
3 3 3 3 3 3 4 4 4 4 2 2 2
8 . 8 a b c a a bc a a bc + + + = +
T dn n
3 3 3 4 4 4
3 3 3 2 2 2 4 4 4
1
8 8 8
a b c a b c
a bc b ca c ab a b c
+ +
+ + =
+ + + + +

ng thc xy ra khi v ch khi a b c = =

LI GII 2. Ch n bt ng thc sau
( ) ( )( )( )
3
3 3 3 3 3 3
3 24 a b c a b c a b b c c a a b c abc + + = + + + + + + + + +
ng thc xy ra khi v ch khi a b c = = .
Tr li bi ton, t v tri bt ng thc cn chng minh l P . p dng bt ng thc Cauchy Schwarz ta c
( )
( )
2
2 2 2
8 8 8 a b c P a a bc a a bc a a bc + + + + + + +
( )( ) ( )
2
3 3 3
24 P a b c a b c abc P a b c + + + + + + +
T suy ra 1 P .

LI GII 3. t
( ) ( ) ( )
2 2 2
8 8 8 Q a a bc b b ca c c ab = + + + + +
p dng bt ng thc ta c ( )
3
2 3 3 3
24 P Q a b c a b c abc Q + + + + + =
T dn n 1 P .

LI GII 4. t
2 2 2
, ,
ab bc ca
x y z
c a b
= = = th 1 xyz = , bt ng thc tr thnh
1 1 1
1
8 1 8 1 8 1 x y z
+ +
+ + +

( )( )( )
( )
8( ) 2 8 1 8 1 8 1 8 1 8 1 8 1 510 x y z x y z x y z + + + + + + + + + + + +
p dng bt ng thc AM GM ta c.
3
3 3 x y z xyz + + = , ( )( )( ) ( ) ( )
6
8 1 8 1 8 1 1 512 8 64 729 3 x y z xyz x y z xy yz zx + + + = + + + + + + + =
( )( )( )
6
8 1 8 1 8 1 3 8 1 8 1 8 1 9 x y z x y z + + + + + + + +
T ta suy ra iu phi chng minh .

LI GII 5. t
3 3 3
2 2 2
, ,
ab bc ca
x y z
c a b
= = = th 1 xyz = , ta cn chng minh
3 3 3
1 1 1
1
8 1 8 1 8 1 x y z
+ +
+ + +

Theo bt ng thc AM GM th
( )( )
2
3 2 2
2 1 4 2 1
8 1 2 1 4 2 1 2 1
2
x x x
x x x x x
+ + +
+ = + + = +
Ta cn chng minh
2 2 2 2 2 2
2 2 2
1 1 1
1 3
2 1 2 1 2 1
x y y z z x
x y z
+ + + +
+ + +

Bt ng thc cui cng ng theo bt ng thc AM GM nn bi ton c gii quyt xong

LI GII 6. Ta s chng minh mt kt qu tng qut hn
2 2 2
3
1
a b c
k
a kbc b kca c kab
+ +
+
+ + +

Bt ng Thc T Cc Cuc Thi Cc Quc Gia V Vng Lnh Th
This file was downloaded from VietNam Inequality Mathematic Olympiad Resource Page http://www.ddbdt.tk

Vi k l s thc ty ( 8) k
t
2 2 2
a b c
P
a kbc b kca c kab
= + +
+ + +
,
( ) ( ) ( )
2 2 2
Q a a kbc b b kca c c kab = + + + + +
Tng t Li gii 3 ta chng minh c
( )
( )
3
3
2 2
3 3 3
3
a b c
P Q a b c P
a b c kabc
+ +
+ +
+ + +

V th ta cn chng minh
( )
( )( ) ( )( )( )( )
3
3 3 3
3 3 3
9
8 3 1 27
3 1
a b c
k a b c k a b b c c a kabc
a b c kabc k
+ +
+ + + + + + +
+ + + +

Bt ng thc cui cng ng theo bt ng thc AM GM , nn ta c pcm.

BI O 24. Cho , , a b c l cc s thc dng. Chng minh rng
( )
( ) ( ) ( )
2 2 2
9
4
a b c
a b c
b c c a a b
| |
| + + + +
|
+ + +
\

Darij Grinberg
LI GII 1. p dng bt ng thc Cauchy Schwarz ta c
( )
( )
2
3
2 2
a b c
a b c
b c c a a b ab bc ca
+ +
+ +
+ + + + +

ng thc xy ra khi v ch khi a b c = = .
Tip tc p dng bt ng thc Cauchy Schwarz , ta li c
( ) ( ) ( )
( )
2
2 2 2
9
4
a b c a b c
a b c
b c c a a b
b c c a a b
| |
| |
| + + + + + +
|
|
+ + +
\ + + +
\

T ta suy ra iu phi chng minh, ng thc xy ra khi v ch khi a b c = = .

LI GII 2. Chun ha 3 0 , , 3 a b c a b c + + = < < , bt ng thc tr thnh
( ) ( ) ( )
2 2 2
3
4
3 3 3
a b c
a b c
+ +


Ta c nhn xt nh sau: Vi mi s thc dng ( ) 0 3 t t < < th
( )
2
1 1
2 4
3
t
t
t


Bt ng thc ny l ng v n tng ng vi ( ) ( )
2
1 9 2 0 t t , ng thc xy ra khi v ch khi 1 t =
p dng vo bi, ta c

( ) ( ) ( )
( )
2 2 2
1 3 3
2 4 4
3 3 3
a b c
a b c
a b c
+ + + + =


Bi ton c chng minh xong.

BI O 25. Cho cc s dng , , x y z tha mn iu kin 3 x y z + + = . Chng minh rng
x y z xy yz zx + + + +
Russia 2002
LI GII. Bt ng thc cn chng minh tng ng vi
( )
2 2 2
2 9 x y z x y z + + + + +
p dng bt ng thc AM GM ta c
2 2 3
3 . 3 x x x x x x x + + =
V th
( )
( )
2 2 2
2 3 9 x y z x y z x y z + + + + + + + =
ng thc xy ra khi 1 x y x = = = .

BI O 26. Cho , , , a b c d l cc s thc dng. Chng minh rng
Bt ng Thc T Cc Cuc Thi Cc Quc Gia V Vng Lnh Th
This file was downloaded from VietNam Inequality Mathematic Olympiad Resource Page http://www.ddbdt.tk

4
a c b d c a d b
a b b c c d d a
+ + + +
+ + +
+ + + +

IMO Short List 1978
LI GII. p dng bt ng thc AM GM ta c
( )
( ) 4 1 1 a c a c c a
a c
a b c d a b c d a b c d
+ + + | |
+ = + +
|
+ + + + + + +
\

Tng t
( ) 4 b d b d d b
b c d a a b c d
+ + +
+
+ + + + +

Cng theo v hai bt ng thc ny li ta c iu phi chng minh. ng thc xy ra khi v ch khi , a c b d = =

BI O 27. Cho , , a b c l cc s dng. Chng minh rng
3 3 3 3 3 3
1 1 1 1
a b abc b c abc c a abc abc
+ + +
+ + + + + +

USAMO 1997
LI GII. Ta c nhn xt sau
( )
( ) ( )
2 2 3 3
3 3
1 1 1
2 a b ab a b ab a b
a b abc ab a b abc ab a b c
+ + + =
+ + + + + +

V th
3 3 3 3 3 3
1 1 1 1 1 1 1 1
a b abc b c abc c a abc a b c ab bc ca abc
| |
+ + + + =
|
+ + + + + + + +
\

ng thc xy ra khi v ch khi a b c = = .

BI O 28. Cho cc s dng , , x y z tho mn iu kin
2 2 2
3 x y z + + = . Chng minh rng
3
xy yz zx
z x y
+ +
France 2005
LI GII 1. Bnh phng hai v bt ng thc trn ta c

( ) ( )
2 2 2 2 2 2
2 2 2 2 2 2
2 2 2
2 2 3
x y y z z x
x y z x y z
z x y
| |
+ + + + + + +
|
\

Mt khc theo bt ng thc AM GM th
( )
2 2 2 2 2 2 2 2 2 2 2 2 2 2 2 2 2 2
2 2 2
2 2 2 2 2 2 2 2 2
2 2
x y y z z x x y y z y z z x z x x y
x y z
z x y z x x y y z
| | | | | | | |
+ + = + + + + + + +
| | | |
\ \ \ \

Bi ton c chng minh xong. ng thc xy ra khi v ch 1 x y z = = =

LI GII 2. p dng bt ng thc quen thuc ( ) ( )
2
3 a b c ab bc ca + + + + ,ta c
( )
2
2 2 2
3 9 3
xy yz zx xy yz zx
x y z
z x y z x y
| |
+ + + + = + +
|
\

T suy ra iu phi chng minh.

BI O 29. Cho cc s thc , , tha mn iu kin 1 xyz = . Chng minh rng
5 5 5 5 5 5
1
xy yz zx
x y xy y z yz z x zx
+ +
+ + + + + +

IMO Short List 1996
LI GII. Ta c nhn xt sau y

( ) ( )
2 2 5 5 2 2
5 5 2 2
1
2 ( )
1
xy xy z
x y xy x y x y x y
x y xy x y x y xy xy x y x y z
+ + + = =
+ + + + + + + +

V th
5 5 5 5 5 5
1
xy yz zx x y z
x y xy y z yz z x zx x y z
+ +
+ + =
+ + + + + + + +

Bt ng Thc T Cc Cuc Thi Cc Quc Gia V Vng Lnh Th
This file was downloaded from VietNam Inequality Mathematic Olympiad Resource Page http://www.ddbdt.tk


ng thc xy ra khi 1 x y z = = =

BI O 30. Cho cc s dng , , tha mn iu kin x y z . Chng minh rng:
2 2 2
2 2 2
x y y z z x
x y z
z x y
+ + + +
VMO 1991
LI GII. p dng bt ng thc ta c
2 2 2 2 2 2
2 2 2 2
( )
x y y z z x x z y x z y
x y z
z x y y z x
| || |
+ + + + + +
| |
\ \

Mt khc v x y z nn

( )( )( )( )
2 2 2 2 2 2
0
x y x z y z xy yz zx
x y y z z x x z y x z y
z x y y z x xyz
+ +
+ + =
T suy ra iu phi chng minh. ng thc xy ra khi v ch khi x y z = =

BI O 31. Cho , , l cc s dng khng nh hn 1 v tha mn iu kin
1 1 1
2
x y z
+ + =
Chng minh rng 1 1 1 x y z x y z + + + +
Iran 1998
LI GII. Ta c
1 1 1 1 1 1
2 1
x y z
x y z x y z

+ + = + + =
p dng bt ng thc Cauchy Schwarz ta c
( )
( )
2
1 1 1
1 1 1
x y z
x y z x y z x y z
x y z
| |
+ + = + + + + + +
|
\

Vy 1 1 1 x y z x y z + + + +
ng thc xy ra khi v ch khi
3
2
x y z = = = .

BI O 32. Cho a,b,c l cc s thc dng. Chng minh rng
( )
2 2 2
2 1 2 a b c abc ab bc ca + + + + + +
Darij Grinberg
LI GII 1. Theo nguyn l Dirchle th hai trong ba s ( ) ( ) ( ) 1 , 1 , 1 a b c s cng du, gi s l ( ) 1 a v
( ) 1 b . Khi ( )( ) 1 1 0 c a b .
( ) ( ) ( ) ( )( )
2 2
2 2 2
2 1 2 1 2 1 1 0 a b c abc ab bc ca a b c c a b + + + + + + = + +
ng thc xy ra khi v ch khi 1 a b c = = = .

LI GII 2. p dng bt ng thc AM GM v bt ng thc Schur ta c
( ) ( )
2
3 2 2 2
9
1 3 4
abc
abc abc a b c ab bc ca a b c
a b c
+ + + + + +
+ +

T dn n ( )
2 2 2
2 1 2 a b c abc ab bc ca + + + + + + .
LI GII 3. Khng mt tnh tng qut, gi s { } min , , c a b c = . t
( ) ( )
2 2 2
, , 2 1 2 f a b c a b c abc ab bc ca = + + + + + +
Xt hiu ( )
( ) ( )
2 2
, , , , 2 2 2 f a b c f ab ab c a b ab c a b ab = + +
( ) ( )
2
2 2 0 a b a b ab c = + +
Vy ( )
( )
( ) , , , , , , f a b c f ab ab c f t t c = vi t ab = .
Ta chng minh ( ) , , 0 f t t c
( ) ( ) ( ) ( )
2 2
2 2
2 1 2 2 1 0 1 2 1 0 c c c t t c c t + + + +
Bt ng Thc T Cc Cuc Thi Cc Quc Gia V Vng Lnh Th
This file was downloaded from VietNam Inequality Mathematic Olympiad Resource Page http://www.ddbdt.tk

Php chng minh hon tt

LI GII 4. t 1, 1, 1 a x b y c z = + = + = + vi , , 1 x y z > , bt ng thc tr thnh
2 2 2
2 0 x y z xyz + + +
V ( )
2
. . 0 xy yz zx xyz = nn trong ba s , , xy yz zx s c t nht mt s khng m, gi s l xy , suy ra
( )
2 2 2 2 2
2 2 2 2 1 0 x y z xyz xy z xyz xy z z + + + + + = + +
Bi ton c chng minh xong

BI O 33. Cho cc s dng a,b,c tha mn iu kin 1 abc = . Chng minh rng
( )
2 2 2
2 a b c a b c ab bc ca + + + + + + +
Moscou 2000
LI GII 1. p dng bt ng thc AM GM , Schur v gi thit 1 abc = ta c
( ) ( )
2
3 2 2 2 3
9
3 3 4
abc
a b c abc a b c ab bc ca a b c
a b c
+ + = + + + +
+ +

Suy ra ( )
2 2 2
2 a b c a b c ab bc ca + + + + + + +
ng thc xy ra khi v ch khi 1 a b c = = =

LI GII 2. V 1 abc = . Nn theo bt ng thc AM GM ta c
3
3 3 2 1 a b c abc abc + + = = +
Bt ng thc cn chng minh tng ng vi ( )
2 2 2
2 1 2 a b c abc ab bc ca + + + + + +
T kt qu ca bi ton 9 ta c iu phi chng minh.

LI GII 3. t , , p a b c q ab bc ca r abc = + + = + + = , bi ton tr thnh
2
4 p p q +
Theo bt ng thc Schur th
3
3
9
4 9 0 4
p
p pq r q
p
+
+
Ta chng minh
3
2 2
9
9
p
p p p
p
+
+
Bt ng thc cui cng ng theo bt ng thc AM GM , php chng minh hon tt.

BI O 34. Cho , , a b c l cc s thc dng. Chng minh rng
( )( )( ) ( )
2 2 2
2 2 2 9 a b c ab bc ca + + + + +
APMO 2004
LI GII 1. Khai trin bt ng thc trn, ta cn chng minh
( ) ( ) ( )
2 2 2 2 2 2 2 2 2 2 2 2
2 4 8 9 a b c a b b c c a a b c ab bc ca + + + + + + + + +
Theo bt ng thc AM GM ta c
2 2 2
a b c ab bc ca + + + +
( ) ( ) ( ) ( )
2 2 2 2 2 2
1 1 1 2 a b b c c a ab bc ca + + + + + + +
Mt khc theo bt ng thc Schur ta li c
( ) ( )
2
3 2 2 2 2 2 2
9
1 3 4
abc
a b c a b c ab bc ca a b c
a b c
+ + + + +
+ +

Cng theo v cc bt ng thc ny li ta thu c
( ) ( ) ( )
2 2 2 2 2 2 2 2 2 2 2 2
2 2 3 4 a b c a b b c c a a b c + + + + + + + +
( ) ( ) ( ) ( )
2 2 2
2 4 3 9 ab bc ca ab bc ca a b c ab bc ca + + + + + + + + + +
ng thc xy ra khi v ch khi 1 a b c = = =
LI GII 2. Ta t
2
b c
t
+
= . Khi
( )( ) ( )
2
2 2 2 2 2 2 2 2 2 2
2 2 1 2 2 3 2 2 2 3 3 b c b c b c bc b c b c b c + + = + + + + + + + = + + + + ( )
2 3
3
2
b c + +
Do ta cn phi chng minh
( ) ( ) ( )
2
2
3
2 3 9
2
a b c ab bc ca
| |
+ + + + +
|
\
( )
( )( ) ( )
2
2
2 2 6 b c a ab bc ca + + + + +
Bt ng Thc T Cc Cuc Thi Cc Quc Gia V Vng Lnh Th
This file was downloaded from VietNam Inequality Mathematic Olympiad Resource Page http://www.ddbdt.tk

p dng bt ng thc Cauchy Schwarz ta c
( )
( )( ) ( )
( )
( ) ( )
2
2 2
2
2 2 2 2 2 6 b c a b c a a b c ab bc ca + + + + + = + + + +

Php chng minh hon tt

LI GII 3. Ta chng minh mt bt ng thc mnh hn nh sau
( )( )( ) ( ) ( )
2 2
2 2 2
2 2 2 3 1 a b c a b c abc + + + + + +
Bt ng thc tng ng vi
( ) ( ) ( )
2 2 2 2 2 2 2 2 2
2 4 2 7 9 a b b c c a a b c abc ab bc ca + + + + + + + + +
Theo bt ng thc AM GM th
( ) ( ) ( ) ( )
2 2 2 2 2 2
2 1 2 1 2 1 4 a b b c c a ab bc ca + + + + + + +
( ) ( )
2 2 2
3 3 a a a ab bc ca + + + +
V th ta ch cn chng minh
2 2 2
2 1 2( ) a b c abc ab bc ca + + + + + +
V theo kt qu bi 1.9 ta c iu phi chng minh

LI GII 4. Ta chng minh mt kt qu mnh hn
( )( )( ) ( )
2
2 2 2
2 2 2 3 a b c a b c + + + + +
Khai trin bt ng thc trn ta c
( ) ( )
2 2 2 2 2 2 2 2 2 2 2 2
2 8 6 a b c a b b c c a a b c ab bc ca + + + + + + + + +
Theo bt ng thc AM GM ta c
( ) ( ) ( ) ( )
2 2 2 2 2 2
2 1 2 1 2 1 4 a b b c c a ab bc ca + + + + + + +
V th ta ch cn chng minh ( )
2 2 2 2 2 2
2 2 a b c a b c ab bc ca + + + + + +
Theo nguyn l Dirichlet th hai trong ba s
( ) ( ) ( )
2 2 2
1 , 1 , 1 a b c s cng du, gi s l
( )
2
1 a v
( )
2
1 b . Khi
( )( )
2 2 2 2 2 2 2 2 2 2 2
1 1 0 c a b a b c c b c c a + + .
Nn ( )
2 2 2 2 2 2 2 2 2 2 2 2
2 1 1 2 a b c a b c a b b c c a ab bc ca + + + + + + + + + + +
Php chng minh c hon tt.

BI O 35. Cho , , a b c l cc s thc dng. Chng minh rng
( )( )( ) ( )
3
5 2 5 2 5 2
3 3 3 a a b b c c a b c + + + + +
Titu Andresscu, USAMO 2004
LI GII. Ta c
( ) ( ) ( )( )
5 2 3 2 3 5 2 3
3 2 1 1 0 3 2 a a a a a a a a + + = + +
Tng t
5 2 3 5 2 3
3 2, 3 2 b b b c c c + + + +

Nhn theo v ba bt ng thc ny li ta c
( )( )( ) ( )( )( )
5 2 5 2 5 2 3 3 3
3 3 3 2 2 2 a a b b c c a b c + + + + + +
Vy ta cn phi chng minh
( )( )( ) ( )
3
3 3 3
2 2 2 a b c a b c + + + + +
Theo bt ng thc AM GM ta c
( )( )( )
3
3 3 3
3 3 3
3
1 1 3
2 2 2
2 2 2
a a
a b c
a b c
+ +
+ + +
+ + +

V th

( )
( )( )( )
( )( )( ) ( )
3
3 3 3
3 3 3
3
3
3 2 2 2
2 2 2
a b c
a b c a b c
a b c
+ +
+ + + + +
+ + +

Php chng minh hon tt. ng thc xy ra khi 1 a b c = = =

BI O 36. Cho , , a b c l cc s thc dng. Chng minh rng
Bt ng Thc T Cc Cuc Thi Cc Quc Gia V Vng Lnh Th
This file was downloaded from VietNam Inequality Mathematic Olympiad Resource Page http://www.ddbdt.tk

2 2 2
3 3 3
2 2 2
3
a b c
b c c a a b
| | | | | |
+ +
| | |
+ + +
\ \ \

MOSP 2002
LI GII. p dng bt ng thc AM GM ta c
2 2
3 3
1 3 3
2 2 2 2
b c b c b c a b c b c
a a a a a
+ + + + + + | | | |
+ +
| |
\ \
2
3
2 3 a a
b c a b c
| |

|
+ + +
\

T o dn n
( )
2 2 2
3 3 3
3 2 2 2
3
a b c a b c
b c c a a b a b c
+ +
| | | | | |
+ + =
| | |
+ + + + +
\ \ \

ng thc xy ra khi a b c = =

BI O 37. Cho cc s dng , , a b c tha mn iu kin 1 ab bc ca + + = . Chng minh rng
3 3 3
1 1 1 1
6 6 6 b c a
a b c abc
+ + + + +
IMO Short List 2004
LI GII. p dng bt ng thc Holder ta c
( ) ( )
3
3 3 3
1 1 1 1 1 1
6 6 6 1 6 1 6 1 6 1 1 1 b c a ab bc ca
a b c a b c
| |
| |
+ + + + + + + + + + + + + +
|
|
|
\
\

3
3
3 3 3 3 3 3
27
27 27. . . 1 3
ab bc ca
ab bc ca
abc a b c a b c abc
+ + | |
|
| | \
= = =
|
\

T suy ra iu phi chng minh. ng thc xy ra khi
1
3
a b c = = =

BI O 38. Cho , , a b c l cc s thc dng. Chng minh rng
( )
( )
( )
( )
( )
( )
2 2 2
2 2 2
2 2 2
3
5
a b c b c a c a b
a b c b c a c a b
+ + +
+ +
+ + + + + +

Japan 1997
LI GII 1. Chun ha 3 a b c + + = , khi bt ng thc tr thnh
( )
( )
( )
( )
( )
( )
2 2 2
2 2 2
2 2 2
2 3 2 3 2 3
3
5
3 3 3
a b c
a a b b c c

+ +
+ + +


2 2 2
1 1 1 3
2 6 9 2 6 9 2 6 9 5 a a b b c c
+ +
+ + +

Ta c nhn xt nh gi: Vi mi s thc dng x ta lun c bt ng thc ( ) ( )
2
1 2 1 0 x x + hay l
2
1 2 3
2 6 9 25
x
x x
+

+
. ng thc xy ra khi 1 x =
p dng vo bi ton
( )
2 2 2
2 9 1 1 1 3
2 6 9 2 6 9 2 6 9 25 5
a b c
a a b b c c
+ + +
+ + =
+ + +

Bi ton c gii quyt xong. ng thc xy ra khi v ch khi a b c = =

LI GII 2. t , ,
a b b c c a
x y z
c a b
+ + +
= = = , khi ta c hai bt ng thc sau y ( ) 2 xy yz zx x y z + + + +

v 6 x y z + +
Bt ng thc th nht kh quen thuc, cn bt ng thc th hai th sau mt s php bin i ta thu c
( )( ) ( )( ) ( )( ) 0 a a b a c b b c b a c c a c b + + , y chnh l bt ng thc Schur bc mt, nn bt ng
thc trn ng
Bt ng Thc T Cc Cuc Thi Cc Quc Gia V Vng Lnh Th
This file was downloaded from VietNam Inequality Mathematic Olympiad Resource Page http://www.ddbdt.tk

Tr li bi ton, ta cn chng minh
( ) ( ) ( )
2 2 2
2 2 2
1 1 1 3
1 1 1 5
x y z
x y z

+ +
+ + +

Theo bt ng thc Cauchy Schwarz th
( ) ( ) ( ) ( )
2 2 2 2
2 2 2 2 2 2
1 1 1 3
1 1 1 3
x y z x y z
x y z x y z
+ +
+ +
+ + + + + +

Ta chng minh
( )
2
2 2 2
3 3
3 5
x y z
x y z
+ +

+ + +
( ) ( ) ( )
2
15 3 18 x y z x y z xy yz zx + + + + + + + +
Theo nhn xt trn th
( ) ( ) ( ) ( ) ( ) ( )( )
2 2
15 3 18 9 18 6 3 0 x y z x y z ab bc ca x y z x y z x y z x y z + + + + + + + + + + + + + = + + + +
T suy ra iu phi chng minh.

BI O 39. Cho cc s dng , , x y z tha mn iu kin 1 xyz . Chng minh rng
5 2 5 2 5 2
5 2 2 5 2 2 5 2 2
0
x x y y z z
x y z y z x z x y

+ +
+ + + + + +

Hojoo Lee - IMO 2005
LI GII 1. Bt ng thc tng ng vi
5 2 2 5 2 2 5 2 2 2 2 2
1 1 1 3
x y z y z x z x y x y z
+ +
+ + + + + + + +

Ta s chng minh
( )
( )
2 2
2 5 2 2
2 2 2
3
1
2
y z
x y z
x y z
+

+ +
+ +

T gi thit 1 xyz ta suy ra
4 4 5 2 2
2 2 2 2
2 2
1 1 1
2 x x x y z
y z y z
yz y z

+ +
+ + + +
+

p dng bt ng thc Cauchy Schwarz ta c

( )
( )
( )
2 2
2 2
2 2 4
2
2 2 2 2 2 2 2
4 2 2 2 2 2
2 2
2 2
3
2 1
2 2 2
y z
y z x
y z y z x y z
x y z x y z
y z
y z
| || |
| | | | +
+
| |
+ + + + + + | |
| | | |
+ + +
\ \
+ +
\ \
+

Vy
( )
( )
2 2
2 5 2 2
2 2 2
3
1
2
y z
x y z
x y z
+

+ +
+ +

V th
( )
( )
2 2 2
2 5 2 2 5 2 2 5 2 2 2 2 2
2 2 2
6
1 1 1 3
2
x y z
x y z y z x z x y x y z
x y z
+ +
+ + =
+ + + + + + + +
+ +

ng thc xy ra khi v ch khi 1 x y z = = =

LI GII 2. p dng bt ng thc Cauchy Schwarz ta c

( ) ( )
( )
2 2
2
5 2 2 2 2 2 2 2
2 5 2 2
2 2 2
1
1 1
y z
x
x y z y z x y z
x x y z
x y z
+ +
| |
+ + + + + +
|
+ +
\
+ +

T o dn n
( )
( )
2 2 2
2 5 2 2 5 2 2 5 2 2
2 2 2
1 1 1
2
1 1 1
x y z
x y z
x y z y z x z x y
x y z
+ + + + +
+ +
+ + + + + +
+ +

Bt ng Thc T Cc Cuc Thi Cc Quc Gia V Vng Lnh Th
This file was downloaded from VietNam Inequality Mathematic Olympiad Resource Page http://www.ddbdt.tk

Mt khc t gi thit 1 xyz , ta suy ra
2 2 2
1 1 1
xy yz zx x y z
x y z
+ + + + + +
Nn
( )
( ) ( )
2 2 2
2 2
2 2 2 2 2 2
1 1 1
2
3
x y z
x y z
x y z x y z
+ + + + +

+ + + +

Bi ton c chng minh xong.

LI GII 3. Bt ng thc cho tng ng vi
2 2 2 2 2 2 2 2 2
5 2 2 5 2 2 5 2 2
3
x y z x y z x y z
x y z y z x z x y
+ + + + + +
+ +
+ + + + + +

p dng bt ng thc Cauchy-Schwarz v 1 xyz . Ta c
( )
( )
( )( )
2 2 2 2 2
2
2
2 2 2 5 2 2 5 2 2 2 2
5 2 2 2 2 2
x y z yz y z
x y z x yz y z x y z yz y z
x y z x y z
+ + + +
+ + + + + + + +
+ + + +

V th
2 2 2 2 2 2 2 2 2
5 2 2 5 2 2 5 2 2 2 2 2
2 3
x y z x y z x y z xy yz zx
x y z y z x z x y x y z
+ + + + + + + +
+ + +
+ + + + + + + +

T ta thu c iu phi chng minh.

BI O 40. Cho cc s dng , , a b c tha mn iu kin 1 a b c + + = . Chng minh rng
1 1 1
2
1 1 1
a b c a b c
b c a a b c
+ + + | |
+ + + +
|

\

Japan TST 2006
LI GII. Bt ng thc tng ng vi
3
2
a b c a b c
b c a b c c a a b
+ + + + +
+ + +


3
2
a a b b c c
b b c c c a a a b
| | | | | |
+ +
| | |
+ + +
\ \ \

( ) ( ) ( )
3
2
ac ba cb
b b c c c a a a b
+ +
+ + +

p dng bt ng thc Cauchy-Schwarz ta c
( ) ( ) ( )
( )
( )
2
2 2 2 2 2 2
1 3
2 2
ab bc ca
ac ba cb a c b a c b
b b c c c a a a b abc b c c a a b abc a b c
+ + | |
+ + = + +
|
+ + + + + + + +
\

Bi ton c chng minh xong. ng thc xy ra khi
1
3
a b c = = =

BI O 41. Cho cc s dng , , a b c tha mn iu kin 3 a b c + + = . Chng minh rng
2 2 2
2 2 2
1 1 1
a b c
a b c
+ + + +
Romania TST 2006
LI GII 1. Bt ng thc cn chng minh tng ng vi
( )
2 2 2
1 1 1
2 9 ab bc ca
a b c
+ + + + +
p dng bt ng thc AM GM v Cauchy Schwarz ta c
( ) ( )
2
2 2 2
1 1 1 1 1 1
2 2 ab bc ca ab bc ca
a b c a b c
| |
+ + + + + + + + + + =
|
\

( ) ( )
( ) ( )
2
2 2 2
3 3
3
1 1
3 3 9
3 3
ab bc ca abc a b c
ab bc ca
ab bc ca ab bc ca
abc abc abc
| |
+ + | + + |
+ + | |
| + + + + + + =
| |
|
\
\
\

Php chng minh hon tt. ng thc xy ra khi 1 a b c = = =

LI GII 2. Ta cng chng minh
Bt ng Thc T Cc Cuc Thi Cc Quc Gia V Vng Lnh Th
This file was downloaded from VietNam Inequality Mathematic Olympiad Resource Page http://www.ddbdt.tk

( )
2 2 2
1 1 1
2 9 ab bc ca
a b c
+ + + + +
p dng bt ng thc AM GM v Cauchy Schwarz ta c
( ) ( ) ( )
2 2 2
1 1 1 1 1 1
2 2 3 2 3
a b c
ab bc ca abc a b c abc a b c
a b c ab bc ca abc
+ +
+ + + + + + + + + + = + + +
1
3 9 abc abc
abc
| |
= + +
|
\

LI GII 3. Ta c
( )
( )
2 2 2 2
1 1 1 1 1 1 9 27 a b c
a b c ab bc ca abc abc a b c
ab bc ca
+ +
+ + + + = =
+ +
+ +

Vy ta cn chng minh
( )
( )( )
2
2 2 2 2 2 2
2
27
27 a b c a b c ab bc ca
ab bc ca
+ + + + + +
+ +

p dng bt ng thc AM GM
( )( ) ( )( )( )
2
2 2 2 2 2 2
a b c ab bc ca a b c ab bc ca ab bc ca + + + + = + + + + + +
( )
3
2
2 2 2
27
3 3
a b c
a b c ab bc ca ab bc ca
| |
+ + | | + + + + + + + +
| = =
|
|
\
\


BI O 42. Cho , , a b c l cc s thc dng. Chng minh rng
( )
( )
( )
( )
( )
( )
2 2 2
2 2 2
2 2 2
2 2 2
8
2 2 2
a b c b c a c a b
a b c b c a c a b
+ + + + + +
+ +
+ + + + + +

Titu Andresscu , Zuming Feng USAMO 2003

LI GII 1. Chun ha 3 a b c + + = . Khi bt ng thc tr thnh
( )
( )
( )
( )
( )
( )
2 2 2
2 2 2
2 2 2
3 3 3
8
2 3 2 3 2 3
a b c
a a b b c c
+ + +
+ +
+ + + +

Ta c nhn xt : Vi mi s thc dng x ta lun c bt ng thc
( )
( )
2
2
2
3
4 4
2 3
x
x
x x
+
+
+

Tht vy
( )
( ) ( )
2
2
2 2 2
2
3
6 9 8 6 8 6
1 1 4 4
2 3 2
2 3 1 2
x
x x x x
x
x x
x x x
+
+ + +
= = + + = +
+
+ +

ng thc xy ra khi 1 x =
p dng vo bi ton ta c
( )
( )
( )
( )
( )
( )
( )
2 2 2
2 2 2
2 2 2
3 3 3
8 4 3 8
2 3 2 3 2 3
a b c
a b c
a a b b c c
+ + +
+ + + + + =
+ + + +

Php chng minh hon tt, ng thc xy ra khi a b c = =


LI GII 2. Ch rng
( )
( )
( )
( )
2 2
2 2
2 2
2 2
3
2 2
a b c b c a
a b c a b c
+ + +
=
+ + + +

V th bt dng thc cn chng minh tng ng vi
( )
( )
( )
( )
( )
( )
2 2 2
2 2 2
2 2 2
2 2 2
1
2 2 2
b c a c a b a b c
a b c b c a c a b
+ + +
+ +
+ + + + + +

Mt khc theo bt ng thc Cauchy Schwarz th ( ) ( )
2
2 2
2 a b a b + + , nn
Bt ng Thc T Cc Cuc Thi Cc Quc Gia V Vng Lnh Th
This file was downloaded from VietNam Inequality Mathematic Olympiad Resource Page http://www.ddbdt.tk

( )
( )
( )
( )
( )
( )
( ) ( ) ( )
2 2 2 2 2 2
2 2 2 2 2 2
2 2 2
2 2 2
2 2 2
b c a c a b a b c a b c b c a c a b
a b c
a b c b c a c a b
+ + + + + + + +
+ +
+ +
+ + + + + +

Ta cn chng minh ( ) ( ) ( )
2 2 2
2 2 2
a b c b c a c a b a b c + + + + + + +
y l bt ng thc ng v
( ) ( ) ( ) ( ) ( ) ( ) ( )
2 2 2 2 2 2
2 2 2
0 a b c b c a c a b a b c a b b c c a + + + + + + + = + +
Php chng minh hon tt

LI GII 3. Chun ha 1 a b c + + = , khi bt ng thc tr thnh
( )
( )
( )
( )
( )
( )
2 2 2
2 2 2
2 2 2
1 1 2 1
8
2 1 2 1 2 1
a b c
a a b b c c
+ + +
+ +
+ + +

Ta c nh gi sau Vi mi s thc dng x th
( )
( )
( )
2
2
2
1
1
4 1
3
2 1
x
x
x x
+
+ +
+

Tht vy ta c
( )
( )
( ) ( )
( )
( )
2
2 2 2
2
1 2 4 1 2 4 1
1 1 1
4 1
3 9 6 3 3 3
2 1 3 1 2
x x x
a
x x
x x x
+ + +
= + = + + +
+
+ +

ng thc xy ra khi
1
3
x =
p dng vo bi ta c
( )
( )
( )
( )
( )
( )
( ) ( )
2 2 2
2 2 2
2 2 2
3 1 4 3 1 1 2 1
8
3
2 1 2 1 2 1
a b c a b c
a a b b c c
+ + + + + + +
+ + =
+ + +

Php chng minh hon tt.

BI O 43. Cho , , a b c l di ba cnh ca tam gic. Chng minh rng
2 2 2
1 1 1 0
b c a
a b c
c a b
| | | | | |
+ +
| | |
\ \ \

Vasile Cirtoaje , Moldova TST 2006
LI GII. Bt ng thc tng ng vi
( )
3 2 3 2 3 2 2 2 2
a b b c c a abc a b c + + + +
Ta ch n hng ng thc
( ) ( ) ( ) ( ) ( ) ( )
3 3 3
2 3 3 2 3 3 2 3 3 2 2 2
a b c b c a c a b a b c b c a c a b + + = + +
Khi
( ) ( )
3 2 3 2 3 2 2 2 2
2 2 a b b c c a abc a b c + + + + =
( ) ( ) ( ) ( ) ( ) ( )
( ) ( ) ( ) ( ) ( ) ( )
( ) ( ) ( ) ( ) ( ) ( )
3 2 2 3 2 2 3 2 2 2 3 3 2 3 3 2 3 3
2 2 2 3 3 3
3 3 3 2 2 2
2 2 2
2 2 2
2 2 2
0
a b bc c b c ca a c a ab b a b c b c a c a b
a b c b c a c a b a b c b c a c a b
a b c a b c b c a b c a c a b c a b
= + + + + +
= + +
= + + + + +

Bi ton c chng minh xong. ng thc xy ra khi a b c = =

BI O 44. Cho , , a b c l cc s thc dng. Chng minh rng
( ) ( ) ( )
{ }
2 2 2
3
, ,
3
a b c
abc Max a b b c c a
+ +

Titu Andresscu USA TST 2000
LI GII. Ch rng
( ) ( ) ( )
( ) ( ) ( )
{ }
2 2 2
2 2 2
, ,
3
a b b c c a
Max a b b c c a
+ +


V th ta a bi ton v chng minh mt bt ng thc mnh hn l

( ) ( ) ( )
( )
2 2 2
3 3
3 2
3 3
a b b c c a
a b c
abc a b c abc ab bc ca
+ +
+ +
+ + + + +
t ( )
3 3 3
, , , , 0 a x b y c z x y z = = = > bt ng thc tr thnh
Bt ng Thc T Cc Cuc Thi Cc Quc Gia V Vng Lnh Th
This file was downloaded from VietNam Inequality Mathematic Olympiad Resource Page http://www.ddbdt.tk

( )
3 3 3
3 2 x y z xyz xy xy yz yz zx zx + + + + +
Theo bt ng thc Schur v AM GM th
( ) ( ) ( )
( )
3 3 3
3 2 x y z xyz xy x y yz y z zx z x xy xy yz yz zx zx + + + + + + + + + +
Php chng minh hon tt, ng thc xy ra khi x y z = = , tc a b c = =

BI O 45. Cho a,b,c l cc s thc dng tha mn
1 1 1
1
1 1 1 a b b c c a
+ +
+ + + + + +

Chng minh rng a b c ab bc ca + + + +
Anderei Ciupan
LI GII 1. Theo bt ng thc Cauchy Schwarz ta c
( ) ( )( )
2
2
1 a b c a b c a b + + + + + +
T dn n
( ) ( ) ( )
2 2 2
2 2 2
1 1 1
1
1 1 1
a b c b c a c a b
a b b c c a
a b c a b c a b c
+ + + + + +
+ + + +
+ + + + + +
+ + + + + +

( ) ( )
2
2 2 2
2 a b c a b c a b c ab bc ca a b c + + + + + + + + + + +
Bi ton c chng minh xong. ng thc xy ra khi 1 a b c = = =

LI GII 2. T gi thit v bt ng thc Cauchy Schwarz , ta c
1 1 1
2 1 1 1
1 1 1 1 1 1
a b b c c a
a b b c c a a b b c c a
+ + + | | | | | |
+ + = + +
| | |
+ + + + + + + + + + + +
\ \ \

( ) ( ) ( ) ( )
( )( ) ( )( ) ( )( )
2
1 1 1
a b b c c a
a b a b b c b c c a c a
+ + + + +

+ + + + + + + + + + +

T y suy ra
( ) ( ) ( ) ( ) ( )
2 2 2 2 2 2
2 a b c ab bc ca a b c a b c ab bc ca + + + + + + + + + + + + + ab bc ca a b c + + + +
Php chng minh hon tt.

BI O 46. Cho cc s dng , , a b c tha mn iu kin 1 a b c + + = . Chng minh rng
3
2
a bc b ca c ab
a bc b ca c ab
+ + +
+ +


Canada 2008
LI GII. Ta c
( )( )
2 2
1
1 1 1
a bc bc bc
a bc b c bc b c
+
= =
+
, nn bt ng thc cn chng minh tng ng vi
( )( ) ( )( ) ( )( )
( )( )
2 2 2 3
9 9
1 1 1 1 1 1 2
ab bc ca
ab bc ca abc a b c ab bc ca abc
a b b c c a
+ + + + + + + +


Bt ng thc cui cng dung theo bt ng thc AM GM , nn ta suy ra iu phi chng minh, ng thc
xy ra khi
1
3
a b c = = =

BI O 47. Cho , , l cc s dng tha mn iu kin x y z xyz + + = . Chng minh rng
2 2 2
1 1 1 3
2
1 1 1 x y z
+ +
+ + +

Korea 1998
LI GII. t
1 1 1
, , a b c
x y z
= = = th 1 ab bc ca + + = , bt ng thc tr thnh
2 2 2
3
2
1 1 1
a b c
a b c
+ +
+ + +

Bt ng Thc T Cc Cuc Thi Cc Quc Gia V Vng Lnh Th
This file was downloaded from VietNam Inequality Mathematic Olympiad Resource Page http://www.ddbdt.tk

p dng bt ng thc AM GM ta c
( )( )
2 2
1
2
1
a a a a a
a b a c
a b a c a a ab bc ca
| |
= = +
|
+ +
\ + + + + + +


T dn n
2 2 2
1 3
2 2
1 1 1
a b c a b b c c a
a b b c c a
a b c
+ + + | |
+ + + + =
|
+ + +
\ + + +



BI O 48. Cho cc s dng , , a b c tha mn iu kin 1 abc = . Chng minh rng
( )( )( ) ( ) 4 1 a b b c c a a b c + + + + +
MOSP 2001
LI GII. Bt ng thc cn chng minh tng ng vi
( ) ( ) ( ) ( ) 4 6 0 ab a b ab a b ab a b a b c + + + + + + + +
t v tri bt ng thc cn chng minh l ( ) , , f a b c , ta cn chng minh ( ) , , 0 f a b c . Khng mt tnh tng
qut, gi s a b c th 1 a , khi
( )
( ) ( )( ) ( ) ( )
2 2 2
, , , 2 2 4 2 f a b c f a ab ab a bc b c bc a b c bc b c bc = + + + + + +
( )
( )( )
( ) ( )
2 2
2 4 2 0 b c a b a c a a b c = + + +
Vy ( )
( )
, , , f a b c f a ab ab
Cui cng ta ch cn chng minh bi ton trong trng hp b c = , lc
2
1
b
a
= v bt ng thc thc tr thnh
( ) ( )
2
2 2 1 b a b a b + +
Thay
2
1
b
a
= vo bt ng thc ta c
( ) ( ) ( ) ( )
2 2
2
3 4 3 2 3 2
1 2 2 1 1 2 0 b b b b b b b b
(
+ + + +
(


Php chng minh hon tt, ng thc xy ra khi 1 a b c = = =

BI O 49. Cho , , x y z l cc s thc dng v , a b l cc s dng cho trc. Chng minh rng
( )( ) ( )( ) ( )( )
( )
2 2 2
2
3 x y z
ay bz az by az bx ax bz ax by ay bx
a b
+ +
+ + + + + +
+

Olympic 30/4

LI GII. Theo bt ng thc AM GM th
( )( )
( )
( ) ( )
( )( )
( ) ( )
2
2 2 2
2 2
2
2 2
2
4 2
a b y z
ay bz az by x x
ay bz az by
ay bz az by
a b y z
+ +
+ + +
+ +
+ +
+ +

T dn n
( )( ) ( )( ) ( )( )
( ) ( )
2 2 2 2 2 2
2 2 2 2 2 2 2 2
2 3 x y z x y z
ay bz az by az bx ax bz ax by ay bx y z z x x y
a b a b
| |
+ + + +
|
+ + + + + + + + +
+ +
\

Bi ton c chng minh xong, ng thc xy ra khi x y z = =

BI O 50. Cho cc s dng , , a b c tho mn iu kin 1 abc = . Chng minh rng
1 1 1 1 1 1
1 1 1 2 2 2 a b b c c a a a a
+ + + +
+ + + + + + + + +

Bulgaria 1997
LI GII. t
1 1 1
, x a b z y
a b c
= + + = + + th 3, 3 x y khi bt ng thc tr thnh
Bt ng Thc T Cc Cuc Thi Cc Quc Gia V Vng Lnh Th
This file was downloaded from VietNam Inequality Mathematic Olympiad Resource Page http://www.ddbdt.tk

2
2 2 2 2
2
3 4 12 4
3 6 5 24 3 27 0
2 9 4 2
x y x x y
x y xy xy x y x y
x y x xy x y
+ + + + +
+ +
+ + + + +
( )
2 2 2
5 1 1 3 1 12 1 3 1 1 3 9 0
3 3 9 9 3 3
y x xy xy y x
x y y x x y xy
| | | | | | | | | | | |
+ + + + + +
| | | | | |
\ \ \ \ \ \

(ng do 3, 3 x y )
Bi ton c chng minh xong. ng thc xy ra khi 3 x y = = tc 1 a b c = = =

BI O 51. Cho cc s thc bt k , , . Chng minh rng


Gabriel, Mathematical Relfections 2009
LI GII (VIMF). Chia 2 v bt ng thc cho

th bt ng thc c vit li

1 3


Ch rng


Bt ng thc c vit li thnh
1 1 1 3 3 4 2 8
0 2 2 2 0

0
Bt ng thc trn lun ng. Php chng minh hon tt.

BI O 52. Cho cc s dng , , v s t nhin 1. Chng minh rng


Poland 2009
LI GII (VIMF). Ta vit li bt ng thc cho di dng


Ta s chng minh rng


Tht vy, sau khi quy ng rt gn bt ng thc trn ta c bt ng thc tng ng l


Hay

0
Bt ng thc ny hin nhin ng v hon tt chng minh rng, ta cn c


Bt ng Thc T Cc Cuc Thi Cc Quc Gia V Vng Lnh Th
This file was downloaded from VietNam Inequality Mathematic Olympiad Resource Page http://www.ddbdt.tk

Bt ng thc ny ng theo bt ng thc Holder. Php chng minh hon tt


BI O 53 (VietNam TST 2010). Cho , , l cc s thc dng tha mn iu kin
1 1 1
16( ) a b c
a b c
+ + + + .
Chng minh rng
1
2


1
2


1
2


8
9

LI GII. Ta c
( ) ( ) ( )( )
3
3
27
2
2 2 2
( | | | |
+ +
(
+ + + = + + + + +
( | |
| |
( \ \
a c a c
a b a c a b a b a c
Suy ra
( )
( )( )
( )
( )( )( )
3
4 1 2
27 27
2
(
| |
+ +
(
|
=
(
| + + + + +
+ + +
\
(


cyc
a b c
a b a c a b b c c a
a b a c

Do ta cn chng minh rng
( )
( )( )( )
( )( )( ) ( )
4 8
6
27 9
+ +
+ + + + +
+ + +
a b c
a b b c c a a b c
a b b c c a

T iu kin bi ta c
( ) ( )
2 16 3
16
3 16
+ + + + + + + + ab bc ca abc a b c ab bc ca ab bc ca
Vy
( )( )( ) ( )( ) ( ) ( )( )( ) ( )
3
9 8 6
2
+ + + + + + + + + + + + + + a b b c c a a b c ab bc ca a b c a b b c c a a b c
Php chng minh hon tt. ng thc xy ra khi v ch khi

.
























Bt ng Thc T Cc Cuc Thi Cc Quc Gia V Vng Lnh Th
This file was downloaded from VietNam Inequality Mathematic Olympiad Resource Page http://www.ddbdt.tk


Cc Cuc Thi Bt ng Thc
This file was downloaded from VietNam Inequality Mathematic Olympiad Resource Page http://www.ddbdt.tk

Cc Cuc Thi Bt ng Thc

The VIMF 1
ST
Anniversary Contest

Cuc thi K nim 1 nm din dn bt ng thc Vit Nam (The VietNam Inequality Contest For Junior
2009) do din n bt ng thc Vit Nam t chc cho cc bn hc sinh t lp 8 n 11. Di y l thi
v p n

Bi ton 1. Cho , , 0 a b c tha mn 1 a b c = . Chng minh
1 1 1 15
2
ab bc ca
ab c bc a ca b
+ + +
+ +
+ + +

T Minh Hong
Li gii. Bt ng thc cho tng ng vi
1 1 1 9 1 1 1 9
1 1 1
2 2
ab bc ca c a b
ab c bc a ca b ab c bc a ca b
+ + + | | | | | |
+ + + +
| | |
+ + + + + +
\ \ \

Ch rng
( ) ( )( ), bc a bc a a b c a b a c = =
( ) ( )( ), ca b ca b a b c b a b c = =
( ) ( )( ), ab c ab c a b c c a c b = =
Nn bt ng thc c vit li thnh
2 2 2
1 1 1 9 9
(1 ) (1 ) (1 ) ( )( )( )
( )( ) ( )( ) ( )( ) 2 2
a b c
a b c a b b c c a
a b a c b c b a c a c b

+ + + + + + +
+ + + + + +

p dng bt ng thc Cauchy-Schwarz,c
2 2 2 2
1 4
(1 ) (1 ) (1 ) (3 ) ,
3 3
a b c a b c + + =
3
9 (2( )) 4
( )( )( ) .
2 6 3
a b c
a b b c c a
+ +
+ + + =
Php chng minh hon tt. ng thc xy ra khi v ch khi
1
3
a b c = = = .


Bi ton 2. Cho cc s thc dng , , x y z . Chng minh rng
2 2 2
5 2 2 5 2 2 5 2 2
1 1 1 3
2 2 2
x y z
x x y y y z z z x



Nguyn nh Thi
Li gii. Ta vit li bt ng thc thnh
2 2
2 2 2 2
2
2 2
4 2 2 4 2 2 4 2 2
3
2 2 2
x z
y z x y
y
x z
x x y y y z z x z



p dng bt ng thc Bunhiacopxki Cauchy Schwarz ta c
2
2 2
2 2 2 2
2
2 2
4 2 2 4 2 2 4 2 2 4 2 2 4 2 2 4 2 2
2 2 2 2 2 2
xy zx yz x z
y z x y
z y x y
x z
x x y y y z z x z x x y y y z z x z
1



( )


( )
( )
2 2 2
2 2 2
4 4 4 2 2 2 2 2 2
3 . . .
3
3
3( 2
xy zx zx yz yz xy
x y z
z y y x x z
x y z
x y z x y y z z x
1



( )
= =



Php chng minh hon tt. ng thc xy ra khi v ch khi x y z = = .

Cc Cuc Thi Bt ng Thc
This file was downloaded from VietNam Inequality Mathematic Olympiad Resource Page http://www.ddbdt.tk

Bi ton 3. Cho cc s thc khng m , , a b c sao cho
3 3 3
2 a b c + + = . Tm gi tr nh nht ca
3 3 3
2 2 2 2 2 2
a b c
P
b bc c c ca a a ab b
= + +
+ + +

Nguyn nh Thi
Li gii. Khng mt tnh tng qut, gi s min{ , , } c a b c = th
2 2
( ) 0; ( ) 0 bc c c b c ac c c a c = =
Li c
3 2
3 3 3 3 3
( ) ( )
2 2; 1
4 4
a b a b
a b c a b a b ab
+ +
= + + + + =


Do

3 3 3 3 3 3 3
2 2 2 2 2 2 2 2 2 2
2 a b c a b a b
b bc c c ca a a ab b b a a ab b

+ + + +
+ + + +


( )
2 2 2
3 3
2 2 2 2 2 2 2 2
( ) ( )
2 2
a b a b a ab b
a b
a b
b a a ab b a b a ab b
+ + +
= + + = +
+ +


2 2
2
2 2 2 2 2 2 2 2
( ) .2. .3 2 6( ) 2 2
6( )
a b ab ab a b
a b
a b a ab b a ab b a ab b ab

+ = + +
+ + +


2 2 2
2 2 2 2
2 1
2( ) 2 2 2.2 2 2 a b a ab b ab
a ab b a ab b
| |
+ = + + =
|
+ +
\

Vy gi tr nh nht ca
3 3 3
2 2 2 2 2 2
a b c
b bc c c ca a a ab b
+ +
+ + +

l 2, t c khi v ch khi mt bin bng
khng v 2 bin cn li bng nhau v bng 1.
Nhn xt. Ta c th vit li bt ng thc trn di mt dng tng ng l
( )
3 3 3
3 3 3
3
2 2 2 2 2 2
4
a b c
a b c
b bc c c ca a a ab b
+ + + +
+ + +


Bi ton 4. Cho cc s thc dng , , x y z . Chng minh rng
( ) ( ) ( )
2 2 2
2 2 2 2 2 2 2 2 2
x y y z z x xy yz zx
x y z z x xy y x y yz z y z zx x




Nguyn nh Thi
Li gii 1. Theo bt ng thc Schwarz ta c
( ) ( ) ( ) ( ) ( ) ( )
2 2
2 2
2 2
2 2 2
2 2 2 2 2 2 2 2 2 2 2 2
z x
x y y
x y y z z x y
z
z x xy y x y yz z y z zx x y x xy y z y yz z x x
z
z x
x
z
=


( ) ( ) ( )
( )
( )( )
2
2
3 3 3 3 3 3
2 2 2 2 2 2 2 2 2
xy yz zx
x y y z z x
z x y
y x xy y z y yz z x z zx x xyz x y z x y z
1





( )
=


Nn ta cn chng minh rng
( )
2
3 3 3 3 3 3
( )( ) x y y z z x xyz x y z xy yz zx
V
2
1
( ) ( )
3
xyz x y z xy yz zx nn ta chng minh rng bt ng thc cht hn
( )
2
3 3 3 3 3 3 3
1
( )
3
x x y y z z x y yz zx
t , , xy a yz b zx c = = = th bt ng thc trn tr thnh
( ) ( )
2 3
3 3 3 2 2 2
3 a b c a b c
Bt ng thc ny c nhiu cch chng minh bng Cauchy hay Holder. Phn chng minh rng ny xin dnh
cho cc bn.

Li gii 2. p dng bt ng thc Schwarz ta c
( ) ( ) ( )
2
2 2 2 2 2 2
( ) xy yz zx
VT
yz x xy y zx y yz z xy z zx x



Cc Cuc Thi Bt ng Thc
This file was downloaded from VietNam Inequality Mathematic Olympiad Resource Page http://www.ddbdt.tk

Do ta ch cn chng minh
( )( ) ( )
2 2 2 3 3 3
2 x y z xy yz zx xyz x y z x y y z z x
Hay
2 2 2
3 3 3
( )
x y z
xy yz zx xyz x y z x y z
y z x

Bt ng thc ny hin nhin ng theo bt ng thc Schwarz.
Php chng minh hon tt.

Bi ton 5. Cho cc s thc dng , , a b c . Chng minh rng
( ) ( )
3 3 3
3
3 3 3
1 1 1 1 1 1
1 5 a b c a b c
a b c a b c
1 1




( ) ( )

Nguyn nh Thi
Li gii. Theo bt ng thc Bunhiacopxki, ta c ( )
3
3 1 1 1
a b c
a b c
1


( )

( )
3 3 3
3 3 3
1 1 1 1 1 1 1 1 1
3( )( )( ) 3 a b c a b b c c a
a b c a b b c c a
1 1 1 1

=

( )( )( ) ( )
( )
3 3 3
3 3 3 2 2 2
1 1 1 3( )( )( )
3( )( )( )
a b b c c a
a b c a b b c c a
a b c a b c
1


=


( )

( )
3 3 3
3 3 3
1 1 1 3( )( )( ) a b b c c a
a b c
a b c abc
1



( )

( ) ( ) ( )
3 3 3
3 3 3
1 1 1 3( )( ) 1 1 1 1 1 1
3 3 3
a b c ab bc ca
a b c a b c a b c
a b c abc a b c a b c
1 1 1

=


( ) ( ) ( )

( ) ( ) ( )
3 3 3
3 3 3
1 1 1 1 1 1 1 1 1
3 3 6 a b c a b c a b c
a b c a b c a b c
1 1 1




( ) ( ) ( )

(v ( )
1 1 1
9 a b c
a b c
1


( )
)
Do
( ) ( )
3
3 3 3
3 3 3
1 1 1 1 1 1
1 5 a b c a b c
a b c a b c
1
1 1



( ) ( )
( )

Hay
( ) ( )
3 3 3
3
3 3 3
1 1 1 1 1 1
1 5 a b c a b c
a b c a b c
1 1




( ) ( )

Php chng minh hon tt. ng thc xy ra khi v ch khi a b c = = .

Bi ton 6. Cho cc s thc dng bt k , , , , , a b c x y z v s nguyn dng . Chng minh rng
1 1 1
2 2 2
k k k
k k k
x y z
a b b c c a a b c
x y y z z y



Nguyn nh Thi

Bi ton 7. Cho cc s thc khng m , , sao cho 2. Chng minh rng
( )
2 2 2 2 2 2 2 2 2 2 2 2
1 1 1
3
( )( ) ( )( ) ( ) a ab b b bc c c ca a a ab b b bc c c ca a



Nguyn nh Thi
Li gii. Khng mt tnh tng qut, gi s min{ , , } c a b c = . Khi
2 2
0, 0 c bc c ca .
Do
Cc Cuc Thi Bt ng Thc
This file was downloaded from VietNam Inequality Mathematic Olympiad Resource Page http://www.ddbdt.tk

2 2 2 2
1 1 1 1
b a
b bc c c ca a



Mt khc
( )
( )
2 2
2 2 2 2
1 2
( )
( )
ab bc ca ab bc ca
a ab b ab bc ca
a ab b a ab b ab bc ca

=


( ) ( )
( )
2
2 2
( ) 2
2
ab bc ca ab bc ca ab bc ca ab
a b a b c ab a b ab a b ab
a b c a b ab

= = =



Do
( )
2 2 2 2 2 2 2 2 2 2 2 2
1 1 1
( )( ) ( )( ) ( ) a ab b b bc c c ca a a ab b b bc c c ca a


( )
2 2 2 2 2 2 2 2 2 2
1 1 1 1
( ) a ab b b bc c c ca a b bc c c ca a
1


=



( )
1 1 1 1 1 1
1
ab ab a b ab ab
a b ab a b ab a b ab ab ab ab a b ab ab
1 1


= =


( ) ( )
( )
3
2 2
3 3
1 3 3
3 3
2 2
ab a b c
ab a b ab a b c


1 1




( ) ( )

Vy, ta c iu phi chng minh. ng thc xy ra khi v ch khi 1, 0 a b c = = = v cc hon v.

Bi ton 8. Cho cc s thc bt k , , a b c sao cho a b c abc = . Tm gi tr nh nht v gi tr ln nht ca
2 2 2
1 1 1
a b c
P
a b c
=


Nguyn nh Thi
LI GII 1. T iu kin bi ton ta c th t , ,
x y y z z x
a b c
x y y z z x

= = =


Khi
( ) ( ) ( )
2 2
2
2 2 2 2 2 2
2 2 2 2 2 2 2 2 2
2 2 2
1
1 1
x y y z
z x
x y y z x y y z z x z x
P
x y y z z x z x
x y y z
z x
x y y z
1 1
1





( ) ( ) ( )
= =
1 1 1






( )
( ) ( )

Mt khc p dng ng thc . .
x y y z z x x y y z z x
x y y z z x x y y z z x

=


Ta c
( )( )( )
( )( )( )
2 2 2 2 2 2
2 2 2 2 2 2
2
x y y z z x
P
x y y z z x

=


n y ta c 2 cch phn tch :
-
( )( )( ) ( )( )( ) ( )
2 2 2 2 2 2 2 2 2 2 2 2 2 4 2 4 2 4 2 2 2
2 x y y z z x x y y z z x x y y z z x x y z =
( )( )( )
2 2 2 2 2 2
x y y z z x
-
( )( )( ) ( )( )( ) ( )
2 2 2 2 2 2 2 2 2 2 2 2 4 2 4 2 4 2 2 2 2
2 x y y z z x x y y z z x x y y z z x x y z =
( )( )( )
2 2 2 2 2 2
x y y z z x
Do
1 1
2 2
P
Vy
1
2
minP= t c khi ( ) , , ( 1, 1,1) a b c = v cc hon v ; v
1
2
maxP= t c khi
( ) , , ( 1,1,1) a b c = v cc hon v
Cc Cuc Thi Bt ng Thc
This file was downloaded from VietNam Inequality Mathematic Olympiad Resource Page http://www.ddbdt.tk


LI GII 2. t , , p a b c q ab bc ca r abc = = = . T iu kin bi th p r = . Khi
( )( ) ( )( ) ( )( )
( )( )( )
2 2 2 2 2 2
2 2 2
1 1 1 1 1 1
1 1 1
a b c b c a c a b
P
a b c

=


( ) ( )( )
( ) ( ) ( ) ( )
2 2
2 2 2
3
2 2 1
abc ab bc ca a b c ab bc ca abc a b c
a b c ab bc ca abc a bb c a b c ab bc ca

=


2 2 2 2 2
3 4
2 2 1 2 4 1
qr pq r p r
r q pr p q q q r

= =


D on
1
2
minP= , t c khi ( ) , , ( 1, 1,1) a b c = v cc hon v. Hay ta chng minh
2 2
2 2
4 1
2 4 8 1 0
2 4 1 2
r
q q r r
q q r




T bt ng thc ng( ) ( ) ( )
2 2 2
0 ab c bc a ca b , ta suy ra
2 2 2 2 2 2 2 2 2
6 0 a b b c c a a b c abc
( ) ( ) ( ) ( )
2 2
2 2 6 0 ab bc ca abc a b c a b c ab bc ca abc
2 2 2 2
2 2 6 0 2 3 6 0 q pr p q p q q r r
Do ( )
2
2 2 2
2 4 8 1 2 1 1 0 q q r r r r r =
Vy, ta c iu phi chng minh.
D on
1
2
maxP= , t c khi ( ) , , ( 1,1,1) a b c = v cc hon v. Hay ta chng minh
2 2
2 2
4 1
2 4 8 1 0
2 4 1 2
r
q q r r
q q r




T bt ng thc ng( ) ( ) ( )
2 2 2
0 ab c bc a ca b , ta suy ra
2 2 2 2 2 2 2 2 2
6 0 a b b c c a a b c abc
( ) ( ) ( ) ( )
2 2
2 2 6 0 ab bc ca abc a b c a b c ab bc ca abc
2 2 2 2
2 2 6 0 2 3 6 0 q pr p q p q q r r
Do ( )
2
2 2 2
2 4 8 1 2 1 1 0 q q r r r r r =
Vy, ta c iu phi chng minh.









K thi Olympic bt ng thc VIC 2009
K thi Olympic bt ng thc VIC 2009 (VietNam Inequalities Contest) do din n bt ng thc Vit
Nam t chc kt thc tt p. Di y l nhng bi thi v nhng li gii c o m chng ti tuyn
Cc Cuc Thi Bt ng Thc
This file was downloaded from VietNam Inequality Mathematic Olympiad Resource Page http://www.ddbdt.tk

chn v do cc bn bu chn c sp xp theo th t tnh hay v p ca mi bi ton. Nu bn c nhng
li gii mi hay nhng m rng, tng qut cho nhng bi ton ny c th post trc tip ln forum
http://www.ddbdt.tk/ hoc gi qua hm th vif.vimf@gmail.com

BI VIC O 1. Cho cc s thc khng m , , . Tm gi tr nh nht ca biu thc

1
2


1
2


1
2


Nguyn nh Thi
LI GII (VIC STAFF VIC STAFF VIC STAFF VIC STAFF). Khng mt tnh tng qut, gi s , , . Khi
1
2


1
4

4 0 ng
1
2

4 0 ng
V , , 0 nn
Do

1
2


1
4



2 1

,
vi


Xt


2 1


1
2
, 0,


2 1

42 1
2 1


1
4

0 8

12

34

43

24 4 0
Dng my tnh b ti hoc kim tra th thy phng trnh ch c 3 nghim gn ng l

2,097878850loi v

0,2626067385

2,515979352


T , lp bng bin thin, ta thy hm s i du t sang khi i qua

. Nhng v


5,038581952, 3 1,181220153
Vy,

1,181220153. t c khi v ch khi


0

v cc hon v.

BI VIC O 2. Cho cc s thc bt k , , . Chng minh rng
1
3


1
3


1
3


21
26


Nguyn nh Thi
LI GII 1 (VIC STAFF VIC STAFF VIC STAFF VIC STAFF).
t 3 , 3 , 3 th

, , 0.
Suy ra

6
52

Do


1
3


1
3


1
3

6
52

1


V ta ch cn chng minh bt ng thc sau
7

6
1

42
Cc Cuc Thi Bt ng Thc
This file was downloaded from VietNam Inequality Mathematic Olympiad Resource Page http://www.ddbdt.tk

Theo nguyn l th 2 trong 3 s , , cng du. Khng mt tnh tng qut, gi s 0.
Khi
1


Mt khc
7

6 4

6
2

6 5

6 7


Do
7

6
1

6 7


Ta c
5

6 7

56

48


47
5

6
56


48

47
vi


Xt
5

6
56


48

47,

10 6
118


48


Dng my tnh b ti hoc tnh

0 th ta c 2 nghim gn ng duy nht l


1,6014463,

2,017203138
T lp bng bin thin ta thy hm s i du 2 ln t sang khi i qua

, nhng ta thy

.
Do min

42,07707339 42. T y, ta c suy ra iu phi chng minh.



LI GII 2 (BI C ANH). Cng nh li gii trn ta phi chng minh
2 2 2 2 2 2 2
1 1 1 42
3( ) 4( ) x y z x y z x y z
+ +
+ + + + +


Do tnh i xng, c th gi s rng , 0. Ni cch khc ta phi chng minh bt ng thc sau
, ,
2 2 2 2 2 2 2
1 1 1 42
3( ) 4( ) x y z x y z x y z
+ +
+ + + +
0
Xt 2 trng hp
Trng hp 1. Nu ,ta chng minh , , , ,
Xt
( )( )
2 2 2 2 2 2 2 2 2
( , , ) ( , , ) 21( 7 )
0
( ) 2 ( ) 3( ) 4(
f x y z f x y x y x y z x y z
z x y z x y x y x y z x y x y z
+ + + + +
=
+ + + + + + +

Bt ng thc trn ung do
7 4( ), x y z x y z + + + +
2 2 2
2( ) ( ), x y x y x y + + + +
2 2 2 2 2
11 3( ) 4( ) z z x y x y z + + +
Ngoi ra d thy
2 2 2
2 2 2
1 1 1
4( ( ) ) 54
( )
x y x y
x y x y
| |
+ + + + +
|
+
\
, , 0
Trng hp 2. Nu .Khi , , , , vi
2
x y +

, ,
2 2 2 2 2
2 1 42
3(2 ) 4(2 ) t z t z t z
+
+ +

Do bt ng thc ng bc nn ta c th ly 1 quy ng mu s ta c
4 3 2
20 12 5 24 14 0 t t t t + + vi t
1
2

Kim tra trc tip ta suy ra bt ng thc trn ng

Cc Cuc Thi Bt ng Thc
This file was downloaded from VietNam Inequality Mathematic Olympiad Resource Page http://www.ddbdt.tk

NHN XT. Vi li gii 1 trn ta hon c th gii quyt c bi ton tng qut
Tm gi tr nh nht ca


vi cc s thc bt k , , , , , v 0
+ 3, 1, 0 ta c bt ng thc trn
+ 2, 1, 0 l mt bt ng thc kh hay ca


1
2


1
2


1
2


11
7


BI VIC O 3. Cho cc s thc dng , , sao cho 3. Chng minh rng

0
Nguyn nh Thi
LI GII (VIC STAFF VIC STAFF VIC STAFF VIC STAFF). Bt ng thc tng ng vi

1 3
Hay

3
Theo bt ng thc th


Suy ra


Tng t ta cng c


Cng cc bt ng thc trn v theo v ta c


Mt khc, theo bt ng thc vi ch rng

2
2 2, ta c


4
1

Ngoi ra t gi thit 3 ta d dng suy ra
4
1
3
Vy, ta c iu phi chng minh. ng thc xy ra khi v ch khi 1.

BI VIC O 4. Cho cc s thc khng m , , sao cho 1. Chng minh rng
3 1


3 1


3 1

4
Nguyn nh Thi
LI GII 1 (VIC Staff, BI C ANH). Bt ng thc cho tng ng vi
3


3


3

4
Hay
4


4


4

4
Ch rng
Nn theo bt ng thc th



1


Do
Cc Cuc Thi Bt ng Thc
This file was downloaded from VietNam Inequality Mathematic Olympiad Resource Page http://www.ddbdt.tk

4


4


4


4

4
Php chng minh hon tt. ng thc xy ra khi v ch khi , , 0; 1; 1 v cc hon v

LI GII 2 (NGUYN CH HIU, NGUYN C TON, NGUYN HI KHNH).
Bt ng thc cho tng ng vi
[ ]
2
2
2 2 2
2
(3 1)( )( ) 4( )( )( )
(3 1)( ) 4 ( )( )
(3 1)(1 ) 4( ) 4
( ) 3( ) 3 ( ) 3 4( ) 4
( ) 2( ) 3 3 ( ) 3 4(
ab b c c a a b b c c a
ab ab bc ac c a b c ab bc ac abc
ab c a b c abc
a b c ab ac bc abc a b c a b c abc
a b c ab ac bc abc a b c
+ + + + + +
+ + + + + + + +
+ + + +
+ + + + + + + + + + +
+ + + + + + + + +

2
2
2
) 4
( ) (3 3 3 4) 4( ) 4 0
( ) 4( ) 4 (3 3 3 4) 0
( 2) (3 3 3 4) 0
a b c abc
a b c abc a b c a b c
a b c a b c abc a b c
a b c abc a b c
+ +
+ + + + + + + + +
( + + + + + + + + +

+ + + + + +


Php chng minh hon tt.

LI GII 3 (L QUANG HUY).
D on ng th xy ra ti bin vi 1, 0 v cc hon v. Vi 0 th 1. V ta c

4


1


4

4
V by gi ta ch cn dn mt bin v 0 l hon thnh chng minh.
t ( )
3 1 3 1 3 1
, ,
ab bc ca
f a b c
a b b c c a

=

. Ta s dn bin ( )
1
, , ; ;0 f a b c f a b
a b
1


( )

Ch rng ( )
3 1 3 1 3 1 1 1 1
4 3
ab bc ca
a b c
a b b c c a a b b c c a
1


=


( )

Khng mt tnh tng qut, gi s , , . Xt hiu
, , ;
1

; 0
4
1


1


1

3 4
1

1


1


3
1

4
1


1



1

3
1


4
1

1


1

1



1


3

4
1
1


1
1


1
1

1
3


c 0 th ta ch vic chng minh
1
1


1
1


1
1

1
2


21

0
T gi thit , , v 1,th 21 2

. Suy ra 0 v php chng minh hon tt.



BI VIC O 5. Cho cc s thc bt k , , . Chng minh rng
3

4 1


Nguyn nh Thi
LI GII 1 (VIC STAFF VIC STAFF VIC STAFF VIC STAFF). Khai trin bt ng thc cho ta c bt ng thc tng ng l
5

23 8 8
Cc Cuc Thi Bt ng Thc
This file was downloaded from VietNam Inequality Mathematic Olympiad Resource Page http://www.ddbdt.tk

Theo nguyn l th 2 trong 3 s

1,

1,

1 cng du. Khng mt tnh tng qut, gi


s

1 0, suy ra

1 0. Hay

. Do ta ch cn
chng minh bt ng thc sau
5

23 8 8
Bt ng thc ny tng ng vi bt ng thc ng sau
4 1

4 1

4 1

3 1

4 1

4 1

0
D nhin bt ng thc ny l lun ng
Vy, ta c iu phi chng minh. ng thc xy ra khi v ch khi 1.

LI GII 2 (NGUYN VIT HNG). t , 1. Ta c
3

24

0 ng
23

0 ng
Nhn 2 bt ng thc trn vi nhau, v theo v ta c
3

4 2 2

4 1



LI GII 3 (BI C ANH). p dng bt ng thc ta c
( )
2 2 2 2 2 2 2 2 2
( 3)( 3) ( 1)( 1) 2( ) 8 ( ) ( ) 8 2 ( ) 4 a b a b a b a b a b a b + + = + + + + + + + + + = + +

( )
2 2 2 2 2
( 3)( 3)( 3) 2 ( ) 4 ( 3) a b c a b c + + + + + +
Ta cn chng minh
( )
2 2
2 ( ) 4 ( 3) a b c + + + 4
2
( 1) a b c + + +
( )
2 2
( ) 4 ( 3) a b c + + + 2
2
( 1) a b c + + +
Li p dng bt ng thc ta c
( )
2 2
( ) 4 ( 3) a b c + + + =
( )
2 2
( ) 2 2 (2 1 ) a b c + + + + +
( )
2
( ) 2 2 2 a b c + + + = 2
2
( 1) a b c + + +
T y suy ra iu phi chng minh.

LI GII 4 (NGUYN QUANG HUY). t

. Theo bt ng thc ta c

3 9

1 3

8 2 3

8 2

8 8


Ta cn chng minh

38 8

4 2 1

iu ny tng ng vi 2 1

2 10

0
Bt ng thc trn lun ng. Vy ta c iu phi chng minh.

LI GII 5 (PHAN PHNG C). Bt ng thc cn chng minh tng ng vi
5

23 8 8

4 1

4 1

4 1

7 11 0
Ta c

7 1
V ta ch cn chng minh 0.
D nhn thy rng nu mt trong ba, hoc hai trong ba hoc c ba s , , l m th bt ng thc s yu
hn. V th ta s chng minh trong trng hp , , 0
t 1, 1, 1 , , 1
Ta c
1 1 1 3 1

7 1 1 1 11
3


Theo nguyn l th 2 trong 3 s , , cng du. Khng mt tnh tng qut, gi s 0
Ta c 1 0 1
V d thy 3

0
T y ta suy ra iu phi chng minh.

BI VIC O 6. Cho cc s thc bt k , , sao cho
1

8

1
3

Tm gi tr nh nht v gi tr ln nht ca
Nguyn nh Thi
LI GII 1 (VIC STAFF VIC STAFF VIC STAFF VIC STAFF). Ta s s dng b sau
B . Vi cc s thc bt k , , th
Cc Cuc Thi Bt ng Thc
This file was downloaded from VietNam Inequality Mathematic Olympiad Resource Page http://www.ddbdt.tk

2 3


Chng minh. Khai trin bt ng thc ny ta c bt ng thc tng ng l

8 6
Theo nguyn l th 2 trong 3 s

1,

1,

1 cng du. Khng mt tnh tng qut, gi


s

1 0, suy ra

1 0. Hay


Do ta ch cn chng minh bt ng thc sau l

8 6
Bt ng thc trn tng ng vi bt ng thc ng sau

2 1

3 1

3 1

0
T ta suy ra iu phi chng minh. ng thc xy ra khi v ch khi 1.
Tr li vi bi ton.
T gi thit v theo bt ng thc th
1

8

1
6

1
6

8

1
6

8

1
3

8
.

8

Tng t ta cng c 2 bt ng thc na l
1

8

1
3

8
.

8
,
1

8

1
3

8
.

8

T , nhn cc bt ng thc trn, v theo v v rt gn ta suy ra

2 27
Do , theo b trn th ta c
3

2 27
Hay

9 3 3
Vy, gi tr nh nht ca l 3, t c ti 1 ; v gi tr ln nht ca l 3,
t c ti 1.

LI GII 2 (PHAN PHNG C).
Nhn xt. Khi ta i du , , th iu kin ra vn khng thay i nn gi tr nh nht v gi tr ln nht ca
l 2 gi tr i nhau v t max khi , , 0 v t min khi , , 0. Do ta ch cn tm max
vi , , 0
u tin xt b : vi , , 0 v 3 th
1

8

1
3

Tht vy, ta chng minh bt ng thc ng sau
1

8

21
81

1
9

vi 3
Bt ng thc trn tng ng vi

1
9

21
81
0 19 1 2

8 0 12

9 7 0
1

7 2 0
D thy bt ng thc trn ng vi 0 3
Tng t cho 2 bt ng thc cn li i vi b v c,cng 3 bt ng thc ny li v theo v ta c
1

8

1

8

2
81
3
1
3

1
3

Vy b c chng minh
Tr li bi ton. Gi s tn ti b b s , , tha mn 3. Xt 3 1. Khi

1
Theo b , ta c
1

8

1
3

1

8

1

8

iu ny v l bi vi 1 th
Cc Cuc Thi Bt ng Thc
This file was downloaded from VietNam Inequality Mathematic Olympiad Resource Page http://www.ddbdt.tk

1

8

1

8


LI GII 3 (NGUYN VIT HNG).
t ||, ||, ||. Khi ch rng || || || | |. Do tm gi tr
nh nht v gi tr ln nht ca ta ch cn tm gi tr ln nht ca , , , 0
T iu kin bi ta suy ra
1

8

1
3

8

1
3

1
8

1
8

1
12

Nn

4 0 2. Tng t ta cng c 0 , 2.
Li t iu in ta c
1

8

1
9

1

1
9

1
9
0
1

8
0
1 .
1

8
1 .
1

8
1 .
1

8
0
Khng mt tnh tng qut, gi s th 1 1 1 .
Li do2 0 nn
1

8

8

8
0
1

8

1

8

Tng t ta cng c
1

8

1

8

V ta thit lp c 2 dy bt ng thc ngc chiu l

1 1 1
1

8

1

8

1


Nn theo bt ng thc th
0 1 .
1

8
1 .
1

8
1 .
1

8

1
3
3
1

8

Suy ra 3
Nh theo suy lun ban u ta c th thy gi tr nh nht v gi tr ln nht ca l 3 v 3.

BI VIC O 8. Cho cc s thc khng m , , sao cho . Chng minh rng


Nguyn nh Thi
LI GII (VIC STAFF VIC STAFF VIC STAFF VIC STAFF).
Bt ng thc cho tng ng vi

2
Hay


Theo bt ng thc th


nn ta ch cn chng minh


Hay


Bt ng thc ny tng ng vi 0
Cc Cuc Thi Bt ng Thc
This file was downloaded from VietNam Inequality Mathematic Olympiad Resource Page http://www.ddbdt.tk

ng theo iu kin 0 u bi.
Vy, bi ton c chng minh xong. ng thc xy ra khi v ch khi hoc , 0 v cc hon
v tng ng.

BI VIC O 9. Cho cc s thc phn bit , , v s thc bt k 0; 1. Chng minh rng


7
8

Nguyn nh Thi
LI GII (VIC STAFF VIC STAFF VIC STAFF VIC STAFF). t

th
1 1 1 1 1 1 1
Ta c

1
4

1
4
1

2 3 3

1
4
1

2
1
1
5
1
1

1
4
1
1
1

4 4
1

4 4
4 1

Do vic cn li l chng minh

4 4
4 1

7
8
2

8 8 7 7 12 1 0
Bt ng thc ny hin nhin ng vi 0; 1
Vy, ta c iu phi chng minh. ng thc xy ra khi v ch khi 1. Khi



BI VIC O 9. Cho cc s thc , , 1 sao cho 1. Chng minh rng
3
1


3
1


3
1


16
1 1 1

47
16

Nguyn nh Thi
LI GII (VIC STAFF VIC STAFF VIC STAFF VIC STAFF). Ta s s dng b sau
B . Vi mi s thc bt k , , th


.





.





.



8

1
Chng minh xin dnh cho bn c
Tr li vi bi ton.
Do 1 nn t

. Khi bt ng thc c vit li thnh


3


16
1


47
16

Hay
( ) ( ) ( )
( )( )( )
2 2 2
2 2 2
3 3 3 16 47
16
a ab b bc c ca abc
a b b c c a
a b b c c a





By gi p dng b trn vi mt cht phn tch bin i n gin ta c
( ) ( ) ( )
( )( )( )
2 2 2
2 2 2
3 3 3 16 a ab b bc c ca abc
a b b c c a
a b b c c a





( ) ( ) ( ) ( ) ( ) ( )
( )( )( )
2 2 2
2 2 2 2 2 2
3 3 3 16 a b c ab bc ca abc
a b b c c a
a b b c c a a b b c c a
=



Cc Cuc Thi Bt ng Thc
This file was downloaded from VietNam Inequality Mathematic Olympiad Resource Page http://www.ddbdt.tk

( ) ( )
( )
( ) ( )
( )
( ) ( )
2 2 2 2 2 2 2 2 2
2 2 2
3 1
1 1 1
4 4
a b a b b c b c c a c a a b b c c a
a b b c c a c a
a b b c
l l
1 1 1
l l
=
l l
( ) ( ) ( )
l l
l l

( )( )( ) ( )( )( )
2 2 2
16 16 3
2
abc a b b c c a abc a b b c c a
a b b c c a a b b c c a a b b c c a a b b c c a
1 1 1 1


=



( ) ( ) ( ) ( )
2 2 2
3 3
2 . . . 2
2 2
a b b c c a a b b c b c c a c a a b a b b c c a
a b b c c a a b b c b c c a c a a b a b b c c a
1 1 1 1 1

=


( ) ( ) ( ) ( ) ( )

2 2
3 3 7 3 7 9 47
2 2 2 4 2 16 16
a b b c c a a b b c c a a b b c c a
a b b c c a a b b c c a a b b c c a
1 1 1

= =


( ) ( ) ( )

Vy, bi ton c chng minh xong.

LI GII 2 (NGUYN C TON).
t 1 ;1 ;1 ; , , x a y b z c a b c p ab bc ca q abc r + = + = + = + + = + + = =
Do 1 nn ( 1)( 1)( 1) 1 2 a b c p q r = = +
Bt ng thc cho tng ng vi
2 2 2
4 4 4 16 47
16
a b c
abc a b c

+ + +
( )
2
2 2 2
47
4 8 ( ) ( ) 16
16
ab bc ca abc a b c abc ab bc ca abc a b c + + + + + + +
2
2 2 2 2
47 47 9
4 8 16 0 4 8 (2 ) 16 0 2 0
16 16 4
q pr qr r r q r q r qr r r q r
| |
+ + +
|
\

Bt ng thc trn lun ng. Vy ta c iu phi chng minh.

BI VIC O 10. Cho , , l 3 gc ca mt tam gic nhn. Tm gi tr nh nht ca
2 9 17
Nguyn nh Thi
LI GII 1 (VIC STAFF VIC STAFF VIC STAFF VIC STAFF). t 4,

,
Ch rng nn
4
5
3
4.
5
3
.
Hay 12 5 3 20, , , 0
V c biu theo , , thnh 8 15 17
Do , theo bt ng thc cho 20 s v 40 s ta c
20

12 5 38 15 17

20.

. 40.

32000
Suy ra 20

32000. Hay 40
Vy 40, t c khi . Ch rng 12 5 3 20 nn 20 20

, suy ra
1, hay 4,

, 1, tc l 4,

, 1.

LI GII 2 (VIC STAFF VIC STAFF VIC STAFF VIC STAFF). Ta s c gng a bt ng thc v mt bin bng k thut kho st hm s
Ch rng nn ta chuyn bt ng thc v dng i s bng cch t
, , th bi ton c vit li nh sau:
Cho cc s thc dng , , sao cho .
Tm gi tr nh nht ca 2 9 17
T iu kin , suy ra

, 1, thay vo ta c
2 9
17
1

Xt 2

vi l tham s dng
Ta c

trn


34

1
2

Cc Cuc Thi Bt ng Thc
This file was downloaded from VietNam Inequality Mathematic Olympiad Resource Page http://www.ddbdt.tk

Khi qua

th hm s i du t sang nn t cc tiu ti


T 9

9
Xt
9
19

2
34

9
19


68

34

1
,

0 9

1934

1 68


5
3

T lp bng bin thin ta thy hm s i du t sang khi i qua

nn t cc tiu ti

40.
Vy,

40. ng thc xy ra khi v ch khi 4,


, 1.

BI VIC O 11. Cho cc s thc khng m , , sao cho 1. Tm gi tr ln nht ca
, ,


Nguyn nh Thi
LI GII 1 (VIC STAFF VIC STAFF VIC STAFF VIC STAFF).
Khng mt tnh tng qut, gi s , , th

0,

0
Do
, ,


Mt khc, theo bt ng thc th

1
256

Vy, gi tr nh nht ca , , l

, t c ti , ,

, 0 v cc hon v tng ng.



LI GII 2 (PHAN PHNG C, BI C ANH, NGUYN VIT HNG).
Khng mt tnh tng qut, gi s , , . Khi d thy
3 3
3 3
2 2
c c
a b a b
| | | |
+ + + +
| |
\ \
,
3
3 3
2
c
a a c
| |
+ +
|
\
,
3
3 3
2
c
b b c
| |
+ +
|
\

Do
, ,



2
,

2
, 0
t

th 1 v ta cn tm gi tr nh nht ca


p dng bt ng thc cho 4 s ta c

1 3. .
1 3
4

1
256


LI GII 3 (NGUYN CH HIU, NGUYN QUANG HUY).
Khng mt tnh tng qut, gi s 0. Khi d thy
3 3 3 3 3
( ) a b c a b c + + + + ,
3 3 3 3 3 3 3 3
( ) a b b c a c b c a + + +
Do
, ,

, , 0
Do ta ch cn gii bi ton trong trng hp 0. Tc l tm gi tr ln nht ca

, vi
1
Cch gii hon ton tng t nh trn.

Cc Cuc Thi Bt ng Thc
This file was downloaded from VietNam Inequality Mathematic Olympiad Resource Page http://www.ddbdt.tk

BI VIC O 12. Cho , , l di 3 cnh ca mt tam gic c din tch , v cc s thc dng , , sao
cho 4

. Chng minh rng

43
Nguyn nh Thi
LI GII (VIC STAFF VIC STAFF VIC STAFF VIC STAFF). Ta s s dng cc b sau
B 1. Vi + + = , th
2 2 2
2 1
2 2 2 2 2 2
sin sin sin sin sin sin

+ + + =

Chng minh. Ta c
2 2 2 2 2
1 1
1
2 2 2 2 2 2 2 2 2
cos cos
sin sin sin sin cos cos sin
+ | | | |
+ + = + + = +
| |
\ \
1 1 2
2 2 2 2 2 2
sin cos cos sin sin sin
| | + | | | |
= =
| | |
\ \ \

Do b 1 c chng minh.
B 2. Vi + + = , v cc s thc bt k , , th
2 2 2
2 2 2 yzcos zxcos xycos x y z + + + +
B 3. Vi , , l di 3 cnh ca mt tam gic c din tch th
2

43
B 2 v 3 l 2 b kh quen thuc nn chng ti khng chng minh li y na
Tr li bi ton. T gi thit 4

, ta c

4
1
Nn theo b 1, tn ti cc gc , , sao cho + + = vi ,
2 2
a
sin
yz

= ,
2 2
b
sin
zx

=
2 2
c
sin
xy

=
p dng b 2, ta c
2 2 2
4 4 4 2 (1 ) 2 (1 ) 2 (1 )
2 2 2
yzsin zxsin xysin yz cos zx cos xy cos

+ + = + +
( )
2 2 2
2( 2 ) 2( ) xy yz zx yzcos zxcos xycos xy yz zx x y z = + + + + + +
T y kt hp vi b 3 vi ch rng
2 2 2 2 2 2
4 4 4
2 2 2
yzsin zxsin xysin a b c

+ + = + +
Ta thu c

43 . Php chng minh hon tt.



BI VIC O 13. Cho tam gic ABC ni tip trong ng trn (O, R) c

0
45 BAC = . Gi s AH l ng cao
ca tam gic. BO, CO ln lt ct cc cnh AC, AB ti D v E; HO ct DE ti K. Chng minh rng
( ) 1
HK
cos B C
R


Nguyn nh Thi













L
Q
P
I
S
O
M
N A'
F
K
D
E
H
C
B
A
Cc Cuc Thi Bt ng Thc
This file was downloaded from VietNam Inequality Mathematic Olympiad Resource Page http://www.ddbdt.tk



LI GII (VIC STAFF VIC STAFF VIC STAFF VIC STAFF).

Trc tin ta s chng minh HK DE .
Ta c ~ ( . . )
OD OC
COD AHB g g g
BH AH
= , ~ ( . . )
OE OB
BOE AHC g g g
CH AH
=
T
OD BH
OE CH
= .
V HM OB th
BH BM MH
CH OM OM
= = (tam gic OBC vung cn).
T

( . . )
OD MH
OMH EOD c g c OED HOM DOK
OE OM
= = =
M ta c

0
90 EOD= nn HK DE .
By gi ta chng minh ( )
HK
cos B C
R
:
Ta c

( ) ( )

0 0
( )= 90 90 '
AA' ' 2 '
AH OF R
cos B C cos BAH CAH cosHAA
OA OA
= = = = (vi F l chn ng cao
t O n BC)
Do ta ch cn chng minh
2
( ) ' (*)
2
R
OH OK OA +
Gi s OH ct (O) ti S v I (nh hnh v), v HQ vung gc vi NI, ON l phn gic ca gc

HOC, th
HN HO HO HO
HC HO OC HO R HS
= = =
+ +

Ta c

1
2
HON HOC IBC = =
Nn t gic BONI ni tip, suy ra

HNQ BOH DOK OEK = = =
Nn HQN OKE ,vi ch

0
45 HIQ OBH HIQ = = vung cn ti Q, nn
OE 2
HN HQ HI
OK OK
= =
OE.HI 1 . 1 1 . .
. . . . . . (**)
.
2 2 2. 2.
2.
OE HI HS OB HP R HL R
OK AH HP
HC OH
HC OH AH OH OH OH
HS
= = = = =
(vi L l trc tm tam gic, d dng ta chng minh c HL=HP).
Ta li c
OF '
;
AH ' 2.
R OA
OA R AH
= =
+

2.OF= .2 2
2
R
AL R = =
( ' ) ( ')
2.
2. ' 2. '
R OA R R R OA
HL AH AL R
OA OA
+
= = =
Do t (**) ta suy ra
2
( ')
2. '.
R R OA
OK
OA OH

=

Nn bt ng thc (*) tng ng vi
( )
2
2 2
2
2. '
( ') 1 '
' . '
2. 2. 2. '. 2 2
R OH R OA
R R OA R R OA
OH OA OH OA R
OH OH OA OH
+
| | | |
+ =
| |
\ \

Hay
( ) ( ) ( ) ( ) ( )
2 2 2 2
2. ' 2. 2. ' 2. 0 R OH R OA OH R OH R OA OH R R +
M
( ) ( )
2 2 2
2. ' 2. ( ) 2 1 0
2 2
R R
OH OH R OA OH R R R R R R + + = >
Vy ta c iu phi chng minh. ng thc xy ra khi v ch khi tam gic ABC cn ti A.
By gi ta tip tc chng minh 1
HK
R
:

Cc Cuc Thi Bt ng Thc
This file was downloaded from VietNam Inequality Mathematic Olympiad Resource Page http://www.ddbdt.tk

Theo chng minh trn th ( . . )
OE
HQ HN
HQN OKE g g g
OK
= .

Do ta cn phi chng minh HO OK HO HI OK HI + +

Hay

1 1 . 1
1
OE ( ) 2 2 2
HI HQ HN HCOH
OK OK OE R OH

+
(do ON l phn gic

HOC)
. 1 ( ' ). 1
'
( ) 2 2. '. ( ) 2
AH OH R OA R OH
OH OA
R R OH OA R R OH
+

+ +

iu ny hin nhin ng do

0 0
90 HA'O 90 ' ' HA'O ' HAO AHO OHA OHA OA OH
Vy ta c iu phi chng minh. ng thc xy ra khi v ch khi tam gic ABC cn ti A.


















Cc Cuc Thi Bt ng Thc
This file was downloaded from the VietNam Inequality Mathematic Olympiad Resource Page http://ddbdt.tk
Cuc thi bt ng thc MIC 2009

Cuc thi bt ng thc MIC 2009 c t chc cng phu bi din n Ton hc Vit Nam
http://www.maths.vn/. Trong tp san ny, chng ti s tuyn tp mt s li gii hay cho mt s bi ton ca
cuc thi ny. Cc bn c th tip tc ng gp li gii mi trc tip ln forum http://www.ddbdt.tk/ hoc qua
hm mail vif.vimf@gmail.com

BI MIC O 1 . Cho , , l cc s dng v , , l di 3 cnh ca mt tam gic. Chng minh
z x y y z x x y z
ab ca bc a b c
z y x
+ + +
+ + + +

TRN QUC LUT
LI GII 1 (MIC Staff). S dng bt ng thc ta c



1
4


Nn ta ch cn chng minh c


1
4

,
4

22

.
Ch rng 4

0 v 4

0 nn sau khi p dng bt ng thc , ta ch cn chng


minh bt ng thc sau
4

.
Bt ng thc ny tng ng vi bt ng thc sau

4
t , , th , , 0 v bt ng thc trn tr thnh









2

2


2
Bnh phng 2 v ca bt ng thc ny ta c bt ng thc tng ng l


Bt ng thc ny hin nhin ng theo bt ng thc 3 s.
Php chng minh hon tt.

LI GII 2 (MIC Staff). Gi s rng , , l di 3 cnh ca tam gic . Khi

2

.
iu ny dn n
Cc Cuc Thi Bt ng Thc
This file was downloaded from the VietNam Inequality Mathematic Olympiad Resource Page http://ddbdt.tk


cos

2
cos

2
cos

cos

2
cos

2
cos

2

cos

2
cos

2
cos

2

2 2 2 2 2 2
Mt khc, vi ch bt ng thc quen thuc
2 2 2


Ta c th d dng suy ra kt qu ca bi ton.

LI GII 3 (VIMF).
Do , , l di 3 cnh ca mt tam gic nn t , , , , 0
Bt ng thc cn chng minh tng ng vi
2 2 2 k n m
ab ca bc a b c
k n m n m k
+ + + +
+ + +

Khng mt tnh tng qut, gi s l s gia v . Theo bt ng thc , ta c
2 2 2 2 2 2
. .
k n m ka na k n mc
ab ca bc b c b
k n m n m k k n k n m n m k
+
+ + = + +
+ + + + + + +

1 2 2 2 1 2
2 2
2 2
ka na k n mc k n m
b c b a b c
k n k n m n m k m n m k
( + + | | | |
+ + + + + = + + +
| | (
+ + + + + +
\ \

Do ta ch cn chng minh
2
. ( )( ) 2 ( ) 2( )( )
2
c k n m
c k m k n m m n m n m k
m n m k
+ | |
+ + + + + + +
|
+ +
\

2
( )( ) 0 k mn km kn k m k n + +
Bt ng thc trn ng do l s gia v .
Vy ta c iu phi chng minh. ng thc xy ra khi v ch khi v .

BI MIC O 2. Cho cc s dng , , . Bit rng a b c v
1 1 1
a b c
a b c
+ + = + + . Chng minh rng
1
1
b
a c

+

Vasile Cirtoaje

LI GII 1 (MIC Staff).
T gi thit ta suy ra 0, hay l , t dn n

1

,
Suy ra

2 1

0,
V ta thu c

1

hoc
1

,
Mt khc, ta thy m

nn khng th xy ra trng hp

. Nh
vy, ta phi c

1

1
Hay l

1
1

Bt ng thc ca ta c chng minh xong. D thy rng t cc lp lun trn ta c ng thc xy ra khi v ch
khi 1.
LI GII 2 (MIC Staff).
Cc Cuc Thi Bt ng Thc
This file was downloaded from the VietNam Inequality Mathematic Olympiad Resource Page http://ddbdt.tk
t

th ta phi chng minh


1 vi v

. T
gi thit

, ta suy ra c
2
1

4
Do ta phi chng minh

1

4 2
1

1
Hay l

4
1

1 2
Do

1

1 2
1

0
Nn ta c th bnh phng 2 v bt ng thc trn v thu gn n li thnh
4 1 1

0
Bt ng thc ny ng nu 1 . Tuy nhin, p dng gi thit

mt ln na, ta thy
rng
2 2
1

6
T suy ra


3
1,
1

3
1 1
Php chng minh ca ta hon tt.

LI GII 3 (VIMF). t , , . Do a b c nn x y z
Ta c
2 2 2
1 1 1
a b c ab bc ca a bc ab c abc x y z xy yz zx
a b c
+ + = + + + + = + + + + = + +
V bt ng thc cn chng minh c vit li l 1 1
zx
ab bc b x z
y
+ + + +
Ch rng x y z nn
2
( )
3 3 1
3
x y z
x y z xy yz zx x y z x x
+ +
+ + = + + + +
By gi ta xt 2 trng hp
Trng hp 1. Nu 1 z th ta ch cn chng minh
zx
x
y

M do x y z v 1 x nn 1.
zx
x x
y

T ta c iu phi chng minh
Trng hp 2. Nu 1 z
T x y z xy yz zx + + = + + , suy ra
1
x z xz
y
x z
+
=
+
. Do
( 1)
1 1 1 ( 1)( )
zx xz x z xz
x z x z x z x z x z xz xz
y x z xz x z xz
+
+ + + + + +
+ +

2
( ) ( )( 1) 0 ( )( 1) 0 ( )( 1)( 1) 0 x z x z xz x z x z xz x z x z + + + + + +
Bt ng thc trn ng do 1 x z
Vy ta c iu phi chng minh. ng thc xy ra khi v ch khi 1 hay 1.

Cc Cuc Thi Bt ng Thc
This file was downloaded from the VietNam Inequality Mathematic Olympiad Resource Page http://ddbdt.tk
LI GII 4 (NGUYN C TON). T gi thit ta suy ra

1
3

3
kt hp vi gi thit th ta suy ra 1
Do
1

0
1

2
1

0
1

2
1

2
1

1
1


T y ta suy ra

. Php chng minh hon tt.



LI GII 4 (NGUYN VIT HNG). T gi thit ta d thy 1 0, by gi ta ch cn xt
Trng hp 1. Nu 1 th
1
1

0
1


V ch

2 nn cng 2 bt ng thc trn v theo v ta c



1

2
1


Trng hp 2. Nu 0 1 th
1
1

0
1


V ch

2 nn cng 2 bt ng thc trn v theo v ta c



1

2
1


Vy trong c 2 trng hp ta lun c

1

2
1

, 0
V ta s chng minh 1

theo gi thit ny l xong


Tht vy, gi s 1

1 th
1
1

1
1

1 2
Cng 1 v 2 li ta thu c

1

2
1


iu ny v l theo gi thit trn
Do 1

. Php chng minh hon tt.



BI MIC O 3. Cho cc s thc dng , , tha mn

3. Chng minh rng


82 2 2
TRN QUC LUT
LI GII 1 (MIC Staff).
Gi s , , , suy ra 0 1. Ta c cc nh gi sau
22 2 8 4 2 8 4

1
2

3

2
3

1,

2

3

2

3 2

2
.
V vy, chng minh bt ng thc cho ta ch cn chng minh bt ng thc sau l
8 2 3

3 2


L mt bt ng thc hin nhin ng v
82 3

3 2

0
Vy ta c iu phi chng minh. ng thc xy ra khi v ch khi 1.

Cc Cuc Thi Bt ng Thc
This file was downloaded from the VietNam Inequality Mathematic Olympiad Resource Page http://ddbdt.tk
LI GII 2 (MIC Staff). Tng t nh li gii 1, ta cng c nh gi 22 2 1


Mt khc theo bt ng thc th 22 4 2 4


Do vy, theo bt ng thc th
42 2 2

1.

1

Ngoi ra, p dng bt ng thc ta cng c
2

1
2

Kt hp vi kt qu trn ta thu c iu phi chng minh.

LI GII 3 (VIMF). Khng mt tnh tng qut, gi s , , , khi
3

1, 3

2 1 1,
Ta c 4 2

2 1


nh gi tng t i vi v . Nhn cc bt ng thc li vi nhau ta c
82 2 2


V ta ch cn chng minh c


T iu kin

3 ta thu gn bt ng thc ny thnh


2

1
Hay ta ch cn chng minh
2

1 2 1 1 1 2 1
iu ny hin nhin ng theo cc nh gi 1 , 2 1. Php chng minh hon tt.

BI MIC O 4. Chng minh rng nu , , l cc s dng th



2
3 3 2


TRN QUC ANH
LI GII 1 (MIC Staff). Theo bt ng thc th

1
4


4


2


Do ta c


Mt khc ta thy
3 3 2


1
8
. 8. 3 3 2. 3 3 2
1
8

8 3 3 2 3 3 2
3


Suy ra


2
3 3 2


Nn t y ta d dng suy ra c iu phi chng minh. Php chng minh hon tt.

LI GII 2 (MIC Staff). Ta vit bt ng thc cn chng minh li thnh
2


3 3 2



Sau khi phn tch v thu gn ta thy bt ng thc c dng tng ng l
2
1


3 3 2



n y p dng bt ng thc , ta thy rng
2
1

3 3 2



3 3 2



Cc Cuc Thi Bt ng Thc
This file was downloaded from the VietNam Inequality Mathematic Olympiad Resource Page http://ddbdt.tk
Php chng minh ca ta hon tt.

LI GII 3 (VIMF). Chun ha 1. Khi bt ng thc cn chng minh c vit li



1 1
3


1
1

1

1


Theo bt ng thc th
1
1

1

1
4


1

1

1
4
1

1

1

1
1


Do ta ch cn chng minh
1
1


Hay 1

3 1

0
Bt ng thc trn hin nhin ng. Php chng minh hon tt.

LI GII 4 (NGUYN NH TON). Theo bt ng thc ta c

3 3 2
6

3 3 2
6

4
3

2
3 3 2


Php chng minh hon tt.

LI GII 5 (BCH NGC THNH CNG). Bt ng thc tng ng vi


3 3 2

8
3 3 2

2
8

2

Theo bt ng thc th

2

Php chng minh hon tt.
LI GII 6 (NGUYN HUNH KHI NGUYN). t 0 1
c a b c
x x
a b a b
+ +
= > + =
+ +
. Khi
2
3 3 2


1 2
3 2

,
2
1 1 1 1
. . . .
( )( ) ( ) 4 ( )
( )( )( ) 4 ( 1) 1
1 1
( )
abc c c
x x
b c c a c a b c c a b c
a b b c c a a b a b x x
ab ab a b
= = =
+ + + + + +
+ + + + + + +
+ +
+

Vy ta cn chng minh
( )
3 2 2 3
2
1 2
8 12 6 1 (9 4 12 ) 4 1 3
4 ( 1) 1
3 2
x x
x x x x x x x x
x x
x
+
+ + + + + +
+ +
+

Theo bt ng thc 12 s th
3 12
12
8
1 1
4 8. 12 3
8 8
x x x + = . Php chng minh hon tt.

LI GII 7 (NG VIT TRUNG). t x a b = + , y b c = + , z c a = + . Vit li bt ng thc di dng
( )( )( ) ( )
( )
2
8
2
x y z x z y y z x x z y
xyz
x y z
+ + + +

+ +

V bt ng thc l ng bc nn c th gi s 1. Khi , ta cn chng minh
( )( )( ) ( )
( )
( )
( )
2 2
1 2 1 2 1 2 1 1 4 3 1
8 8
1 1
x y z x x xy yz xz x
xyz xyz
x x
+ + + +

+ +

Cc Cuc Thi Bt ng Thc
This file was downloaded from the VietNam Inequality Mathematic Olympiad Resource Page http://ddbdt.tk
( )
( )
( )
( )
2
2
2 2
1 4 2 1 1 3 1 1 5
8 2 8
1 2 1
x y z x x x
xyz x xyz
x x x
+ + +
+
+ +

Nu
2
1 5 0 x , bt ng thc hin nhin ng. Nu
2
1 5 0 x th bt ng thc tng ng vi
( ) ( )
2 2
2
2 1 1
4
1 5
x x
yz
x
+


Ta c ( ) ( )
2 2
4 1 yz y z x + = nn cn chng minh
( )
( ) ( )
( )
2 2
2 2
4 3 2 2
2
2 1 1
1 9 6 0 3 1 0
1 5
x x
x x x x x x
x
+
+


Php chng minh hon tt.




































Cc Cuc Thi Bt ng Thc
This file was downloaded from the VietNam Inequality Mathematic Olympiad Resource Page http://ddbdt.tk

thi hc sinh gii cc tnh thnh Vit Nam nm 2008

BI TON 1. Cho cc s khng m phn bit , , . chng minh
( )
( ) ( ) ( )
2 2 2
2 2 2
1 1 1 11 5 5
2
a b c
a b b c c a
| |
+
| + + + +
|

\

B RA VNG TU
LI GII (VIMF).
Khng mt tnh tng qut, gi s , , . Khi 0, 0. Do


By gi ta c

0. Xt
5

4
2

2 4
2


2 1

0,

0, 0
5 1
2

Lp bng bin thin ta thy hm s i du t sang khi qua

, nn t gi tr nh
nht ti

. Vy

, t c ti

.

BI TON 2. Cho hnh hp ch nht vi kch thc 3 cnh l , , v di ng cho l 3. Chng minh


3
2

H NI
LI GII (T Minh Hong). Do di ng cho l 3 nn
2 2 2
3 a b c + + =
a bt ng thc v dng
2
3
2 3
a
a

. Ta chng minh
2
2 4 4 2
2
2 3 3
2 3
a a
a a a a a a a
a
+ +


Bt ng thc trn ng theo bt ng thc , tng t cho 2 bt ng thc cn li ri cng v theo v
ta c
2 2 2
2 2
( ) 3
2 2
a a b c
b c
+ +
=
+


Php chng minh hon tt.

BI TON 3. Cho cc s khng m , , sao cho

1. Tm min v max ca


1


1


1

H NI
LI GII 1.
Gi tr nh nht. Ta c
Cc Cuc Thi Bt ng Thc
This file was downloaded from the VietNam Inequality Mathematic Olympiad Resource Page http://ddbdt.tk

2

1

2
2 1

1 1
Tng t ta cng c 1, 1. Do

1
Vy, gi tr nh nht ca l 1, t c khi v ch khi , , 0; 0; 1 v cc hon v
Gi tr ln nht. Trc tin ta chng minh
1

2
21

1 4 2

0
Bt ng thc trn l ng, v tng t ta cng c
1

2
, 1

2

Cng cc bt ng thc trn v theo v ta c 2
Vy, gi tr ln nht ca l 2, t c khi v ch khi , , 1; 1; 0 v cc hon v.

NHN XT. Bn V Quc B Cn a ra thm li gii phn max v bn T Minh Hong a ra thm li gii
phn min nh sau:
LI GII (V Quc B Cn - T Minh Hong).
Gi tr nh nht (T Minh Hong). bt ng thc tng ng vi
( 1)( 1) ( 1)( 1)( 1) x xy xz xy yz zx + + + + +

2 2 2 2 2 2 2
( ) 1 ( )
sym
xyz x y z x y x y z x y z xy yz zx xyz x y z + + + + + + + + + + + + +

2 2 2
( )( ) 2 1 x y z xy yz zx x y z x y z xyz xy yz zx + + + + + + + + + + + +
2 2 2
( 1)( 1) 2 x y z xy yz zx x y z xyz + + + + + +
2 2 2
2( )( 1)
2
1
xy yz zx xy yz zx
x y z xyz
x y z
+ + + + +
+
+ + +

Theo bt ng thc ta c
2 2 2
3
6 2( )( 1)
1 1
x y z xy yz zx xy yz zx
x y z x y z
+ + + + +

+ + + + + +

Ch
1
3 3
abc , nn bt ng thc s c chng minh nu 6 ( 1)( 2) x y z xyz + + + +
Bt ng thc ny ng do
1
3 3
abc v 3 a b c + +

Gi tr nh nht (V Quc B Cn).
Ch rng t gi thit, ta suy ra
1
, , .
2
xy yz zx Dn n
2
.
1 1 3
x xyz
x x xyz
yz yz
=
+ +

Tng t i vi hai bt ng thc cn li ri cng v theo v ta c 2 .
1
x
x y z xyz
yz
+ +
+


Mt khc, theo bt ng thc th
2 2 2 2 2 2 2
[ (1 2 ) ( )] [ ( ) ][(1 2 ) 1] 2(1 2 )(1 2 2 ) 2[1 2 (1 2 )] 2. x yz y z x y z yz yz yz y z y z yz + + + + + = + + =
Dn n 2 2. x y z xyz + +
Kt hp bt ng thc ny v bt ng thc trn, ta thu c 2.
1
x
yz

+


ng thc xy ra khi
1
, 0
2
x y z = = = v cc hon v tng ng.

BI TON 4. Cho cc s thc dng , , . Tm gi tr nh nht ca

2

1


Cc Cuc Thi Bt ng Thc
This file was downloaded from the VietNam Inequality Mathematic Olympiad Resource Page http://ddbdt.tk
H NI
LI GII (V Thanh Tng). Ta t
1
, , a x b y c
z
= = = ta c
7 14 7 7 7 2
5 2 6 12 2 7 6 12 2 12 2 12 2 7 6 7 6 7 6
( )
2 2 2 2 2
a a a b c
P
a b b c a b a b c a b b c c a a b c b c a c a b
+ +
= =
+ + + + + + +


p dng bt ng thc ta c
14 7 7 12 2
5 2 7 a a b a b + ,
7 7 7 7 7 6
2( 6 ) 2.7 a c a b a b c +
Tng t ta c
7 7 7 2 14 14 14 7 7 7 7 7 7 12 2 12 2 12 2 7 6 7 6 7 6
7( ) 5( ) 16( ) 7( 2 2 2 ) a b c a b c a b b c c a a b b c c a a b c b c a c a b + + + + + + + + + + + + 1 P
Vy 1 Pmin = khi
1
a b c x y
z
= = = = . Php chng minh hon tt.

BI TON 5. Cho cc s thc a,b,c. chng minh
( )( )( ) ( )
2
2 2 2
1 1 1 1 a b c ab bc ca + + + + +
TP. H CH MINH
LI GII. Khai trin bt ng thc ny ta c

2 2
Hay

0
Php chng minh hon tt.

BI TON 6. Cho cc s thc dng , , tha mn

1


Chng minh

3


2


TP. H CH MINH
LI GII (T Minh Hong). T gi thit, ta c
9
a b c
a b c
+ +
+ +
, v
ab bc ca
a b c
abc
+ +
+ +
Nn
2 2
3
( ( )) ( ) 3 ( ) ( ) 3 abc a b c ab bc ca abc a b c abc a b c a b c
abc
+ + + + + + + + + +
Do
2( ) 3 2
3 3
a b c a b c
a b c
a b c abc
+ + + +
+ + = + +
+ +

Php chng minh hon tt. ng thc xy ra khi v ch khi 1.

BI TON 7. Cho cc s dng , , . Chng minh



8

2
TP. H CH MINH
LI GII 1 (T Minh Hong).
Bt ng thc tng ng vi
2 2 2
8
1 1 0
( )( )( )
a b c abc
ab bc ca a b b c c a
+ +
+
+ + + + +

2 2
( ) ( )
0
2( ) ( )( )( )
a b a b c
ab bc ca a b c b c a


+ + + + +

2
( ) (( )( )( ) 2 ( )) 0 a b a b c b c a c ab bc ca + + + + +
Bt ng thc c a v dng SOS, trong

2
Cc Cuc Thi Bt ng Thc
This file was downloaded from the VietNam Inequality Mathematic Olympiad Resource Page http://ddbdt.tk
Khng mt tnh tng qut, gi s a b c , khi , 0
b c
S S v
2
2 ( ) 0
a b
S S c a b + = +
Php chng minh hon tt. ng thc xy ra khi v ch khi .

LI GII 2 (T Minh Hong). Ta s dng ng thc sau
2 2 2
( )( ) ( )( ) ( )( )
1 ,
a b c a b a c b c b a c a c b
ab bc ca ab bc ca
+ + + +
=
+ + + +

8 ( )( )( ) ( )( )( ) ( )( )( )
1
( )( )( ) ( )( )( )
abc b c a b a c a c b c b a a b c a c b
a b c c c a a b c c c a
+ + + + +
=
+ + + + + +

Bt ng thc tng ng vi
1 1
( )( ) 0
) ( )( )
a b a c
ab bc ca a b a c
| |

|
+ + + +
\


Khng mt tnh tng qut, gi s a b c . Khi
1 1 1 1 1 1
0
( )( ) ( )( ) ( )( ) ab bc ca a b a c ab bc ca b c b a ab bc ca c a c b

+ + + + + + + + + + + +

Theo bt ng thc , ta c iu phi chng minh.

BI TON 8. Cho cc s thc dng , , tha 1 4. Chng minh
xy yz zx x y z + + + +
NGH AN
LI GII 1 (VIMF).
Theo nguyn l th 2 trong 3 s 1, 1, 1 cng du. Khng mt tnh tng qut, gi s
1 1 0. Khi 1 ,v ta ch cn chng minh 1
T iu kin bi ta c

, nn thay vo bt ng thc trn ta c bt ng thc tng ng l


1 1 1
1
4 1
1

1 4 1

0
Bt ng thc trn hin nhin ng. Php chng minh hon tt. ng thc xy ra khi v ch khi 1.
LI GII 2 (T Minh Hong).
Gi thit bi ton tng ng vi
1 1 1
1
2 1 2 1 2 1 a b c
+ + =
+ + +

t
1 1 1
, ,
2 1 2 1 2 1
x y z
a b c
= = =
+ + +
, suy ra x y z 1 + + = v
1 1 1 1
1, 1,
2 2 2 2
a b c
x y z
= = =

Bt ng thc cn chng minh tng ng vi
1 1 1 1 1 1 1 1 1 1 1 1 1 1 1 1 1 1
2 2 2 2 2 2 2 2 2 2 2 2 2 2 2 2 2 2 x y y z z x x y z
| | | | | | | | | || |
+ + + +
| | | | | |
\ \ \ \
\ \

3
1 1 1 9 1 1 1
( ) 9 4( )( )
4 4 4 4
x y z xyz x y z x y z
xy yz zx x y z
+ + + + + + + + + + + +
y l bt ng thc quen thuc. Php chng minh hon tt.

BI TON 9. Cho cc s dng , , . chng minh bt ng thc
( )
( )
( )
( )
( )
( )
2 2 2
2 2 2
2 2 2
1
2
2 2 2
a b c b c a c a b
a b c b c a c a b

+ +
+ + + + + +

THANH HA
LI GII (Lng Quang Trung). Chun ha 1. Khi bt ng thc tr thnh
( )
( )
2
2
2 2 2
2
2 1 4 4 1 4 4 1
3 3 2 1 9 6 3
2 1
a a a a
a a a a
a a
+ + (
= =
(
+ +
+


Cc Cuc Thi Bt ng Thc
This file was downloaded from the VietNam Inequality Mathematic Olympiad Resource Page http://ddbdt.tk
Ta ch cn chng minh
2
4 1 7
2 9 6 3
a
a a
+


M ( )
2
2
2
4 1 1
2 9 6 3 2 3 1 0
2 9 6 3
a
a a a a
a a
+
+ +
+

Tng t cho hai bt ng thc cn li v cng cc bt ng thc v theo v ta c
2
4 1 3 7
2( )
2 2 9 6 3
a
a b c
a a
+
+ + + =
+


Php chng minh hon tt. ng thc xy ra khi v ch khi .

NHN XT. Ta cng c mt cch chun ha khc nhng vi cch chun ha ny th li gii s n gin hn bi
khng cn n k thut cn bng h s: chun ha cho 3. Khi
( )
( )
2
2 2
2 2 2 2 2
2
(3 2 ) 9 12 4 4 4 3 4 4 3
3 3 6 2 (3 ) 3 6 9 3( 1) 6
2
a b c
a a a a a
a a a a a
a b c

+ + +
= = =
+ + +
+ +

Tng t ta cng c
( )
( )
( )
( )
2 2
2 2
2 2
4 4 3 4 4 3
,
3 6 3 6
2 2
b c a c a b
b c
b c a c a b

+ +

+ + + +

Cng cc bt ng thc v theo v ta c
( )
( )
( )
( )
( )
( )
2 2 2
2 2 2
2 2 2
4( ) 9 1
4
6 2
2 2 2
a b c b c a c a b
a b c
a b c b c a c a b

+ + +
+ + =
+ + + + + +

Php chng minh hon tt.

BI TON 10. Cho cc s dng

, ,

tha mn

1. Chng minh rng


2 2 2
1 2 1 2
1 1 1 2( )
n n
a a a a a a + + + ++ + + ++
BNH PHC
LI GII.
Trc tin ta chng minh

1 2 1. Bnh phng 2 v bt ng thc ny v sau khi rt gn


ta c bt ng thc tng ng l 1

0 (lun ng). Ln lt thay bi

v cng li
v theo v ta c

1 2


M theo bt ng thc th


Do

1 2



BI TON 11. Cho cc s thc dng , , . Chng minh rng



2


BNH PHC
LI GII (T Minh Hong). Ta c
3 2 2
2 2 2 2
2 2
a ab ab b
a a a
ab a b a b
= =
+ +

Tng t i vi hai bt ng thc cn li v cng cc bt ng thc v theo v ta suy ra iu phi chng minh.
Php chng minh hon tt. ng thc xy ra khi v ch khi .

Cc Cuc Thi Bt ng Thc
This file was downloaded from the VietNam Inequality Mathematic Olympiad Resource Page http://ddbdt.tk
BI TON 12. Cho , , 0. Chng minh
1

21


BNH PHC
LI GII. Bt ng thc tng ng vi


Theo bt ng thc , ta c

,
Tng t ta cng c


Cng cc bt ng thc trn v theo v ta suy ra iu phi chng minh. ng thc xy ra khi v ch khi
.

BI TON 13.Chng minh x th
2 3
1
2! 3!
x
x x
e x + + +
HI DNG
LI GII (Nguyn c Ton). t
3 2
( ) 1
3! 2!
x
x x
f x e x =
Xt hai trng hp
Trng hp 1. Nu 0 x
2
( ) 1
2!
x
x
f x e x = , ( ) 1
x
f x e x = , ( ) 1 0
x
f x e = >
Suy ra

ng bin, ( ) (0) 0 f x f =
Suy ra

ng bin, ( ) (0) 0 f x f =
Suy ra ng bin, ( ) (0) 0 f x f =
Trng hp 2. Nu 0 x
t ( 0) x t t =
3 2
1
( ) 1
3! 2!
t
t t
f t t
e
= + + ,
2
1
( ) 1
2
t
t
f t t
e
= + + ,
1
( ) 1
t
f t t
e
= +
1
( ) 1 0
t
f t
e
= + >
Suy ra ng bin, ( ) (0) 0 f t f =
Suy ra ng bin, ( ) (0) 0 f t f =
Suy ra ng bin, ( ) (0) 0 f t f =
Vy ta c iu phi chng minh. ng thc xy ra khi v ch khi 0
NHN XT. Ta cng c th tng qut cho bi ton thnh :
0
!
i n
x
i
x
e
i
=


vi mi * n N v n l

BI TON 14. Cho , , l cc s dng sao cho
9
5, 8
x y z
x x y
+ + =

. Chng minh rng


15 xyz
HI DNG
LI GII (T Minh Hong). T gi thit suy ra 4 y z + v 1 z . Nn
Cc Cuc Thi Bt ng Thc
This file was downloaded from the VietNam Inequality Mathematic Olympiad Resource Page http://ddbdt.tk
3 3 3
(3 5 15 ) (3 3 3 2 2 10 ) 45
3 5 15 15
27 27 27
x y z x y z y z z
x y z xyz
+ + + + + + +
=
Vy GTLN ca l 15 khi , , 5,3,1

BI TON 15. Cho cc s dng , , . tm gi tr ln nht ca
3 3 3
bc ca ab
P
a bc b ca c ca
= + +
+ + +

HU
LI GII (Nguyn c Ton). Ta c
2
1 1 1 ( )
1
3 3 3 3 3 3( )
bc a a b c
P
a bc a bc a b c ab bc ca
| |
+ +
= =
|
|
+ + + + + + +
\


2 2
2 2
2
1 ( ) ( ) 1 3
3 4 4 ( ) ( )
( )
3
a b c a b c
P
a b c ab bc ca a b c
a b c
+ + + +
= =
+ + + + + + +
+ + +

Vy, gi tr ln nht ca l

, t c khi v ch khi .

BI TON 16. Cho cc s khng m , , sao cho 1. Tm gi tr nh nht ca
1 1 1
1 1 1
x y z
P
x y z

= + +
+ + +

H TNH
LI GII. ng bc ha bt ng thc ny ta c


2


2


2

t , , . Khi



2


2


2

2
Vy, gi tr nh nht ca l 2, t c khi v ch khi , , 1; 0; 0 v c hon v tng ng.

BI TON 17. Cho cc s thc dng , , tha mn 1. Chng minh rng







3
2

NAM NH
LI GII. Bt ng thc cn chng minh tng ng vi










3
2








3
2

Theo bt ng thc ta c
Cc Cuc Thi Bt ng Thc
This file was downloaded from the VietNam Inequality Mathematic Olympiad Resource Page http://ddbdt.tk

1
2



















3
2

Php chng minh hon tt.

BI TON 18. Cho t gic li ABCD c ; ; ; . Chng minh rng
2 2 2 2
13 6 2 4 2 a b c d S + +
NAM NH
LI GII (V Thanh Tng). Trc ht chng minh b :
Cho tam gic ABC c 3 cnh ln lt l , , din tch vi mi b s , , tha mn
0
0
0
0
m n
n p
m n
mn np pm
+

+ +


Ta lun c
2 2 2
4 ma nb pc mn np pmS + + + +

Tr li bi ton
t . p dng b ta c
2 2 2
13 6 4 4 2
ABC
a b x S + ,
2 2 2
2 4 4 2
ADC
c d x S + +
Cng 2 bt ng thc trn v theo v ta c iu phi chng minh.

BI TON 19. Cho cc s dng , , sao cho 3. Tm gi tr nh nht ca
2 2 2
3 3 3
2 2 2
a b c
P
a b b c c a
= + +
+ + +

NG THP
LI GII (Lng Quang Trung).
Theo bt ng thc ta c
2 3 3
3 2
3 3
3 6
2 2 2
3 2 2
3
a ab ab
a a a a b
a b a b
ab
( (
(
= =
( (
(
+ +




p dng bt ng thc AM-GM,ta c
( )
2 2
3 3
2 1 2 1
3 3
a b a
a a b
+ +

Tng t cho cc bt ng thc cn li ri cng v theo v ta c
2
3 2 2
3
2 2 7 4
( 2( )) ( ) 1
3 3 27 2 9
a
a a b a b c a b c ab bc ca a b c
a b
(
+ + + + + + + + + =
(
+



Vy GTNN ca P = 1 khi 1.

BI TON 20. Cho cc s thc , , tha
1
2 3 6
11 27 54
x y z
y z
x z

. Tm gi tr ln nht ca
2 2 2
1 2008 2009
( , , ) P x y z
x y z
= + +
NG THP
LI GII 1 (Lng Quang Trung). Tac
2 2 2 2 2 2 2
1 2008 2009 1 1 1 1
2008
x y z x z y z
| |
| |
+ + = + + +
| |
\
\

p dng bt ng thc ta c
( ) ( )
2 2
2
2 2 2 2 2
2 2 2 2
729 1458 121 2
2916 54 11 27 2 121 729 121 729 1458 1
121 729 121
z z
x z x z x z
x x x x
= + + + + +
Cc Cuc Thi Bt ng Thc
This file was downloaded from the VietNam Inequality Mathematic Olympiad Resource Page http://ddbdt.tk
Suy ra
2 2 2 2
2 2 2 2 2 2 2 2 2 2
1 1 2 1 1 121 1 121 850
1 1
729 729 729
z z z z
x z x z x x z x x x
| |
+ = + + + + + =
|
\
(v 1 z )
p dng bt ng thc ta c
( ) ( )
2 2
2
2 2 2 2
2 2 2 2
9 18 4 2
36 2 3 2 4 9 4 9 18 1
9 4 4
z z
y z y z y z
y y y y
+ + + + +
Suy ra
2 2 2 2
2 2 2 2 2 2 2 2 2 2
1 1 2 1 1 4 1 4 13
1 1
9 9 9
z z z z
y z y z y y z y y y
| |
+ = + + + + + =
|
\
(v 1 z )

Vy GTLN ca l
2115274
729
. ng thc xy ra khi v ch khi ( )
27 3
, , , ,1
11 2
x y z
| |
=
|
\

LI GII 2 (V Thanh Tng).
Ta c
2 2 2 2 2 2 2
1 2008 2009 1 1 1 1
2008
x y z x z y z
| |
| |
+ + = + + +
| |
\
\

2 2 2
2 2 2 2
2 2 2 2 2
11 1 1 11 1 1 1 11
1
27 27 27
x x
z z z
z x z x x
| |
| | | | | | | |
+ = + = + + |
| | | |
|
\ \ \ \
\
2
2 2 2 2 2 2 2 2
11
1 1 1 1 1 2 1 1 27
2
x
z
z x x z x x x z
| |
+
|
\
+ + = +
Tng t
2 2 2
2 2 2
2 2
2 1 1 11 2
1 1 2008 1
3 27 3
P
y z
| |
| | | | | |
+ + + + + |
| | |
|
\ \ \
\

Php chng minh hon tt. ng thc xy ra khi v ch khi ( )
27 3
, , , ,1
11 2
x y z
| |
=
|
\


BI TON 21. Cho , , l 3 gc ca mt tam gic. Chng minh rng
1 cos

2
1 cos

2
1 cos

2
1
3
4


BNH NH
LI GII (Lng Quang Trung).
Bt ng thc tng ng vi
1
3 3
3
2
3
1 cos 1
2 4
A
| |
| |
+ < +
|
|
|
\
\

.
p dng bt ng thc v , ta c
3
2
1
3
2 2
1 1
3 3 1 2 sin 3 cos
1 2 2 3 3
1 cos 1 cos
2 2 3 3 3
A A
A A
| | | | | |
+ + +
| | |
| | | | \
| | + < +
| |
| | \ \
|

=
|
\ \


(v
2
sin 1 2 sin
2 2
A A
=

)
3
3
1 1
3 2
3 4
3
1
3 4
| | | |
+ +
| |
| |
\
|
= +
|
|
|
\
|
\
(v
1
sin
2 8
A

)
Vy, ta c iu phi chng minh.

BI TON 22. Cho
0 a b c d
bc ad
<

. Chng minh rng

1
THI BNH
Cc Cuc Thi Bt ng Thc
This file was downloaded from the VietNam Inequality Mathematic Olympiad Resource Page http://ddbdt.tk
LI GII (Lng Quang Trung).
t , , th 1 z y x . Bt ng thc cn chng minh tng ng vi
( ) ( ) ( ) ( ) ( ) ( ) (1) (2)
ay az a ay ax a
ax az y z x y
a ax ay az a az ay ax x y z xy z
t tip , th 1 v u
( ) ( ) 1 1
(2) . (3)
y y vy
y
y v u y u u
y vy y yv
y y y v
u u u u
| |
|

\
| | | |

| |
\ \

D thy khi 1 hay 1 th (3) ng
Xt , 1 y u > . Ta c
1
1
(3) (*)
y
u
y
v
y v



Nh vy ta ch cn chng minh bt ng thc ng bng cng c o hm.C hai trng hp cn xem xt:
Trng hp 1. v y >
Khi ta xt hm s
1
1
( ) , 1
t
f t t t

= >
1
ln ( ) ln
1
f t t
t
=


( )
( )
2
( ) 1 1
ln
( ) 1
1
f x
t
f x t t
t

= +

(ly o hm hai v)
( )
( )
2
( )
( ) 1 ln
1
f t
f t t t t
t t
=


M ta c bt ng thc quen thuc
1
ln 1 , 1 t t
t
> > . Suy ra ( ) 0 f t < vi mi 1 t >
( ) f t nghch bin trn khong ang xt. Suy ra
1
1
1 1
1 1 1
( ) ( )
y
y u
y y
v v v
f y f v y v y v v


> > > (v y u )
Tng t ta xt trng hp 2. v y
Khi ta xt hm s
1
( ) , 1
t
t
f t t t

= > ln ( ) ln
1
t
f t t
t
=

( )
2
( ) 1 1
ln
( ) 1
1
f t
t
f t t
t

= +

(ly o hm hai v)
( )
( )
2
( )
( ) 1 ln
1
f t
f t t t
t
=


M ta c bt ng thc quen thuc ln 1, 1 t t t < > . Suy ra ( ) 0 f t > vi mi 1 t >
( ) ( ) f y f v >
1
1 1
y
t u
y
v v
y v v


> (v v u )
Php chng minh hon tt.

BI TON 23. Cho 3 s thay i , , tha mn
2
3
3
x y z
x y
x y z

+ +

. Tm gi tr ln nht ca tng
2


BC NINH
LI GII. T gi thit suy ra
2

4, 2

8, 2

8
4
2

1,
4
2


2
2

2,
4
2


2
2


1
2

3
Ta c
4 2 1
4
2


4
2


2
2


4
2


2
2


1
2

12

22

3. 2


Suy ra 7. Vy GTLN ca l 7. ng thc xy ra khi v ch khi 2, 1, 0 .

BI TON 24. Cho , , l cc s thc dng sao cho 1. Chng minh rng


1
PH YN
LI GII.
Cc Cuc Thi Bt ng Thc
This file was downloaded from the VietNam Inequality Mathematic Olympiad Resource Page http://ddbdt.tk
Ta c


Suy ra



1





Tng t cho hai bt ng thc cn li v cng cc bt ng thc v theo v ta suy ra











1
Php chng minh hon tt. ng thc xy ra khi v ch khi 1.

BI TON 25. Cho cc s thc thay i tha mn 6. Tm gi tr nh nht ca


NGH AN
LI GII (V Thanh Tng).
Ta c
2 2 2
2 2 2 2 2 2 2 2 2 2 2 2
( ) ( ) ( )
2 ( ) ( ) ( ) ( )
2 2 2
a b x y x y a b
F a b c x y z x a b y c z c z c z
+ + + + +
= + + + + + + + + + + + + + + + + +

t 2; 2 a b d x y t + = + = . Khi ( ) ( ) 3 3 x y c a b z tc dz + + = =
Lc ny
2 2 2 2 2 2
2 ( ) ( ) F d t c z d t c z + + + + + + +
2 2
2 2
( ) ( ) 3
2 2 4
d c c d
F T t z
+
= + + + +
Xt h ta im , ,
2 2
c d
M
| |
|
\
ng thng : 3 0 dz tc + =
Vi mi im ( ; ) A z t th
2 2
( / ) 2 2
3
2 2
M
c d
MA d z t
c d

| | | |
+ + +
| |
+
\ \

Suy ra
2 2
2 2
3 3( )
3 3
4
c d
F T F
c d
+
+
+
.
Vy minF 3. = ng thc xy ra chng hn nh
1 6
1; 0; ;
2 2
a b c x y z = = = = = =


















Bt dng thc t k thi tuyn sinh ai hc Cao dng v cc cuc thi khc
This file was downloaded from the VietNam Inequality Mathematic Olympiad Resource Page http://ddbdt.tk
Bt ng thc t k thi tuyn sinh i hc Cao ng v cc
cuc thi khc

BI D O1 . Cho cc s thc dng , , , , , . Chng minh rng

2
( )
( )( ) ( )( ) ( )( ) 4( )
bcx cay abz a b c
x y x z y z y x z x z y x y z
+ +
+ +
+ + + + + + + +

Thi Ti Nng Ton Hc Tr THPT http://www.truongtructuyen.vn
LI GII. t , , m x y n y z k z x = + = + = + th bt ng thc cho tng ng vi

2
( ) ( ) ( ) ( ) bc m k n ca m n k ab n k m a b c
mk mn nk m n k
+ + + + +
+ +
+ +


2 2 2 2 2 2 2 2 2
2
( 2 ) ( 2 ) ( 2 )
( )
bc m k n mk ca m n k mn ab n k m nk
a b c
mk mn nk
+ + + + + +
+ + + +
2 2 2 2 2 2 2 2 2
2 2 2
( ) ( ) ( ) bc m k n ca m n k ab n k m
a b c
mk mn nk
+ + +
+ + + +
Nhng t cch t trn th ta thy ngay , , l dai 3 cnh ca tam gic ABC . Nn bt ng thc trn
c th vit li thnh

2 2 2
2 cos 2 cos 2 cos bc A ca B ab C a b c + + + +
Bt ng thc ny l mt bt ng thc qu qun thuc. Php chng minh hon tt.

BI D O 2. Chng minh rng vi cc s thc dng , , tha mn 3, ta c

3 5


Tuyn Sinh i Hc Khi A 2009
LI GII. Trc tin l 2 b c dng xuyn sut trong cc li gii ca chng ta
B 1.
Chng minh 1. Theo bt ng thc ta c
3

2 3 0
Chng minh 2. T iu kin ta c
4 2. 2
B 2. 2
Chng minh 1. Theo bt ng thc ta c

3
3
4



2

3
2
0 2
Chng minh 2. T iu kin ta c
3

9
2
Chng minh 3. Theo b 1 th


2
2

LI GII 1 (P N B GD&T PHAN HUY KHI).
t , , th

v iu kin 3
tr thnh

.
Bt ng thc cn chng minh tng ng

3 5

3 5

3 5

3 5


Theo b 2 ta thu c 2
Suy ra 2

v 3


T y ta suy ra iu phi chng minh. ng thc xy ra khi v ch khi hay .

LI GII 2 (L THNG NHT v cc cng s). T gi thit ta c 4
t , . Ta c

v 4
Bt dng thc t k thi tuyn sinh ai hc Cao dng v cc cuc thi khc
This file was downloaded from the VietNam Inequality Mathematic Olympiad Resource Page http://ddbdt.tk
Mt khc

4 2


Ta li c 3 12 3


Cng 2 bt ng thc trn v theo v ta c iu phi chng minh.

LI GII 3 (NGUYN ANH DNG v cc cng s). t , t gi thit suy ra


Theo b 2 th 2
Bt ng thc phi chng minh tng ng vi
2

3 2 3 5

3 2 5


3
5

22

3 2

0
V 0

3 2

0
T y suy ra iu phi chng minh.

LI GII 4 (BO TUI TR Online & THANH NIN Online).
T iu kin 3 suy ra 1

3.


Theo b 2 th 2
Chia 2 v ca bt ng thc cn chng minh cho

th ta c bt ng thc tng ng l
1

31 1 5

31

1 31 1

31 1 5

61 1 5

61 5

61
1
3
5

4 0 2 1 2 0
Bt ng thc cui hin nhin ng vi 2.

LI GII 5 (canhang_2007). Theo b 1 th v ch 4
S dng hng ng thc

ta c

2
42


Ngoi ra ta cng c 3 12 3


Nn cng 2 nh gi ny ta c iu phi chng minh.

LI GII 6 (tanpham90). Bt ng thc cho tng ng vi

2 3 2


2
3
3 2


t 2. Ta c

3 4


2
3
2

16

4 4 0
Theo b 2 ta c iu phi chng minh.

LI GII 7 (VIMF). Theo b 2 th 2
p dng ng thc


Ln lt thay , , bi , , ta c

3 5

3 4

3 4


Do 2 nn ta ch cn chng minh
Bt dng thc t k thi tuyn sinh ai hc Cao dng v cc cuc thi khc
This file was downloaded from the VietNam Inequality Mathematic Olympiad Resource Page http://ddbdt.tk

3 4

3 3

2 0
Bt ng thc trn ng do 2.
Vy, ta c iu phi chng minh. ng thc xy ra khi v ch khi .

Li gii 8 (VIMF). Theo b 1 th . Do

1
4
2 4 4 4 3

1
4

2 4 4 4
3

3

2
2


Vy, ta c iu phi chng minh.

Da vo cc cch gii trn cc bn hy gii bi ton sau, ng trn tp ch Ton Hc Tui Tr s thng 7 nm
2009
Cho cc s thc dng , , sao cho

2 5. Chng minh rng


"













Bt ng Thc Su Tm V Sng To
This file was downloaded from VietNam Inequality Mathematic Olympiad Resource Page http://www.ddbdt.tk

BT NG THC SU TM V SNG TO

LTG. y l nhng bi ton m chng ti su tm c t nhiu ngun ti liu khc nhau, ch yu
l trn cc din n Ton, c bit l Din n Bt ng Thc Vit Nam http://www.ddbdt.tk/ v chng ti
ch tng thut li theo nguyn vn ca ngi gi bi ln.

BI ST1. Cho cc s thc dng , , sao cho 3. chng minh rng
1
1

1
1

1
1

9
2

VIMF
LI GII (NGUYN ANH TUN). Theo bt ng thc ta c
1
1
1

1
1

2

Tng t i vi 2 bt ng thc cn li, cng cc bt ng thc li vi nhau ta c
1
1

1
1

1
1
3

2
3

15
4

4

Do ta ch cn chng minh
15
4

4

9
2

t . Ch rng 3
Nn 3 3. V bt ng thc cn chng minh c vit li thnh
15
4

4

9
2

15 18 0 3
33 3
2

33 3
2
0
D thy 3

nn nh gi trn ng vi 3, 3
Php chng minh hon tt.

BI ST 2. Cho cc s thc dng , , . Chng minh rng

3


.

2
.

3


LI GII 1. p dng bt ng thc ta c
.

2
.

3


27


T y ta c ngay iu phi chng minh.

LI GII 2. Theo bt ng thc th ta s chng minh bt ng thc mnh hn l


.

2

.

2
.

3


Hay

3
Theo bt ng thc th


1.
2

.
3

1
3
1
2


3

,


1.1.
3

1
3
1 1
3


1
3
2
3

,
Bt ng Thc Su Tm V Sng To
This file was downloaded from VietNam Inequality Mathematic Olympiad Resource Page http://www.ddbdt.tk


1.
2

.
3

1
31
2


3

.
Cng cc bt ng thc ny v theo v ta thu c iu phi chng minh.

BI ST 3. Cho cc s thc dng , , . Chng minh rng

1

1
1

1
1

1
1

1
1

1
1

1 3
Vasile Cirtoaje
LI GII. t

th bt ng thc c vit li thnh


1 1 1 1 1 1 3
Theo nguyn l th 2 trong 3 s 2, 2, 2 cng du. Khng mt tnh tng qut, gi
s 2 2 0 4 2 2 2 2 4 1
Li c

1

2 2 2 1 2 1
1 2 4 2 2. 2
T 1 v 2 ta suy ra 2 2 4
Vy, ta c iu phi chng minh. ng thc xy ra khi v ch khi , hay 1

BI ST 4. Cho , , l di 3 cnh ca mt tam gic sao cho

3. Chng minh rng


2
NGUYN ANH TUN
LI GII (VIMF).
t , , th

2 3


Do , , l di 3 cnh ca mt tam gic nn ta c mt bt ng thc quen thuc l
2

9 3

18 3 3 18

3
2
6

3
12

6

Bt ng thc cn chng minh tng ng vi 2. Theo bt ng thc trn th ta cn chng minh

3
12

6
2

17 24 0 3
3 41
2

41 3
2
0
T gi thit

3 suy ra

3 3
Ch rng 3

nn bt ng thc trn l hin nhin ng


Vy, ta c iu phi chng minh. ng thc xy ra khi v ch khi .

BI ST 4. Cho , , l di ba cnh ca mt tam gic. Chng minh
0,8 0,8 0,8
0,2
2 2 2
3
8 8 8
a b c
a bc b ca c ab
| | | | | |
+ +
| | |
+ + +
\ \ \

VIMF
LI GII (VIMF). Theo bt ng thc 10 s ta c
8 8
2 2 2 2 2
10 10
8
8 8 8 8 8 6 8
... 1 1 10. 10.
3 3 3 3 3
numbers
a bc a bc a bc a bc a a bc
a a a a a
| | | |
+ + + + + +
+ + + + | |
| |
\ \


Hay
0,8
2 2 2
3 3 .10 15
8 8 8 6 4 8 3
a a a
a bc a bc a a bc a
| |
=
|
+ + + + +
\

V tng t ta c
Bt ng Thc Su Tm V Sng To
This file was downloaded from VietNam Inequality Mathematic Olympiad Resource Page http://www.ddbdt.tk

0,8
2 2
3 15
,
8 4 8 3
b b
b ca b ac b
| |

|
+ + +
\

0,8
2 2
15
8 4 8 3
c c
c ab c ab c
| |

|
+ + +
\
.
Do
0,8 0,8 0,8
2 2 2
3 3 3
8 8 8
a b c
a bc b ca c ab
| | | | | |
+ +
| | |
+ + +
\ \ \
2 2 2
15 15 15
4 8 3 4 8 3 4 8 3
a b c
a bc a b ac b c ab c
+ +
+ + + + + +

( ) ( ) ( )
2 2 2
2 2 2 2 2 2 2 2 2
15 15 15
4 8 3 4 8 3 4 8 3
a b c
a a bc a b b ac b c c ab c
= + +
+ + + + + +

( ) ( ) ( )
( )
( )
2
3 3 3 2 2 2
15( )
( )
4 8 8 8 3
a b c
Schwarz
a a abc b b abc c c abc a b c
+ +

+ + + + + + + +

( ) ( )
2
3 3 3 2 2 2
15( )
( )
4 ( ) 24 3
a b c
Bunhiacopxki
a b c a b c abc a b c
+ +

+ + + + + + + +

Do ta ch cn chng minh
( ) ( )
2
3 3 3 2 2 2
15( )
3
4 ( ) 24 3
a b c
a b c a b c abc a b c
+ +

+ + + + + + + +

Hay
( ) ( )
2 2 2 3 3 3
2 10( ) 4 ( ) 24 a b c ab bc ca a b c a b c abc + + + + + + + + + +
Do , , l ba cnh ca mt tam gic nn t ; ; . Ta c
* ( ) ( )
2 2
2
4 4 ( ) a b c x y z p p x y z + + = + + = = + + ;
* ( )
2
2
( ) ab bc ca x y z xy yz zx p q q xy yz zx + + = + + + + + = + = + + ;
* ( )( ) (r=xyz) abc x y z xy yz zx xyz pq r = + + + + =
* ( )
3
3 3 3 3
24 3( )( ) 27 2 21 27 a b c abc a b c a b c ab bc ca abc p pq r + + + = + + + + + + + = +
Do
( ) ( )
2 2 2 3 3 3
2 10( ) 4 ( ) 24 a b c ab bc ca a b c a b c abc + + + + + + + + + +
( ) ( ) ( ) ( ) ( )
2
2 2 3 2 3
4 3 2 2 2 21 27 7 3 4.2 2 21 27 p p q p p pq r p q p p pq r + + + + +


4 2 2
33 9 216 126 p q pr p q + +
M theo bt ng thc th
( )
3
3 4 2
9 4( )( ) 9 4 31, 5 283, 5 126 x y z xyz x y z xy yz xz p r pq p pr p q + + + + + + + + + .
Nn ta ch cn chng minh
4 2
1,5 9 67,5 p q pr +
D thy bt ng thc trn ng theo bt ng thc
Vy ta c iu phi chng minh. ng thc xy ra khi v ch khi .

BI ST 5. Cho cc s thc khng m , , . Chng minh bt ng thc
2 2 2
3 3 3
2 2 2
3 . ( ) ( ) ( )
a b b c c a
a b c abc ab a b bc b c ca c a
ab bc ca
+ +
+ + + + + + + +
+ +

LI GII 1 (VIMF).
Nu a b c th
2 2 2 2 2 2
a b b c c a ab bc ca + + + + , nn theo bt ng thc th
2 2 2
3 3 3 3 3 3
2 2 2
3 . 3 ( ) ( ) ( )
a b b c c a
a b c abc a b c abc ab a b bc b c ca c a
ab bc ca
+ +
+ + + + + + + + + + +
+ +

Nu c b a th bt ng thc c vit li nh sau
2 2 2
3 3 3
2 2 2
3 3 . 1 ( ) ( ) ( ) 6
a b b c c a
a b c abc abc ab a b bc b c ca c a abc
ab bc ca
| | + +
+ + + + + + + +
|
+ +
\

2 2 2 2 2 2
2 2 2
1 3 ( )( )( )
( ) ( ) ( ) ( ) ( ) ( ) ( )
2
abc a b b c c a
a b c a b b c c a a b c b c a c a b
ab bc ca

( + + + + + +

+ +

2 2 2
2 2 2
1 1 1 3 ( )( )( )
( )( ) ( )( ) ( )( )
2 2 2
abc a b b c c a
a b c a b b c a b c c a b c a
ab bc ca

+ + + + +
+ +

Theo tiu chun 1 K thut phn tch bnh phng cho bt ng thc hon v th ta ch cn chng minh
Bt ng Thc Su Tm V Sng To
This file was downloaded from VietNam Inequality Mathematic Olympiad Resource Page http://www.ddbdt.tk

2 2 2
3 ( )
2 0
abc c a
ac c a b
ab bc ca

+ +
+ +

Quy ng, rt gn v nhm cc s hng li vi nhau ta c bt ng thc tng ng l
( )
2 2 2 2 2 2 2 3 2 2 2
2 ( ) ( ) 2 2 2 0 bc ac a ab c b bc c b a c a b a c ab ac ca ac a bc + + + + + + + +
Bt ng thc trn ng do . c b a

Vy ta c ta c iu phi chng minh. ng thc xy ra khi v ch khi
ba bin bng nhua hoc mt trong 3 bin bng 0 v 2 bin cn li bng nhau.

LI GII 2 (dangtrung).
Nu a b c th theo bt ng thc ta c
2 2 2
3 3 3 3 3 3
2 2 2
3 . 3 ( ) ( ) ( )
a b b c c a
a b c abc a b c abc ab a b bc b c ca c a
ab bc ca
+ +
+ + + + + + + + + + +
+ +

Nu a b c th bt ng thc a v dng
2
2 2 2
6 ( )( )( )
( , , ) ( ) ( ) 0
abc a b b c c a
f a b c a b a b c
ab bc ca

= +
+ +

( )
2 2 2 2
( ) 6 ( )( )( ) ab bc ca a b abc a b b c c a + +


M
2
( ) 4( )( ) a b b c c a v
( ) ( )
2 2 2 2 2 2 2
4 6 ( ) 2 3 3 ab bc ca bc a b ab bc ca b c abc + + + + + , hin
nhin ng nn ta ch cn chng minh bt ng thc trong trng hp 0
Hay cn chng minh
2 2 2 2
( ) ( ) ( ) ( ) 0 2( )( ) 0 b b c b c b c c c b b c b c + + + +
Bt ng thc trn hin nhin ng. Vy ta c iu phi chng minh. ng thc xy ra khi v ch khi
hoc , 0 v cc hon v.

BI ST 6. Cho cc s thc khng m , , sao cho khng c 2 s no cng bng 0. Chng minh

2 2 2
1 1 1 1
( 2 ) ( 2 ) ( 2 ) a b b c c a ab bc ca
+ +
+ + + + +

PHM KIM HNG

LI GII (V QUC B CN). Ta xt 2 trng hp
Trng hp 1. Nu
2 2 2
4( ) ab bc ca a b c + + + + th
2 2 2
2 2 2 2 2 2 2 2 2
1 1 1 ( 2 ) ( 2 ) ( 2 )
( 2 ) ( 2 ) ( 2 ) ( 2 ) ( 2 ) ( 2 ) ( 2 ) ( 2 ) ( 2 )
a c b a c b
a b b c c a a b a c b c b a c a c b
+ + +
+ + = + +
+ + + + + + + + +
2
2 2 2 2 2 2
9( )
( )
( 2 ) ( 2 ) ( 2 ) ( 2 ) ( 2 ) ( 2 )
a b c
Schwarz
a b a c b c b a c a c b
+ +

+ + + + + + + +
.
Ta chng minh
( ) ( ) ( ) ( ) ( ) ( )
2 2 4 2
2 2
9 ( 2 ) ( 2 ) 9 18 a ab a b a c a ab a ab ( + + +


( )( )
2 2
4 0 a ab ab a

(ng do
2 2 2
4( ) ab bc ca a b c + + + + )
Trng hp 2. Nu
2 2 2
4( ) a b c ab bc ca + + + + . Khng mt tnh tng qut, gi s , , . Ta
chng minh 2( ) a b c + . Bt ng thc ny ng bi v nu ngc li 2( ) a b c + th
2 2 2
4( ) ( 2 2 ) ( ) ( ) 4 0 a b c ab bc ca a a b c b b a c c a bc + + + + = + +
Tr li bi ton, theo bt ng thc ta c
2 2
1 1 2
( 2 ) ( 2 ) ( 2 )( 2 ) a b b c a b b c
+
+ + + +
nn ta ch cn
chng minh
2 1
( 2 2 ) 0
( 2 )( 2 )
b a b c
a b b c ab bc ca

+ + + +
(ng do 2 2 a b c + )
Vy ta c iu phi chng minh. ng thc xy ra khi v ch khi a b c = = .




V K thut phn tch bnh phng cho bt ng thc hon v . . cc bn c th xem thm trong Chuyn bt ng thc
THPT ca Din n Bt ng Thc Vit Nam http://www.ddbdt.tk/ hoc bi vit v chuyn ny trong mc Gii ton nh th
no? bn di.
Bt ng Thc Su Tm V Sng To
This file was downloaded from VietNam Inequality Mathematic Olympiad Resource Page http://www.ddbdt.tk


BI ST 7. Cho cc s thc khng m , , . Chng minh rng

4



VIMF
LI GII (VIMF). Xt


1


1


1


1


1


1



By gi ta c



Php chng minh hon tt.

BI ST 8. Chng minh rng vi mi , , khng m ta c
) ( 3
4
) ( 3
5 1 1 1
2 2 2 2 2 2 2 2 2
c b a ca bc ab a ca c c bc b b ab a + +
+
+ +

+ +
+
+ +
+
+ +

DNG C LM
LI GII (TRN QUC ANH). Ta vit li bt ng thc
2 2 2 2 2 2 2
( ) 5 4( )
3 3( )
a b c bc ab bc ca
b bc c b bc c a b c
+ + +
+ +
+ + + + + +


M theo bt ng thc quen thuc
2 2
( )
2
a b c
b bc c
+

+ +

th ta ch cn chng minh
2 2 2 2 2
1 4( )
3 3( )
bc ab bc ca
b bc c a b c
+ +
+
+ + + +


Dng bt ng thc ta s chng minh bt ng thc cht hn l
2 2 2
2 2 2 2 2 2 2 2 2 2 2 2 2 2 2
1 2 4( ) 2( ) ( ) 2( )
3 2
3 3 3( )
bc ab bc ca bc a b c b c a b c
b c a b c b c a b c b c a b c
+ + + + + + +
+ +
+ + + + + + + + +


Bt ng thc ny ng theo . Php chng minh ca ta hon tt.

BI ST 9. Cho cc s thc dng , , sao cho 3. Chng minh rng

3
Jaanin
LI GII (VIMF). Theo bt ng thc th



Bt ng Thc Su Tm V Sng To
This file was downloaded from VietNam Inequality Mathematic Olympiad Resource Page http://www.ddbdt.tk

Tng t cho 2 bt ng thc cong li v cng li vi nhau ta c



Do ta ch cn chng minh
2


3
Hay


Bt ng thc ny ng do


Vy ta c iu phi chng minh. ng thc xy ra khi v ch khi 1.

BI ST 10. Cho cc s dng , , tha mn . Chng minh bt ng thc
1
1


1
1


1
1


3
19

dragon1
LI GII (NGUYN VIT HNG).
t

, t iu kin bi ta c 1 v bt ng thc cho tng ng


1
1
1

1
1
1

1
1
1

3
19

Theo bt ng thc th
1
1
1

1
1
1

1
1
1

1
1
2


1
1
1
2

1
1
2


2

Ch rng vi mi s dng t th

162
361

1
361

29 1

361 2
0
Nn

2

162
361

1
361

p dng bt ng thc ny vi , , th ta c


2

162
361

3
361

64
361


3
361

3
19

Bi ton c chng minh xong. ng thc xy ra khi v ch khi 3

BI ST 11. Cho cc s thc dng , , sao cho

1. Chng minh rng


33
2

LI GII 1. Bt ng thc cho tng ng vi


33
2


33
2

Xt 1

vi 0; 1
Ta c

1 3

0; 1
T lp bng bin thin ta thy

, 0; 1, khi ta c


33
2


33
2

Php chng minh hon tt.

LI GII 2 (NGUYN C TON). Bt ng thc cho tng ng vi


33
2


Ta s chng minh i din cho mt bt ng thc v tng t cho 2 bt ng thc cn li


33
2


2
33

Bt ng Thc Su Tm V Sng To
This file was downloaded from VietNam Inequality Mathematic Olympiad Resource Page http://www.ddbdt.tk

Theo bt ng thc th


1
2
. 2

. 1


1
2
.
2

4
27

Hay
1


2
33

Php chng minh ca ta hon tt.

LI GII 3 (VIMF). Ta c


T bt ng thc ng

0, ta c 1

, tng t i vi 1

v 1

ta thu
c

2
3

4
3

2
3

4
3

2
3

4
3

Theo bt ng thc th

2
3

4
3

2
3

4
3

2
3

4
3

2
3


4
3

2
3


4
3

2
3


4
3

2
3


4
3


2
3

4
3


V ta ch cn chng minh

2
3

4
3


33
2

Bt ng thc ny tng ng vi bt ng thc hin nhin ng 3

0.
Php chng minh ca ta hon tt.

BI ST 12. Cho cc s thc dng , , sao cho 1 4. Chng minh rng
1

3
1


LI GII (VIMF). Theo bt ng thc th
1 4
4
27

3
3
2

3
T
1



3
1



12
1


12.3
4
3
By gi ta chng minh
3
1

3
3
2
1
Bnh phng 2 v bt ng thc ny ta c bt ng thc tng ng l
5 9 8
T iu kin bi ta c th thy bt ng thc ny tng ng vi
4 4 8 8 2 2
t

th bt ng thc ny c vit li thnh

2 2
Bt ng thc ny c chng minh trong li gii 3 bi O 5, trang 5 trn.
Php chng minh hon tt.

BI ST 13. Cho cc s thc dng , , . Chng minh rng
Bt ng Thc Su Tm V Sng To
This file was downloaded from VietNam Inequality Mathematic Olympiad Resource Page http://www.ddbdt.tk


TRN QUC ANH
LI GII (conan236).
t

th 1 v bt ng thc cn chng minh c vit li thnh


3

27


Mt khc theo mt bt ng thc quen thuc th
27

27 4

27
Do ch cn chng minh

27 3

2 3 0
Bt ng thc trn hin nhin ng. Php chng minh hon tt.

BI ST 14. Cho cc s thc dng , , . Chng minh rng

3

2math
LI GII (VIMF). Theo bt ng thc ta c
1 1 1

3

Mt khc ta li c bt ng thc quen thuc

8


9

T y ta c iu phi chng minh. Php chng minh hon tt.

BI ST 15. Cho cc s dng , , tha mn 1 4. Chng minh rng
4 4 4
1 1 1 3
a b c b c a c a b a b c
+ +
+ + + + + + + +

LI GII (BI C ANH). Theo bt ng thc ta c
4 4 4 2 2 2 2 3 3
4
3 3 3 3 4 2 2 2 2
( ) 1 1
1 1 ( )
a b c a b c b c
a b c
b c b c a b c a b c
+ + + +
+ + = + +
+ + + + + +

Tng t ta cng c
3 3 3 3
4 2 2 2 2 4 2 2 2 2
1 1 1 1
,
( ) ( )
c a a b
b c a a b c c a b a b c
+ + + +

+ + + + + + + +

T suy ra
3 3 3
4 4 4 2 2 2 2
1 1 1 3 2( )
( )
a b c
a b c b c a c a b a b c
+ + +
+ +
+ + + + + + + +


V ta ch cn chng minh bt ng thc sau l
3 3 3
2 2 2 2
3 2( ) 3
( )
a b c
a b c a b c
+ + +

+ + + +

Bt ng thc ny tng ng vi

0
T gi thit th 4 1 4

1
Nn


Do bt ng thc trn hin nhin ng. Php chng minh hon tt.

BI ST 16. Cho cc s thc dng , , sao cho 1. Chng minh rng

27 1 1
NGUYN VIT HNG
LI GII 1 (VIMF). Ta c
Bt ng Thc Su Tm V Sng To
This file was downloaded from VietNam Inequality Mathematic Olympiad Resource Page http://www.ddbdt.tk

3 3
Do ta ch cn chng minh 3

27

27
t th bt ng thc trn c vit li thnh

27

27
Hay

53

63 9 3

0
Bt ng thc trn lun ng. Php chng minh hon tt.

LI GII 2 (NGUYN VIT HNG). Theo bt ng thc ta c

27
Ta s i chng minh
27 27 1 1

1
1

0
Php chng minh hon tt.

BI ST 17. Cho , , khng m tho mn

3. Chng minh rng

1


1


1

3
4

NGUYN VIT HNG
LI GII 1 (NGUYN C TON). t

. Khi 3 v

1


1


1


1


1

Ta chng minh


1

3
4
5 9 3 0
Bt ng thc trn ng theo gi thit , , khng m v 3
ng thc xy ra khi c 2 s bng 0, s cn li bng 3

LI GII 2 (NGUYN VIT HNG). Tng t nh li gii trn, ta chng minh vi 3 th


1

3
4

Ta c


1


1


1


1


1

3
4

Php chng minh hon tt.

BI ST 18. Cho cc s thc phn bit , , . Chng minh rng


9
2

O HI LONG
LI GII 1 (O HI LONG). Trc tin ta c cc ng thc (cc bn t chng minh)


.





.





.


1,


.





.





.


1.
T y p dng bt ng thc ng

0, ta suy ra


5
2
,

2.
Cng 2 bt ng thc trn ta suy ra iu phi chng minh. Php chng minh hon tt.

LI GII 2 (VIMF). Khng mt tnh tng qut, gi s . p dng bt ng thc , ta c
1

,
Mt khc, li p dng bt ng thc , ta c

2

Bt ng Thc Su Tm V Sng To
This file was downloaded from VietNam Inequality Mathematic Olympiad Resource Page http://www.ddbdt.tk

Do

2

9
2

ng thc xy ra khi v ch khi , , , , 0, (vi 0), v cc hon v tng ng.
Php chng minh hon tt.

BI ST 19. Cho cc s thc dng , , sao cho 3. Chng minh rng


1

3
1

NGUYN QUANG HUY
LI GII (VIMF). Bt ng thc cho tng ng vi
3
1


1


1


1

Hay
3
1

1
3
M theo bt ng thc th

1









.


1
3


3
1

Do
3
1

1

3
1

3
1
3
Php chng minh hon tt. ng thc xy ra khi v ch khi 1.


BI ST 20. Cho cc s thc bt k , sao cho 1 v || 2. Chng minh rng

7
NGUYN VIT HNG
LI GII 1 (VIMF).T iu kin 1 ta c th thy rng ch xy ra 2 trng hp: c 2 s cng m,
hoc mt s m v mt s dng.
Trng hp 1. Nu c 2 s u m, th ta t , , 0, iu kin bi tr thnh
1, 2, v bt ng thc cn chng minh c vit li thnh

7. Bt ng thc ny hin nhin


bi vi , 0 th

1 7.
Trng hp 2. Nu mt s m v mt s dng. Khng mt tnh tng qut, gi s 0 . Khi t
0 th iu kin bi tr thnh 1, 2, v bt ng thc c vit li thnh

7
- Nu 0 th bt ng thc hin nhin ng v

0 7
- Nu 0 th 1, 2 v bt ng thc trn tng ng vi

7. Bt ng thc
ny ng bi

3 1. 1 3.2 7
Php chng minh hon tt. ng thc xy ra khi v ch khi , 1; 2 hoc 2; 1

LI GII 2 (NGUYN VIT HNG). Bt ng thc ny tng ng vi

1 3 3 2 0
1

2
3 2 0
Bt ng thc ny hin nhin ng theo gi thit cho.

BI ST 21. Cho cc s thc dng , , . Chng minh rng
4


NGUYN C TON
LI GII 1 (VIMF). Khng mt tnh tng qut, gi s , khi 0, 0
- Nu 0 th bt ng thc hin nhin ng
- Nu 0, kt hp vi 0, 0 th ta ch cn chng minh bt ng thc
khi , , l di 3 cnh ca mt tam gic
Vi cc k hiu thng thng. p dng cng thc v ch

. Ta c
Bt ng Thc Su Tm V Sng To
This file was downloaded from VietNam Inequality Mathematic Olympiad Resource Page http://www.ddbdt.tk



. 16

. 16

44


Php chng minh hon tt.

LI GII 2 (NGUYN C TON). Khai trin, bt ng thc tng ng vi
6 4 4 2 2 2 2 3 3 3 2 2 2
( ) ( ) 2 3 2 ( ) 6 0 a ab a b a b a b a b abc a abc ab a b a b c + + + + + +


( ) ( ) ( )
6 3 4 4 2 2 2 2 3 3 3
3 ( ) ( ) 2 2 3 ( ) 0 a abc a ab a b a b a b a b abc a abc ab a b + + + + + + +


Ta s dng cc hng ng thc
6 3 4 4 4 4 4 4 2
1
3 ( ) ( )( ) ( )( ) ,
2
a abc a ab a b a a b a c a b c a b + + = = +


3 2
1
3 ( ) ( )( ) ( )( )
2
a abc ab a b a a b a c a b c a b + + = = +


T a bt ng thc cn chng minh v dng
2 4 4 4 2 2
( ) ( 2 2 ( )) 0 a b a b c a b abc a b c + + +


t
4 4 4 2 2
2 2 ( ),
a
S b c a b c abc b c a = + + +
4 4 4 2 2
2 2 ( ),
b
S a c b a c abc a c b = + + +
4 4 4 2 2
2 2 ( )
c
S a b c a b abc a b c = + + +
Khng mt tnh tng qut, gi s a b c . Ta c
2 2 2 3
( ) (2 ) 2 ( ) 0,
c
S a b c a b c abc b c = + +
4 4 4
( ) 2 ( )( ) 0,
b
S a b c ac b c a b = + +
2 2 6 2 4 4 2 4 2 4 3 2 2 3 6 2 4 2 4 4 2 4 3 2 3 2
2 2 2 2 2 2 2 2
b a
a S b S b b c a b b c b ac b ac a b c a a c a b a c a bc a bc a cb + = + + + + + + +
6 6 2 4 4 2 3 3 2 2 4 2 4 3
( ) 2 ( ) 2 ( ) 2 ( )( ) a b a b a b ab c b a b c a b b c a c a c a b b c = + + + + + +
6 6 2 4 4 2 3 2 3 4 2 4 3 2 3 2
( ) 2 ( ) 2 ( ) ( 2 2 2 2 ) b a a b a b b c a b b ac b a a c a bc a bc a cb > + + + + + +
2 4 3 2 2 2 2 2 2 2 3 4 2 2
( ) ( 2 2 2 2 2 2 2 2 ) 0 a b a a b a b a c a bc c ab b ac b a b b c = + + + + + + >
T theo nh l . . ta c iu phi chng minh. Php chng minh hon tt.

CH . Mt kt qu tng t nhng yu hn xut hin trong cun Old And New Inequality, l



BI ST 22. Cho , , l di ba cnh ca mt tam gic khng t. Chng minh rng
2

1
DNG C LM
LI GII (VIMF). Vi , , l di 3 cnh tam gic, ta c cc ng thc sau (k hiu vn nh thng)
4, 2,

8 2

12 6


Th vo bt ng thc cn chng minh th c bt ng thc tng ng l
22

12 6

22

8 2


4
2

12 6

6 3

6 3

6 3

6 3


Bt ng thc ny tng ng vi

8 3


V phn chng minh bt ng thc cc bn c th tham kho thm trong cun NHNG VIN KIM CNG
TRONG BT NG THC TON HC trang 685 ca Trn Phng nm 2009
Php chng minh hon tt. ng thc xy ra khi v ch khi hoc , 0 v cc hon v, hoc
2 2 v cc hon v.

BI ST 23. Cho , , , l cc s khng m. Chng minh
2 2 2 2
2 2 2 2 81
| || || || |
+ + + + | | | | | | | |
+ + + + | | | |
| | | |
| | | |
+ + + +
\ \ \ \
\ \ \ \
a b c d a d b c
a c a c b d b d

VIMF
LI GII (leedt26). Theo bt ng thc ta c
( )( ) ( )
2
2 2
2 2 1 + + + + x y x y (1)
p dng vo bt ng thc trn ta c
Bt ng Thc Su Tm V Sng To
This file was downloaded from VietNam Inequality Mathematic Olympiad Resource Page http://www.ddbdt.tk

2 2 2
2 2 2
| |
+ + + | | | | | |
+ + + |
| | |
|
+
| |
|

+ +
\ \

|
\
\
\

a b c d b d
a c a c a c

Tng t
2 2 2
2 2 2
| || |
+ + + | | | | | |
+ + + | |
| | |
| |
+ + +
\ \ \
\ \
a d b c a c
b d b d b d

Nhn v vi v hai bt ng thc ny ri li dng (1) thu c
2 2 2 2 2 2
2 2 2 2 2 2
| || || || |
+ + + + + + | | | | | | | | | | | |
+ + + + + + | | | |
| | | | | |
| | | |
+ + + + + +
\ \ \ \ \ \
\ \ \ \
a b c d a d b c b d a c
a c a c b d b d a c b d
4
4
1 3 81
| |
+ +
+ + =
|
|
+ +
\
b d a c
a c b d

Php chng minh hon tt.

BI ST 24. Cho , , l cc s dng tho mn 1. Chng minh
2 2 2
2
+ + +
+ +
+ + +
a b b c c a
b c c a a b

LI GII 1 (NGUYN VIT HNG). Ta c
2 2
2( ) ( ) ( )( ) + + + +
+ = +
+ + +
a b a b a b a c
a b
b c b c b c
2 2
1 ( ) ( )( )
2
2
| | + + + +
= +
|
+ + +
\

a b a b a b a c
b c b c b c

Bt ng thc cui ng theo v gi thit 1. Php chng minh hon tt.

LI GII 2 (VIMF). Theo bt ng thc th
2 2 2 2 2 2
2 2 2
3( ) 3
( )
2( ) 2
+ +
+ + = + +
+ + + + +
a b c a b c
a b c
b c c a a b a b c

V
2
2 2
( ) 2
1
+ +
+ + =
+ + + + +

b c a a b c
b c c a a b ab c c

Cng 2 bt ng thc ny th c
2 2 2
2 2 2
2
3 2
( )
2 1
+ + +
+ + + + +
+ + + +

a b b c c a
a b c
b c c a a b c

D dng kim tra bt ng thc
2 2 2
2
3 2
( ) 2
2 1
+ + +
+

a b c
c

ng vi
2
1
3

c
Php chng minh hon tt.

BI ST 25. Cho cc s dng , , tho mn: . Chng minh
2 2 2
3 1 1 1 + + + + + + + + xy yz zx x y z
LI GII (NGUYN VIT HNG). t , , , , l 3 gc ca 1 tam gic
Bt ng thc cho tng ng vi
2 2
1 1
( 1) 1 1
.
x y tgA tgB cosC
xy x x
z tgC cosA cosAcosB cosA
+ +
+ +


Bt ng thc cui ng theo . Php chng minh hon tt.

BI ST 26. Cho , , l cc s thc dng tha mn
2 2 2
3. a b c + + =

Chng minh rng

5 5 5
3 3 3
3
2 2 2
+ +
+ + +
a b c
a bc b ca c ab

minhhoang
LI GII (NGUYN ANH TUN). t
5 5 5
3 3 3
3 3 3
, ( 2 ) ( 2 ) ( 2 )
2 2 2
= + + = + + + + +
+ + +
a b c
A B a a bc b b ca c c ab
a bc b ca c ab

Theo bt ng thc th
( )
3
2 2 2 2
27 + + = A B a b c
Bt ng Thc Su Tm V Sng To
This file was downloaded from VietNam Inequality Mathematic Olympiad Resource Page http://www.ddbdt.tk

Ta s chng minh 9 B
( )
2
4 4 4 2 2 2
6 + + + + + a b c abc a b c
2 2 2 2 2 2
3 + + a b b c c a abc
t
2 2 2
, , = = = x a y b z c

th 3. Khng mt tnh tng qut, gi s , , , suy ra 1 x

Ta cn chng minh 3 + + xy yz zx xyz
t ( , , ) 3 = + + f x y z xy yz zx xyz
( ) ( )
( )
2 2
1
( , , ) , , 6 0
2 2 4
+ + | |
= +
|
\
y z y z
f x y z f x y z x y z
( )
2
1
( , , ) , , 1 (3 ) 0
2 2 4
+ + | |
=
|
\
y z y z
f x y z f x x x
iu ny c ngha l 9 B suy ra
2
3 A . Php chng minh hon tt.

NHN XT. Bn minhhoang b sung thm mt cch chng minh
2 2 2 2 2 2
3 + + a b b c c a abc
Bnh phng 2 v, bt ng thc tng ng vi
2 2 2 2 2 2 2 2 2 2 2 2 2 2 2 2
( ) 9 3 ( ) a b b c c a a b c a b c a b c + + = + +
y l bt ng thc quen thuc
2
( ) 3( ) + + + + x y z xy yz zx , trong :
2 2 2 2 2 2
, , = = = x a b y b c z c a

BI ST 27. Cho , , l cc s dng. Chng minh
2 2 2 2 2 2
2( )
3
+ + + +
+ + +
+ +
a b b c c a a b c
ab bc ca
a b c

NGUYN ANH TUN
LI GII (NGUYN VIT HNG). t 1 v
2 2 2
1 + + = a b c x , ta c
2 2 2
2
1 1
( ) 2
+ +
=
+ + + + +
a b b c c a
a b c a b c x
2 ( 1)
1 0
2 3 3( 2)

= =
+ +
x x x
VT VP
x x

Php chng minh hon tt.

BI ST 27. Cho cc s thc dng , , sao cho

1. Chng minh rng




4

LI GII 1 (VIMF). T gi thit ta c , nn


,
Theo bt ng thc th


8


8

3
4



4
8
,
Tng t ta cng c



4
8
,



4
8
,
Cng cc bt ng thc trn vi nhau ta c iu phi chng minh.

LI GII 2 (VIMF). T gi thit ta c

, suy ra



1

1
1

V tng t



1
1
,



1
1

Cng li ta c


1

V ta chng minh


1


4
4

3
Ch rng

9, nn t gi thit ta c 9. Do
Bt ng Thc Su Tm V Sng To
This file was downloaded from VietNam Inequality Mathematic Olympiad Resource Page http://www.ddbdt.tk


1
3

3 , 3


T 2 bt ng thc ny ta c iu phi chng minh.

LI GII 3 (Toanlc_gift). Ta c
2
2 2 2
( )
1
( )( )( )
ab ab a b abc a
a abc
a a abc a abc
b abc c abc a abc a bc a bc a bc
abc
+ + +
| |
= = =
|
+ + + + + +
\



[ ] [ ]
2 2 2
2 2
( ) ( )( )
2( )( )( )
( )( )( ) ( )( )( )
abc ab a b a b c ab bc ca
a b b c c a
a abc a a b c
a b b c c a a b b c c a
abc
+ + + + + + +
+ + +
= =
+ + + + + +



2 2 2 2 2 2
9 9 9( )
( )( )( ) 8( )( ) 8( ) 8( )
a b c a b c abc ab bc ca
a a a a
a b b c c a a b c ab bc ca a b c a b c
+ +
= = =
+ + + + + + + + + + +

4
a b c + +



LI GII 4 (minhhoang). T gi thit , suy ra
Bt ng thc tng ng vi
2 2 2
3 3
( ) 0 0 0
4 4( )
a a a abc a ab bc ca
a bc a bc a bc


+ + +



Ch rng , , 1 ( t iu kin ), gi s a b c . Khi , d dng nhn thy
2 2 2
1 1 1
, a ab bc ca b ab bc ca c ab bc ca
a bc b ca c ab

+ + +

Theo bt ng thc , ta c
2
2
3 1
( ) 0
a ab bc ca
a ab
a bc a bc
| |

|
+ +
\




LI GII 5 (minhhoang). Theo gi thit suy ra
Theo bt ng thc , c
2 2 2 2 2 2 2 2
3 3 3 3 3 3
( ) ( )
( ) ( )( )
a b c a b c
VT
a b c abc a b c a b c ab bc ca a b c
+ + + +

+ + + + + + + + + + + +

Vy ta i chng minh bt ng thc
2 2 2 2
3 3 3
( )
( )( ) 4
a b c a b c
a b c ab bc ca a b c
+ + + +

+ + + + + + +

2 2 2 2 4 4 4 3 2 2
4( ) 2 2( ) 5 ( )
sym
a b c a b c a b a b abc a b c + + + + + + + + +


4 4 4 2 2 3
3( ) 6 2 5 ( )
sym
a b c a b a b abc a b c + + + + + +


Bt ng thc ny suy ra t cc bt ng thc quen thuc
4 3
2( ( )) 2
sym
a abc a b c a b + + +

( bc 2),
2 2
6 6 ( ) a b abc a b c + +

,
4
( ) a abc a b c + +


Cng cc v ca bt ng thc li, ta c iu phi chng minh.

LI GII 6 (chihieu112). t , , . T gi thit ta c 9, a b c + +

Bt ng thc tng ng vi
( ) ( ) ( ) 3 1 1 1
( )
4 4
a a bc abc b b ca abc c c ab abc a b c
a b c abc
a bc b ca c ab a bc b ca c ab
+ + + + + | |
+ + + + + +
|
+ + + + + +
\

Ta c
2 2 2 2
1 ( 3 ) (2 2) 2
( )
( 2 ) 2 2 1 1
r q pq r pr q p q
abc
a bc r r r p q q pr p p p
+ +
= = =
+ + + + +


V ta ch cn chng minh
2
3 2
3 3 8
4 1
q
p p p q
p

. Hay chng minh


2
2
8( )
3 3
3
p
p p (do
2
3 p q )
2 2
9 9 8 ( 9) 0 p p p p p

Bt ng thc ny hin nhin ng v 3. Php chng minh hon tt.
Bt ng Thc Su Tm V Sng To
This file was downloaded from VietNam Inequality Mathematic Olympiad Resource Page http://www.ddbdt.tk


BI ST 28. Cho cc s thc dng , , , sao cho 1. Chng minh rng
1 2 1 2 1 2 1 2 0
LI GII (minhhoang). Khng mt tnh tng qut , gi s a b c d
Khi , xt
( )
( , , , ) , , , f a b c d f ac b ac d ( 1)( 2) ( 1)( 2) 2( 1) 2) ( a a c c ac ac = +
2 2
( ) 3( ) a c a c =
2 2
( ) (( ) 3) a c a c = +
Ch rng 1 ac (Do ac bd v 1) nn
2
( ) 3 4 3 1 a c + =
Do ( , , , ) ( , , , ) 0 f a b c d f ac b ac d
Theo nh l dn bin mnh , ta ch cn chng minh , , , 0 , trong
3
1 t abc =
Hay 3( 1)( 2) ( 1)( 2) 0 t t d d +
Do
3
1 t d = nn bt ng thc cn chng minh tng ng vi
3 3
1 1
3( 1)( 2) 1 2 0 t t
t t
| || |
+
| |
\ \
3 3
2
6
(1 )(2 )
3 9 6 0
t t
t t
t

+ +
8 7 6 3 2 6 5 4 3 2
3 9 8 3 1 0 ( 1) (3 3 3 2 1) 0 t t t t t t t t t t t + + + + + + ( 1 )
Do 1 t Nn
6 5 4 3 2 5 3 2 2
3 3 3 2 1 2 ( 1) ( 1)( 1) 3 2 1 0 t t t t t t t t t t t t t + + + + = + + + + ng, tc l ( 1 ) ng
Bt ng thc c chng minh. ng thc xy ra khi v ch khi 1

BI ST 29. Cho , , l cc s thc dng.Chng minh bt ng thc sau
cyc
4 4 9
2
xy yz zx
x y
+ +


LI GII 1. Ta c
2 2 2 2 2 2 2
( ) ( 4 4 )( 4 4 ) 4( 2 2 3 ) ( 4 4 )( ) 0 y z zx xy yz xy yz zx xy xz y z yz xyz xyz x y z y z + + + + + + + + + = + +
2
4 4 81
4
yz zx xy
y z
| |
+ +
|
|
+
\

2
( 4 4 )( 4 4 ) 4 4 81
2
( ) ( )( ) 4
zx xy yz xy yz zx yz zx xy
y z z x x y
+ + + + + +
= +
+ + +


2 2 2 2
2
4 4 2 2 3 81
4
( ) ( )( )( ) 4
yz zx xy xy xz y z yz xyz
y z y z z x x y
+ + + + + +
+
+ + + +

2
4 4 12( )( ) 81
( ) ( )( )( ) 4
yz zx xy x y z yz zx xy
y z y z z x x y
+ + + + + +
= +
+ + + +


5 5 4 2 2 4 3 3 4 3 2 2 2
2 2 2
16 ( ) ( ) 30 2 2 ( ) 6
4( ) ( ) ( )
y z yz y z y z y z x yz x yz y z x y z
x y y z z x
+ + + + +
=
+ + +


2 2 2
2 2 2
( ) (16 31 16 ) 2 ( )( )
0
4( ) ( ) ( )
yz y z y yz z xyz x x y x z
y z z x x y
+ + +
=
+ + +



LI GII 2. Ta c
2 2 2 2
2( 2 2 3 )
( 4 4 )( 4 4 )
xy xz y z yz xyz
zx xy yz xy yz zx
y z
+ + + +
+ + + +
+

Ta c
2
4 4 yz zx xy
y z
| |
+ +
|
|
+
\

2
( 4 4 )( 4 4 ) 4 4
2
( )( ) ( )
zx xy yz xy yz zx yz zx xy
z x x y y z
+ + + + + +
= +
+ + +


2 2 2 2
2
2 2 3 4 4
4
( )( )( ) ( )
xy xz y z yz xyz yz zx xy
y z z x x y y z
+ + + + + +
+
+ + + +

2
12( )( ) 4 4
( )( )( ) ( )
x y z yz zx xy yz zx xy
y z z x x y y z
+ + + + + +
= +
+ + + +


2
2 2
45 3 ( )
4
4 ( ) 4 ( )
yz zx xy y z
y z y z
+ +
= + +
+ +

45 9 81
4 0
4 4 4
+ + =
Bt ng thc trn ng do bt ng thc 1996:
2 2 2
4 4 4 9
( ) ( ) ( ) y z z x x y yz zx xy
+ +
+ + + + +


LI GII 3. Ta c
Bt ng Thc Su Tm V Sng To
This file was downloaded from VietNam Inequality Mathematic Olympiad Resource Page http://www.ddbdt.tk

2
2
4 4 ( 4 4 )( 4 4 ) 4 4
2
( )( ) ( )
yz zx xy zx xy yz xy yz zx yz zx xy
y z z x x y y z
| |
+ + + + + + + +
= + |
|
+ + + +
\




2 2 2 2
2
2 2 3 4 4
4
( )( )( ) ( )
xy xz y z yz xyz yz zx xy
y z z x x y y z
+ + + + + +
+
+ + + +



{ }
2 2 2
2 2 2
( ) ( ) 31( ) 33 ( ) 64 ( )
81
4 16( ) ( ) ( )
y z y z x x yz x y z xyz y z
x y y z z x
( + + + + + +

= +
+ + +

81
4


LI GII 4. t
( 4 4 ) 9
2
cyc
xy yz zx
P
x y
+ +
=
+


Ta c
2 2 2 2
4 [( ) ( ) ( 4 4 ) 2( )( )( ) ( 4 4 )( 4 4 )]
cyc
P y z z x xy yz zx x y y z z x xy yz zx yz zx xy = + + + + + + + + + + + +


2 2 2
81( ) ( ) ( ) x y y z z x Q + + + =
Nhng ta c
2 2 2 2 2 2
( ) ( 4 4 )( 4 4 ) 4( 2( ) 3 ) z x xy yz zx yz zx xy x y z y z x zx xyz + + + + + + + + +
2
( ) (4 4 ) 0 x z x y z xyz = + +
Do
2 2 2 2 2 2
( ) ( 4 4 )( 4 4 ) 4( 2( ) 3 ) z x xy yz zx yz zx xy x y z y z x zx xyz + + + + + + + + + .
Nn
2 2 2
4 [( ) ( ) ( 4 4 )]
cyc
P y z z x xy yz zx + + + +

2 2 2 2
2( )( )( ) [4( 2( ) 3 )]
cyc
x y y z z x x y z y z x zx xyz + + + + + + + +


2 2 2 2
48( 2 )( 3 )
cyc cyc cyc cyc
x y xy xyz x y xy xyz + + + + +

2 2
4 [( ) ( ) ( 4 4 )]
cyc
y z z x xy yz zx = + + + +


2 2 2 2
48( 2 )( 3 )
cyc cyc cyc cyc
x y xy xyz x y xy xyz + + + + +


Ta c
2 2
4 [( ) ( ) ( 4 4 )]
cyc
y z z x xy yz zx + + + +

5 4 4 2
16 34 32
sym sym sym
x y x yz x y = + +


3 2 3 3 2 2 2
148 18 40
sym sym sym
x y z x y x y z + + +


V
2 2 2 2
48( 2 )( 3 )
cyc cyc cyc cyc
x y xy xyz x y xy xyz + + + +


4 2 3 3 4 3 2 2 2 2
48 48 48 336 96
sym sym sym sym sym
x y x y x yz x y z x y z = + + + +


V
2 2 2
81( ) ( ) ( ) x y y z z x + + +
4 2 4 3 3 3 2 2 2 2
81 81 486 135
sym sym sym sym sym
x y x yz x y x y z x y z = + + + +


Do
2 5 4 2 3 3 4 3 2 2 2 2
16 15 2 0
sym sym sym sym sym sym
P Q x y x y x y x yz x y z x y z + +


Tht vy
5 4 2 3 3
16 15 0
sym sym sym
x y x y x y

(ng do bt ng thc Muirhead)
V
4 3 2 2 2 2
2 0
sym sym sym
x yz x y z x y z +

(ng)
Do bt ng thc Schur ( ( )( ) ( )( ) ( )( ) 0 xyz x x y x z y y z y x z z x z y + +
Nn
2
P Q P Q .Vy bt ng thc d cho c chng minh.

BI ST 30. Chng minh vi cc s thc khng m , , th
2 2 2 2 2 2
1
( ) 5 ( ) 5 ( ) 5
a b c
b c c c a a a b b
+ +
+ + + + + +

LI GII (quanghuyhungnguyen). t
2 2 2 2 2 2
1,
( ) 5 ( ) 5 ( ) 5
a b c
A
b c c c a a c a a
= + +
+ + + + + +
2 2 2 2 2 2
(( ) 5 ) (( ) 5 ) (( ) 5 ) B a b c c b c a a c c a a = + + + + + + + +
p dng bt ng thc ,ta c
3
( ) AAB a b c + +
Ta ch cn chng minh
3
( ) a b c B + +
iu ny tng ng vi
2 2 2
( ) ( ) ( ) 0 a a b b b c c c a + + (ng)
Bt ng Thc Su Tm V Sng To
This file was downloaded from VietNam Inequality Mathematic Olympiad Resource Page http://www.ddbdt.tk

Vy ta c iu cn chng minh. ng thc xy ra khi v ch khi .

BI ST 31. Cho cc s thc khng m , , . Chng minh
2 2 2 2 2 2
1 1 1 1 2
2 2 2 a bc b ca c ab ab bc ca a b c
+ + +
+ + + + + + +

LI GII (VIMF). Bt ng thc ny tng ng vi
( )
( )( )
2
2 2 2 2 2 2
1 1 1
2 2 2
a b c
a bc b ca c ab ab bc ca a b c
+ +
+ +
+ + + + + + +

Theo bt ng thc th
( )
( ) ( ) ( ) ( ) ( ) ( )
2
2 2 2 2 2 2
2 2 2
1 1 1
2 2 2
2 2
b c c a a b
a bc b ca c ab
b c a bc c a b ca a b c ab
+ + + + +
+ +
+ + +
+ + + + + + + +

Do ta ch cn chng minh
( )( ) ( ) ( ) ( ) ( ) ( ) ( )
2 2 2
2 2 2 2 2 2
4 2 2 ab bc ca a b c b c a bc c a b ca a b c ab + + + + + + + + + + + +
Khai trin ra ta c bt ng thc tng ng l
3 3 3 3 3 3 2 2 2 2 2 2
2( ) a b ab b c bc c a ca a b b c c a + + + + + + +
Bt ng thc ny hin nhin ng theo bt ng thc .
ng thc xy ra khi v ch khi hoc , 0 v cc hon v. Php chng minh hon tt.

BI ST 32. Cho , , 0 a b c Chng minh rng
2
( ) ( ) 4 3 ( )(1) abc a b c a b c abc a b c + + + + + + +

vnisky
LI GII 1 (Toanlc_gift). Khng mt tnh tng qut, gi s 3. Khi
2
( ) ( ) 4 3 ( ) abc a b c a b c abc a b c + + + + + + + ( ) 9 12 abc a b c abc + + +
Theo bt ng thc ta c
3 2 2 2
( ) 3 , abc a b c a b c + +
3
3 3 , a b c abc = + + 6 6 abc
Do
3 2 2 2 3
( ) 9 3( ) 6 12 abc a b c abc a b c abc abc + + + + +

LI GII 2 (Conan Edogawa).
( )
3
(1) 4 3
a b c
a b c
a b c abc
+ +
+ +
+
+ +

Do
6
3 a b c abc + + nn cn chng minh
( )
3
6
3
4 3(2)
a b c
abc
a b c abc
+ +
+
+ +

t
6
6 6
3.
3
a b c abc
x
abc abc
+ +
= = . Khi
3 3 3
3
3 3
(2) 4 3 4 3
3 3 3
x x x
x
x x
+ + + +
Bt ng thc trn ng theo . Php chng minh hon tt.

BI ST 33. Cho , , , , , 0 tha mn . Chng minh rng
x y z a b b c c a + + > + + + + +

vnisky
LI GII (Toanlc_gift). T gi thit 1
a b c
yz zx xy
+ + = . t ; ;
a b c
m n p
yz zx xy
= = = 1
Bt ng thc cn chng minh tr thnh ( ) ( ) ( ) z ym xn x zn yp y zm xp x y z + + + + + < + +
Theo bt ng thc th


Php chng minh hon tt.

BI ST 34. Cho , , 1. Chng minh rng
Bt ng Thc Su Tm V Sng To
This file was downloaded from VietNam Inequality Mathematic Olympiad Resource Page http://www.ddbdt.tk

( ) ( ) ( )
4 4 4
2 2 2
48
1 1 1
x y z
y z x
+ +


LI GII (minhhoang). Ap dng bt ng thc , ta c
4
2
16( 1) 16( 1) 16 32
( 1)
x
y y x
y
+ + +

4
2
32( ) 16
( 1)
x
x y
y
+


Lm tng t cho cc biu thc cn li, cng cc v ca cc biu thc, ta c iu phi chng minh. ng thc
xy ra khi v ch khi 2.

BI ST 35. Cho 2 s thc dng p,q sao cho 2. Chng minh
3 4
q p p q
p q p q +

LI GII (minhhoang). Ta vit li bt ng thc di dng
2
2 2 2 2
3( ) 4
q p p q
p q p q +
p dng bt ng thc suy rng cho VT, ta c
2
2 2
2
3( )
2
p q
VT pq
| | +
+
|
\

V ta ch cn chng minh
2
2 2 4
2
( )
3( ) 4
2 4
p q p q
pq
| | + +
+ =
|
\
3 3 2 2
2 p q q p p q + (ng)
Vy bt ng thc c chng minh. ng thc xy ra khi v ch khi 1.

BI ST 36. Cho cc s thc dng , , sao cho 1. Chng minh
1 1 1
1
3
a b c
a b c


LI GII 1 (minhhoang). Bt ng thc tng ng vi
2 2 2
1
3
b c c a a b
a b c
+ + +


p dng bt ng thc suy rng cho , ta c
2( ) 1
2 3
ab bc ca
VT ab bc ca
+ +
= + + ( do 1 )
Php chng minh hon tt. ng thc xy ra khi v ch khi

.
LI GII 2 (VIMF). Bt ng thc ny tng ng vi


Ch rng 1 3


Do ta ch cn chng minh


Ly logarith Neper 2 v ta c bt ng thc tng ng l

1
3

n y c th dung trc tip bt ng thc i n kt lun hoc cng c th a v
0
V v lun cng du nn bt ng thc trn l ng. Php chng minh hon tt.

BI ST 37. Cho cc s thc dng , , sao cho 3. Chng minh
( ) ( )
2 2 2
1 1 ( 1)
2
a b c a b c
a b ab b c bc c a ca
+ + +
+ +
+ + + + + +

LI GII 1 (NGUYN ANH TUN). Ta c
2 3
3 2 2
( 1)
3
3
a b ab ab ab
a a a
a b ab a b ab
a b
+
= =
+ + + +

Suy ra
2 3 3 3
( 1)
2
3
a b ab bc ca
a b c
a b ab
+ + +
+ +
+ +


ng thc xy ra khi 1

LI GII 2 (minhhoang). p dng bt ng thc , ta c
2 2 2 2
( )
1 1 1 1 1 1
a b c a b c
VT
b c a b c a
a b c a b c
b c a b c a
+ +
= + +
+ + + + + + + +
+ + + + + +

Bt ng Thc Su Tm V Sng To
This file was downloaded from VietNam Inequality Mathematic Olympiad Resource Page http://www.ddbdt.tk

Ta s chng minh
2
( )
2
1 1 1
a b c
b c a
a b c
b c a
+ +

+ + + + +
+ + +
9
2
3
1 1 1
b c a
b c a

+ + +
+ + +
2
1 1 1
b c a
b c a
+ +
+ + +
1 1 1 3
1 1 1 2 b c a
+ +
+ + +

n y, p dng bt ng thc quen thuc
1 1 1 9
x y z x y x
+ +
+ +
, kt hp vi iu kin 1, ta c
iu phi chng minh.

BI ST 38. Cho 0 , , 1. Chng minh rng
( ) ( ) ( ) ( )
1 1 1
2 2 2 4
a b c
a b c
b c c a a b a b c

+ + + + + < + +
LI GII (minhhoang). Theo bt ng thc suy rng th
1
2 ( ) 2 (1 )( )
a a
b c a a b c

+ + +
1
2 ( ) 2 (1 )( )
b b
c a b b a c

+ + +
1
2 ( ) 2 (1 )( )
c c
a b c c a b

+ + +
Cng 3 bt ng thc trn v theo v, ta c 4( ) 2( ) 4( ) VT a b c ab bc ca a b c VP + + + + + + =
Php chng minh hon tt.

BI ST 39. Cho cc s thc bt k , sao cho

3. Chng minh rng


43 3

43 3
LI GII (VIMF). Bt ng thc cn chng minh tng ng vi
43 3

43 3

3

Hay

43 6.

43 6.

43 6.

43 6.


Php chng minh hon tt.

BI ST 40. Cho cc s thc dng , , . Chng minh rng


2
2


2
LI GII (VIMF). Theo bt ng thc ta c


2
2




.


. 2
2

.

2


2
2

4
2

2 2
2
2
Php chng minh hon tt. ng thc khng xy ra.

BI ST 41. Cho cc s thc dng , , sao cho

2
1

3
1
1
Tm gi tr ln nht ca

.
LI GII (VIMF). Theo bt ng thc , ta c
1
1
1

1


1


1
5


1,
Bt ng Thc Su Tm V Sng To
This file was downloaded from VietNam Inequality Mathematic Olympiad Resource Page http://www.ddbdt.tk

1
1
1

1


1


1


1
5


1
.

1
.

1


2,
1
1
5


1
.

1
.

1


3,
1
1
1

1


1


1
5


1
.

1

.

1


4,
1
1
5


1
.

1

.

1


5,
1
1
5


1
.

1

.

1


6,
Nhn 1, 2, 3, 4, 5, 6 vi nhau v theo v ta c
1
1
.
1
1

.
1
1

.

1
.

1

.

1


1
15625

Vy, gi tr ln nht ca

. t c ti

.

BI ST 42. Cho , , 0 a b c . Chng minh rng
2
2 2
( ) 4( ) a b c ab bc ca
b bc c a b c
+ + +

+ + + +


VIMF
LI GII. p dng bt ng thc Cauchy-Schwarz, ta c
2 2 2 2
2 2 2 2 2
( ) ( ) 4( ) 4( )
3
+ + + + + +
= =
+ + + + + + +


sym
a b c a b c ab bc ca ab bc ca
b bc c ab abc ac a b abc a b c

Php chng minh hon tt. ng thc xy ra khi v ch khi .


BI ST 43. Cho cc s thc dng , , . Chng minh
2
3


NGUYN ANH TUN
LI GII (VIMF). Bt ng thc tng ng vi
2




2
2




2
2

2

3

2

3

3
2

3
2

3
2


Ta a v dng . . vi


3
2


3
2


3
2


Khng mt tnh tng qut, gi s . t , , 0. Ta chng minh

0,
hay

4 3

2 3

0
Ta c

6 0
Nn bt ng thc trn l ng, tc l

0
Li c


3
2

4 3

0
Mt khc,
Bt ng Thc Su Tm V Sng To
This file was downloaded from VietNam Inequality Mathematic Olympiad Resource Page http://www.ddbdt.tk


3
2

3
2


2
2

2

2

0
Do

0
Php chng minh hon tt.

BI ST 44. Cho , , , 0, tho mn 1. Chng minh rng bt ng thc
, , ,
1
1 4
x y z t
xy
x y

+ +


NGUYENXUANHUY_DHXD
LI GII. Ta c
1 1 4
( 4 4 4 4)
1 1 1 1
1 36 36 36 9
4 4 4 4
xy xy xy x y xy
x y x y
x y
= + + + + + = + +
+ +
+ + + + +

Lm tng t cho cc biu thc cn li, c
2
, , , , , ,
1 4 1 ( ) 1
( )
1 12 9 12 6 4
x y z t x y z t
xy x y z t
x y z t xy
x y
+ + +
+ + + + + =
+ +


Bt ng thc c chng minh. ng thc xy ra khi v ch khi
1
4
= = = = x y z t


BI ST 45. Cho cc s thc , tha 2 3 v 3 4. Chng minh

10
LI GII (VIMF).
- Nu 1 th

10 (ng)
- Nu 1 th 3 3
Li c 4 2 2 6 3
V 4 nn chng minh

10 th ta ch cn chng minh


Hay 3

10 3

0 33 0
Bt ng thc ny ng do 3 v 3 . Php chng minh hon tt

BI ST 46. Cho a, b, c l ba s thc ty . Chng minh rng
6 2


can_hang2007
LI GII (leedt26). Nu 6 th bt ng thc lun ng
Bt ng thc cn chng minh tng ng vi
4


Ta c 4

,
V 4


p dng bt ng thc Cauchy-Schwarz ta c
16

2 3


Php chng minh hon tt. ng thc xy ra khi v ch khi 0 hoc 0 hoc 0.

BI ST 47. Cho , , 0 . Chng minh rng
2 2 2 2 2 2
2 2 2 2 2 2
( ) ( ) ( ) + + + + +
+ +
+ + + + + + + +
a b c b c a c a b a b c
b bc c c ca a a ab b a b c

LI GII. Ta c
2 2 2 2
2 2 2 2 2 2 2 2
( ) ( )( ) 4 ( )( ) 4 ( )
( )( ) ( ) ( )( )
+ + + + + + + + +
= =
+ + + + + + + + + + + + + +
a b c a b c ab bc ca a b c ab bc ca a ab bc ca
b bc c b bc c ab bc ca b bc c ab bc ca b c a b c

Bt ng Thc Su Tm V Sng To
This file was downloaded from VietNam Inequality Mathematic Olympiad Resource Page http://www.ddbdt.tk

Nn ta i chng minh
2 2 2 2 2
2 2 2
2
4 ( ) 2( ) ( ) 2
( )( ) ( ) ( ) ( )
+ + + + + +
+ +
+ + + + + + + +

a ab bc ca a b c ab bc ca a
a b c
b c a b c a b c a b c b c

p dng bt ng thc Cauchy-Schwarz ta c
2 2 2 2 2
2
( ) 2 ( )2( )
( ) ( )
( )
+ + + + + +

+ + +
+


ab bc ca a ab bc ca a b c
a b c b c
a b c

Bt ng thc s ng nu
2 2 2 2
2 2 2
2
2 2 2 2 3 2 2 2
( )2( )
( )( )
( )
2( )( ) ( )( ) 2 ( ) 0
+ + + +
+ + + +
+
+ + + + + + +


ab bc ca a b c
a b c a b c
a b c
a b c ab bc ca a b c a b c a b a b ab a b

Php chng minh hon tt.

BI ST 48. Cho , , 0 tha mn 3.Tm GTNN ca biu thc
2 2 2
1 3 1 3 1 3
3 3 3
zx y yz x xy z
y xz z xy x yz
+ + +
+ +
+ + +

NGUYENXUANHUY_DHXD
LI GII. Ta s chng minh GTNN ca biu thc bng
3
2

2 2 2
1 3 1 3 1 3 3
3 3 3 2
+ + +
+ +
+ + +
zx y yz x xy z
y xz z xy x yz

T gi thit suy ra
1 1 1
3 + + =
x y z
. t
1 1 1
, , , = = = a b c
x y z
khi 3
Bt ng thc tr thnh
2
3
3
3
+

a
ac b

Xt biu thc
2 2
4( 3) 3 3
3 2( ( )) 2( )( )
+ + +
=
+ + + + + +
a a a
ac b ac b a b c b a b c

Lm tng t cho cc biu thc cn li, quy bt ng thc v chng minh
3 3
( 3)( ) ( 3)( ) ( 3)( ) 3( )( )( )
2( )( ) 2
+
+ + + + + + + + + + +
+ +

cyc
a
a a c b b a c c b a b b c c a
b a b c

D dng nhn thy
3
24( )
3( )( )( ) 24
27
+ +
+ + + =
a b c
a b b c c a
V
2
2 2 2
2( )
( 3)( ) ( 3)( ) ( 3)( ) 18 18 24
3
+ +
+ + + + + + + + = + + + + + + + =
a b c
a a c b b a c c b a b c ab bc ca
Nn bt ng thc c chng minh. ng thc xy ra khi v ch khi 1, tc l 1

BI ST 49. Cho , , 0 a b c > tha mn 1. Chng minh rng
6( ) 8( )
+ + +
+ + + + + + +
a b b c c a
ab bc ca a b c
c a b

NGUYENVANDUNG_CL
LI GII. Khng mt tnh tng qut, gi s c ax{ , , }. = m a b c t
( ) ( ) ( ) ( ) ( ) ( )
( )( ) ( ) ( )
f a, b, c ab a b bc b c ca c a 6 ab bc ca 8 a b c
a b c ab bc ca 6 ab bc ca 8 a b c 3
= + + + + + + + + + +
= + + + + + + + + +

Khi
2 2 2
( , , ) ( , , ) ( ) ( ( ) 6 8) = + + + + f a b c f ab ab c a b ab c a b c c
Xt biu thc trong ngoc
2 2 2
( ) 6 8 2 6 8 2 2 0 + + + + = + + + + + + > ab c a b c c c ab ab bc ca c c c ab
( do 1 c v 1)
Bt ng Thc Su Tm V Sng To
This file was downloaded from VietNam Inequality Mathematic Olympiad Resource Page http://www.ddbdt.tk

Nh vy ( , , ) ( , , ) 0 f a b c f ab ab c
Xt ( , , ) (2 )( 2 ) 6( 2 ) 8(2 ) 3 = + + + + + f ab ab c ab c ab c ab ab c ab ab c
t c x = khi
2
1 x ab = , c
2 2
2 2
6 5 4 3 2 2 2 2
3 3
2 1 1 2
( , , ) ( )( 2 ) 6( 2 ) 8( ) 3
(2 8 12 2 16 6 2 ( 1) (2 ( 1) 10 2)
= + + + + +
+ + + + + +
= =
f ab ab c x x x x
x x x x
x x x x x x x x x x
x x

Php chng minh hon tt. ng thc xy ra khi v ch khi 1

BI ST 50. Cho , , l 3 s thc dng tha mn 3 + + ab bc ca . Chng minh rng
3
2
+ +
+ + +
a b c
a b b c c a

NGUYENXUANHUY_DHXH
LI GII. p dng bt ng thc Holder, c
2
3
( ) ( )
(
( + + + + +
(
+ + +

a b c
a a b a b c
a b b c c a

Ta quy vic chng minh bt ng thc v chng minh bt ng thc sau
3 2 2 2 3 2
9
( ) ( ) 2( ) 9( ) 9( )
2
+ + + + + + + + + + + + + + a b c a b c ab bc ca a b c ab bc ca a b c
C
3 3 2
( ) ( ) 27 9( ) + + + + + + + + VT a b c a b c a b c ( Theo bt ng thc AM-GM )
Vy bt ng thc c chng minh

BI ST 51. Cho , , 0. Chng minh rng
2 2 2
1 1 1 1 1 1 1 1 1 1
+ + + > + + + + +
+ + + + + a b c a b c a b c a b b c c a

TRUNGDEPTRAI
LI GII.
2 2 2
2
2 2 2
3
2 2 2 2 2 2
1 1 1 1 1 1 1 1 1 1
1 1 1
2( )
( )(( ) ) ( )( )( )
( )( )( )( ) 1 1 1 1 1 1
2( ) ( )
( )(
+ + + + + +
+ + + + +
+ +
+ + + + + + + + + +

+ + + + +
+ + + + +
+ + + + +

+ +
+ + + + +
BDT
a b c a b c a b b c c a a b c
a b c a b c ab bc ca a b b c c a
ab bc ca
a b c a b b c c a
a b c a b c
a b c a b c abc
a b c a
ab bc ca a b b c c a
)( )( ) + + + b b c c a


Chun ho 3, t ; a bt ng thc v dng
( ) ( )
( )( )
2 2
2
2 2 2
2
2 2
6 27
54 18 27 27 6 6
3(3 )
6
162
27 27 6 18 6 18 6
6
162 6 18 6
+
+ + +

+
+ + + +
+
+ + +
r
q r q q r qr r q r
q r
q q r
r
q q r r q q r r q q r
q q r
q q r q q r

Ch l 3 q nn 2 .(18 2 ) 144 162 VP q q + <
Php chng minh hon tt. ng thc khng xy ra.

BI ST 52. Cho , , l cc s thc dng. Chng minh rng
2 2 2
2
3 3 3
+ + <
+ + +
a b c
a bc b ca c ab

Bt ng Thc Su Tm V Sng To
This file was downloaded from VietNam Inequality Mathematic Olympiad Resource Page http://www.ddbdt.tk

MINHHOANG
LI GII. Khng mt tnh tng qut , gi s a b c
2 2 2
2 2 2 2 2
2 2 2 2 2
(1 ) (1 )
3 3 3
3 3
3 ( 3 ) 3 ( 3 ) 3
3 3 1
3 ( 3 ) 3 ( 3 ) 3
+ >
+ + +
+ >
+ + + + + + +
+ >
+ + + + + + +
a b c
BDT
a bc b ca c ab
bc ca c
a bc a bc a b ca b ca b c ab
c a
a bc a bc a b ca b ca b c ab

Ta chng minh
2 2 2 2 2
2 2 2
3 1
9 ( 3 ) ( 3 )( 3 )
3 ( 3 ) 3
+ + + +
+ + + +
a
a c ab b ca b ca b
b ca b ca b c ab

Nhng do
2 2 2
3
( 3 ) (3 3 )
2
+ + + b ca b b ac
Nn ta i chng minh
2 2 2 2 3 2 2 4 2 2 2 3 4 2 2 2
3
9 ( 3 ) ( 3 )(3 3 ) 54 18 9 36 27 54 9 36 9
2
+ + + + + + + + a c ab b ca b ac a b a c b b ac a c a b b b ac a c
( ng theo gi thit a b c )

Php chng minh hon tt.

Ch . Vi , , dng th khng xy ra du =
Tuy nhin nu gi thit , , l cc s khng m sao cho khng c 2 s no ng thi bng 0 th du = vn
xy ra. l khi , bt k, 0

BI ST 53. Cho , , l 3 cnh mt tam gic. Chng minh
3 3 3 2 2 2 2 2 2
5( ) 14( ) 19( ) + + + + + + + a b c ab bc ca a b b c c a
MINHHOANG
LI GII. Ta a bt ng thc v dng SOS
3 3 3 2 2 2 2 2 2
2 2 2
5( ) 14( ) 19( ) 0
( ) (8 3 ) ( ) (8 3 ) ( ) (8 3 ) 0
+ + + + + + +
+ +
BDT a b c ab bc ca a b b c c a
a b a b b c b c c a c a

Ta c 8 3 8 3 8 3 5( ) 0 + + = + + a b b c c a a b c
V
2 2 2
(8 3 )(8 3 ) (8 3 )(8 3 ) ((8 3 )(8 3 ) 49( ) ( ) + + = + + + + a b b c b c c a c a a b ab bc ca a b c
Ch l trong tam gic ABC bt k,ta c bt ng thc quen thuc
2 2 2
2( ) + + > + + ab bc ca a b c
Nh vy
( )
2 2 2 2 2 2
49( ) ( ) 24 2( ) ( ) 0 + + + + > + + + + > ab bc ca a b c ab bc ca a b c
Nh vy (8 3 )(8 3 ) (8 3 )(8 3 ) ((8 3 )(8 3 ) 0 + + a b b c b c c a c a a b
Theo tiu chun SOS ta c dpcm

Nhn xt. Trong bi ton trn, ta s dng tiu chun sau
Xt biu thc
2 2 2
( , , ) ( ) ( ) ( ) = = + +
a b c
S f a b c S b c S c a S a b
Nu 0 + +
a b c
S S S v 0 + +
a b b c c a
S S S S S S th S 0
Mt nhn xt kh t c nhc n trong vic nh gi SOS, bi ton c th nh gi thng qua cc tiu chun
1, 2 , xt , ,...... + +
a b b c
S S S S , tuy nhin ta phi chia ra lm nhiu trng hp
Tng qut
3 3 3 2 2 2 2 2 2
( ) ( ) ( )( ) + + + + + + + + x a b c y ab bc ca x y a b b c c a
Ta s lm cht bi ton thng qua vic tm iu kin ca , , cc bn cng xem xt , l mt bi ton kh
th v...


BI ST 54. Cho cc s thc bt k , , . Chng minh rng


Bt ng Thc Su Tm V Sng To
This file was downloaded from VietNam Inequality Mathematic Olympiad Resource Page http://www.ddbdt.tk

LI GII. khng mt tnh tng qut, gi s . Khi 0, 0 nn


Ta xt 2 trng hp
Trng hp 1. Nu 2 , v ta chng minh


t , , 0. V 2 nn 2. V bt ng thc tr thnh

0
Ta c

2 3

. 3

0
Nn bt ng thc trn ng.
Trng hp 2. Nu 2 , v ta chng minh


t , , 0. V 2 nn 2, v nn 0 2. V
bt ng thc tr thnh

4 0
Ta c

10 2 22
V 0 2, nn bt ng thc trn ng.
Php chng minh hon tt.

BI ST 55. Cho cc s thc dng , , sao cho . Chng minh rng
1

1
3
LI GII. V ( )
2
2 2
1
1 1
3
x y x y + + + + , nn chng minh bt ng thc trn th ta ch cn chng minh
1 1 1
1
1 1 1 a b b c c a
+ +
+ + + + + +

Quy ng, rt gn v s dng gi thit ta c bt ng thc tng ng l
( ) ( )( )( ) ( )( )
( )
2
2 2 2
2 2
2
+ + + + + + = + + + +
= + + + + +
ab bc ca a b b c c a a b c ab bc ca abc
a b c abc a b c abc

( ) ( ) ( )
2 2
3 3 a b c ab bc ca abc a b c abc a b c + + = + + + + + +
Li t gi thit th ( )
2
3( ) 3( ) 3 a b c ab bc ca a b c a b c + + = + + + + + +
Do
( ) ( ) ( ) ( ) ( )
2 2
2 2 2 2 2 2
2 2 1 1 1
2,
3 9 3 9 3
a b c a b c a b c a b c a b c abc + + + + + + + + + +

Php chng minh hon tt. ng thc xy ra khi v ch khi 1

BI ST 56. Cho cc s thc khng m , , . Chng minh rng


5
3

LI GII. bt ng thc tng ng vi

3



2


5
3

1
2

5
3
1
1
2



7
6
2

4
3


4
3


Theo bt ng thc Schur th
Bt ng Thc Su Tm V Sng To
This file was downloaded from VietNam Inequality Mathematic Olympiad Resource Page http://www.ddbdt.tk


4
3

5
4
3


V ta ch cn chng minh
2

5
4
3


Bt ng thc ny tng ng vi

2,5 0 (lun ng)


Php chng minh hon tt.

BI ST 57. Cho cc s thc khng m , , . Chng minh rng
3


VIMF
LI GII. Chun ha cho 1 v t , . Khi

1 ,
Tng t ta cng c

1 ,

1 . Do

1 1 1
Ta c
1 1 1 2


Suy ra 1 1 1


Mt khc, 3

2. V by gi ta chng minh
3

1 6

1 4
Ta xt 3 trng hp:
Trng hp 1. Nu

th ta c

v ta ch cn chng minh
1 6 31 4
1
3

Bt ng thc ny ng v


Trng hp 2. Nu

th 1 6 0

1 4
Trng hp 3. Nu

, th ta vit li bt ng thc nh sau: 6 1

4 1
t

, v p dng bt ng thc

th ta ch cn chng minh
1

1 21 2

27

41

27

1

9

Khai trin v rt gn ta c bt ng thc tng ng l 4

0
Hay

2 1

1 0 (lun ng)
Php chng minh hon tt. ng thc xy ra khi v ch khi hoc , 0 v cc hon v tng
tng.

BI ST 58. Cho , , x y z l cc s thc khng ln hn 1 v tha mn 1 + + = x y z . Chng minh rng
2 2 2
1 1 1 27
1 1 1 10
+ +
+ + + x y z

Titu Andresscu, Gabriel Dospinescu

LI GII. Ta c nh gi
Vi mi s thc 1 t th ( )( )
2
4 3 1 3 0 t x hay l ( )
2
1 27
2
1 50

+
t
t

ng thc xy ra khi
1
3
= t
p dng vo bi, ta c ( ) ( )
2 2 2
1 1 1 27 27
6
1 1 1 50 10
+ + + + =
+ + +
x y z
x y z

Php chng minh c hon tt, ng thc xy ra khi v ch khi
1
3
= = = a b c


Bt ng Thc Su Tm V Sng To
This file was downloaded from VietNam Inequality Mathematic Olympiad Resource Page http://www.ddbdt.tk


BI ST 59. Cho cc s dng , , a b c tha mn iu kin
3 3 3
3 + + = a b c . Chng minh rng:
( )
3
3 3 3 3 3 3
4 4 4
1 1 1
| || || |
+ + + + +
| | |
+ + +
\ \ \
a b c
a b b c c a

Nguyn Vn Huyn

LI GII. T gi thit suy ra
3 3 3
0 , , 3 < < a b c . p dng bt ng thc AM GM ta c

( ) ( )
3 3 3
3 3 3
4 4 4
3 2 . 3 4 1 2
3 3 3
+ = + +

a a a
a a a

Dn n
( )( )( )
3 3 3
3 3 3 3 3 3
4 4 4
1 1 1 2 2 2
| || || |
+ + + + + +
| | |
+ + +
\ \ \
a b c
a b b c c a

Vy ta cn chng minh
( )( )( ) ( )
3
3 3 3
2 2 2 + + + + + a b c a b c
Php chng minh hon tt. ng thc xy ra khi 1 = = = a b c


BI ST 60. Cho cc s dng , , a b c tha mn iu kin ( )
1 1 1
4
| |
+ + =
|
\
a b c
a b c
. Tm gi tr nh nht ca
biu thc
( )
4 4 4
4 4 4
1 1 1 | |
= + + + +
|
\
P a b c
a b c

Vacsile Cirtoaje
LI GII. Khng mt tnh tng qut, gi s a b . t 1 =
a
u
b
. V
( ) ( ) ( )
( ) ( ) ( )
2 2
1 1 1 1 1 1 1 1
4 1
1 1 1 1 1 1 1
2 1 1 2 1
| | | | | | | |
= + + = + + + + + +
| | | |
\ \ \ \
| | | |
| | | | | |
+ + + + + = + + = + + | |
| | |
| |
\ \ \
\ \
a b c a b a b c
a b c a b c a b
a b a b a b u
a b a b a b u

Nn
2
1 1
2 1 4 7
| |
+ + +
|
|
\
u u
u u

p dng bt ng thc Cauchy-Schwarz ta c

( ) ( ) ( )
2
2
2
4 4 4 4 4 2 2
4 4 4 4 4 2
1 1 1 1 1 1
1 1 7 1 2304
| |
| | | | | |
+ + + + + + + = + + = |
| | |
|
\ \ \
\
a b c a b u
a b c a b u

Vy
min
2304 = P

t c khi v ch khi
2
= ab c v 7 + =
a b
b a
iu ny tng ng vi
3 5 3 5
,
2 2
+
= =
a b
c c


BI ST 61. Cho cc s thc khng m , , a b c sao cho khng c hai s no cng bng 0. Chng minh rng
2 2 2
2 2 2 2 2 2
3
a bc b ca c ab
b bc c c ca a a ab b
+ + +
+ +
+ + +

Ng c Lc , MIC 2009
LI GII. Khng mt tnh tng qut, gi s { } , , c min a b c = . Khi ta cc nh gi
2 2
, a bc a +
2 2
, b ca b +
2
c ca ab +
v ( )
2 2 2 2
, b bc c b c c b b + = + ( )
2 2 2 2
c ca a a c c a a + = +
Khi
2 2 2 2 2
2 2 2 2 2 2 2 2 2 2
a bc b ca c ab a b ab
b bc c c ca a a ab b b a a ab b
+ + +
+ + + +
+ + + +

p dng bt ng thc AM GM , ta c
Bt ng Thc Su Tm V Sng To
This file was downloaded from VietNam Inequality Mathematic Olympiad Resource Page http://www.ddbdt.tk

2 2
2 2 2 2 2 2
2 2
1 1
a b ab a b ab
b a a ab b b a a ab b
| | | |
+ + + + +
| |
+ +
\ \

2 2
2 2
1 2 2 1 3
a ab b ab a b
ab a ab b b a
| | + | | | |
= + = + + + =
| | |
+
\ \
\

Bi ton c chng xong. ng thc xy ra khi trong ba s c hai s bng nhau s cn li bng 0

BI ST 62. Cho cc s dng , , a b c tha mn iu kin 1 abc = . Chng minh rng
( )
2 3 3 3
9
8 8 8
a b c
a b c
a b c
+ +
+ + +
+ +

Nguyn Vn Huyn
LI GII. p dng bt ng thc AM GM ta c
( )( )( ) ( ) ( )
3 3 3
3 3 3 2
6 6
3 3
8 8 8
8 8 8 2 2 4
+ + =
+ + +
+ + + + +

a b c
a b c
a b c a a a

Ta cn phi chng minh
( ) ( )
( )
( ) ( ) ( )
4
2
3
2
2
6
3 9
9 2 2 4
2 2 4
+ + + +
+ +
+ +


a b c a a a
a b c
a a a

Theo bt ng thc AM GM ta c 3 a b c + + v 3 ab bc ca + + , v th
( )( )( )
( )
( )( )( )
( )
( ) ( )
2
3
2 2 2
2 2 2
3
2
2 2 2 2 2 2
6
2 2 2
3 3
2 12
2 4 2 4 2 4
3
2 6
3 3 3
+ +
+ + +
+ + + + +
+ + + + +
+ + +
+ + + + + + + + + +
=
a b c
a b c
a b c a b c
a b c a b c
a a b b c c
a b c ab bc ca a b c a b c

T hai bt ng thc ny ta suy ra iu phi chng minh. ng thc xy ra khi v ch khi 1 a b c = = =

BI ST 63. Cho cc s thc , a b tha mn iu kin 2 2 a < < , 3 3 b < < v 1 ab = . Chng ming rng

2 2
4 9 12
4 9 5 a b
+


LI GII. T gi thit, p dng bt ng thc AM GM ta c
( )( )
( )
2 2
2 2 2 2 2 2
2 2
2 2
4 9 12 12
4 9
9 4 36 4 9
12 12 12
5
37 9 4
37 2 36
+ =

+
=
+

a b
a b a b a b
a b
a b

ng thc xy ra khi
6 6
,
3 2
a b = = hoc
6 6
,
2 3
a b = =


BI ST 64. Cho cc s dng , , a b c tha mn iu kin 1 a b c + + = . Chng minh rng
( )( )( ) ( )
2
2 2 2 2 2 2 2 2 2 2 2 2
8 a b b c c a a b b c c a + + + + +
LI GII. t
1 1 1
, , m n p
a b c
= = = th
1 1 1
1
m n p
+ + = , ta cn phi chng minh
( )( )( ) ( )
( )( )( ) ( )
2
2 2 2 2 2 2 2 2 2
2
2
2 2 2 2 2 2 2 2 2
8
1 1 1
8
+ + + + +
| |
+ + + + + + +
|
\
m n n p p m m n p
m n n p p m m n p
m n p

Li t
2 2 2 2
2 , 2 m n x n p y + = + = v
2 2
2 p m z + = , bt ng thc tng ng vi
Bt ng Thc Su Tm V Sng To
This file was downloaded from VietNam Inequality Mathematic Olympiad Resource Page http://www.ddbdt.tk

( )
1 1 1 1
x y z
x y z y z x z x y xyz
+ + + +
+ + +

Theo bt ng thc Schur ta c
( ) ( ) ( ) ( )
3 3 3
xyz x y z x y z x y z x y z x y z + + + + + + +
Mt khc theo bt ng thc Holder th
( ) ( ) ( )
( )
3 3 3
3
3 3 3
2 2 2 2
1 1 1 1 1 1
x y z x y z x y z x y z
x y z x x x
y z x y z x y z x y z x y z x y z x
+ + + + +
+ +
= + +
| | | | | | | |
+ +
| | | |
| | | |
+ + + + + +
\ \ \ \

T hai bt ng thc ny ta suy ra iu phi chng minh, ng thc xy ra khi
1
3
a b c = = =


BI ST 65. Cho cc s dng , a b tha mn iu kin 1 a b + . Chng minh rng
2
1 1 2
1 1 1
a b a b
| || | | |

| | |
+
\ \ \

LI GII. Bt ng thc cn chng minh tng ng vi

( ) ( )
( ) ( )
2
2 2
1 1 1 4 4 1 4 1 1 4
1 0 +
+ +
+ +
a b a b
ab a b a b ab a b a b
a b a b

Bi ton c chng minh xong. ng thc xy ra khi
1
2
a b = =

BI ST 66. Cho , , a b c l cc s thc dng. Chng minh rng
( ) ( ) ( )
3 3 3
3 3 3
3 3 3
1
a b c
a b c b c a c a b
+ +
+ + + + + +


LI GII 1. Ta cn chng minh
( ) ( )
( )
( ) ( ) ( )
3 2 3 4
2
3
2 2 2 2 2
3 3 2 2 2 2
3 3
2 2 2
2 + + + +
+ +
+ + + +
+ +
a a a a
a b c b c a b c
a b c
a b c a b c
a b c

Thao bt ng thc AM GM ta c
( ) ( ) ( ) ( )
3
2 6
2 2 2 2
2 8 b c b c b c b c + + + +
Mt khc theo bt ng thc AM GM th
( ) ( ) ( ) ( )
2 3
3
2 2 2 2 2 2 2 2
2 2 2 a b c b c a b c a b c + + + + +
T dn n
( ) ( ) ( )
3 3 3 2 2 2
3 3 3 2 2 2
3 3 3
1
a b c a b c
a b c
a b c b c a c a b
+ +
+ + =
+ +
+ + + + + +

Php chng minh hon tt, ng thc xy ra khi a b c = =

LI GII 2. Ch rng Vi mi s thc dng t, ta lun c bt ng thc
2
2 3
1
2 1
2
t t
| |
+ +
|
\

Tht vy ta c bt ng thc tng ng vi
( ) ( ) ( )
2 2
2 3 2
2 4 1 2 0 t t t t + +
ng thc xy ra khi 2 t =
Tr li bi ton, s dng b v bt ng thc Cauchy Schwarz ta thu c
Bt ng Thc Su Tm V Sng To
This file was downloaded from VietNam Inequality Mathematic Olympiad Resource Page http://www.ddbdt.tk

( )
3 2
3 2 2 2 2 2 2
3 3
2
1 1 1
1
1
1
1
2
= =
+ + +
+ + + | |
+ | |
+
+
+ |
|
\
\
a a
b c a b c
a b c b c
b c
a
a
a

V th dn n
( ) ( ) ( )
3 3 3 2 2 2
3 3 3 2 2 2
3 3 3
1
a b c a b c
a b c
a b c b c a c a b
+ +
+ + =
+ +
+ + + + + +

Bi ton c chng minh xong

BI ST 67. Cho , , a b c l cc s khng m tha mn iu kin 1 a b c + + = . Chng minh rng
2 2 2 2 2 2
4
1 1 1 5
a b c
b c c a a b
+ +
+ + + + + +

Phm Kim Hng

LI GII. Theo bt ng thc Cauchy Schwarz ta c
( ) ( )
( )
2
2 2
2 2
1
1
a
a b c a b c
b c
| |
+ + + +
|
+ +
\


t ( ) ( ) ( ) ( ) , , S a b c ab a b bc b c ca c a = + + + + +
Ta cn phi chng minh ( )
1
, ,
4
S a b c . Tht vy khng mt tnh tng qut, gi s c b a . Khi
( ) ( ) ( ) , , , , 0 2 0 S a b c S a b c ab a b c + = +
Nn ( ) ( ) , , , , 0 S a b c S a b c +
Mt khc ta li c
( ) ( )( ) ( ) ( )
2 1 1
, , 0
4 4
S a b c c a b a b c c a b a b c + = + + + = + + + =
T suy ra iu phi chng minh, ng thc xy ra khi trong ba s c mt s bng 0, hai s cn li bng
1 2

BI ST 68. Cho , , a b c l cc s thc dng. Chng minh rng
3 3 3
8
2 2 2
a b c
b c c a a b
| || || |
+ + +
| | |
+ + +
\ \ \

Trn Tun Anh
LI GII. Theo bt dng thc AM GM th
( ) ( ) ( ) ( ) ( )( ) ( )
( )
( )( )( )
4
4
3 2 2 2 4 + + = + + + + + + + + + + + + b c a a b a c b c a b a c b c a b a c b c
T dn n
( )( )( ) ( )
( )( )( )
4
4
64
3 3 3
8
2 2 2 8
+ + +
| || || |
+ + + =
| | |
+ + + + + +
\ \ \
a b b c c a
a b c
b c c a a b a b b c c a

ng thc xy ra khi a b c = =

BI ST 69. Cho cc s thc dng , , sao cho

1

10
Tm gi tr nh nht v gi tr ln nht ca


Vasile
LI GII. gii quyt bi ton ny ta cn 2 b sau:
B 1.
Bt ng Thc Su Tm V Sng To
This file was downloaded from VietNam Inequality Mathematic Olympiad Resource Page http://www.ddbdt.tk


1

2
B 2.

1


Chng minh b 1. t

. Khi

3,

2 2 3
Thay vo bt ng thc ban u ta c bt ng thc tng ng l
3 5 2

4 4 6 2 2

4 4 4 2

0
Bt ng thc ny hin nhin ng. Php chng minh hon tt. ng thc xy ra khi v ch khi xy ra khi

, hoc 2.
Chng minh b 2. p dng bt ng thc Bunhiacopxki ta c
( )
( ) ( ) ( )
( )
2 2 2
2 2 2
2 2 2 2 2 2
2 2 2 2 2 2
1 1 1 1 1 1 1 1 1
( )( ) 2 2 2 2 2 2.
1 1 1 1 1 1 1 1 1
2
1 1 1
2 ( )(
a b c a b c ab bc ca
a b c a b c ab bc ca
a b c ab bc ca a b c
a b c ab bc ca a b c
a b c a b c
a b c


+ + + + = + + + + + + + + + +





+ + + + + + + + + = + + + +





+ + + + + + +



( )
2
2 2 2
2 2 2
1 1 1 1 1 1
) 1 1 a b c
a b c a b c




+ + + + + + +







Suy ra iu phi chng minh.
T 2 b trn ta d dng suy ra gi tr nh nht ca l

v gi tr ln nht ca l 10 1





















Gii Ton Nhu Th No?
This file was downloaded from VietNam Inequality Mathematic Olympiad Resource Page http://www.ddbdt.tk
GII TON NH TH NO?
N BI TON TNG QUT
VIMF VIMF VIMF VIMF

LTG. Trong k thi Ton Quc t (IMO) nm 2008 c cu 2 l mt bi ton bt ng thc kh hay. Bi ton
ny c rt nhiu cch gii nhng t tng chnh vn l a v bnh phng ca mt tng

0. Trong bi
ny, ti xin gii thiu vi cc bn mt li gii nh th v mt li gii khc na cng p mt bng bt ng
thc , v phn cui s l bi ton tng qut. Chng ta cng bt u vi

BI TON *. Cho , , l cc s thc khc 1 v tho mn 1. Chng minh rng
2 2 2
2 2 2
1
( 1) ( 1) ( 1)
x y z
x y z
+ +


LI GII 1 (VIMF).
Do 1 nn t ,
a
x
b
= , ; , , w
b c a b b c c a
y z u v
c a a b b c c a
+ + +
= = = = =


Ta c
2 2 2
2 2 2
2 2 2
1 1
( 1) ( 1) ( 1)
x y z a b c
x y z a b b c c a
| | | | | |
+ + + +
| | |

\ \ \

2 2 2
1
1 1 1
4
a b b c c a
a b b c c a
(
+ + + | | | | | |
+ + + + +
(
| | |

\ \ \
(

2 2 2
1 2( ) 1 u v w u v w + + + + +
Li c ( 1)( 1)( +1)=( 1)( 1)( 1) = 1 u v w u v w uv vw wu + + + +
Do
2 2 2 2
2( ) 1 ( ) 2( ) 2( ) 1 u v w u v w u v w u v w uv vw wu + + + + + + + + + + + +
2 2
( ) 2( ) 2 1 0 ( 1) 0 u v w u v w u v w + + + + + + + + + (lun ng).
Vy bt ng thc c chng minh xong.

LI GII 2 (V QUC B CN).
Cng nh trn ta phi chng minh
2 2 2
1
a b c
a b b c c a
| | | | | |
+ +
| | |

\ \ \

M theo bt ng thc th
( ) ( ) ( )
2
2 2
2 2 2
2
( ) ( ) ( )
( )
a
a b a c a a c a ac
a b
(
(
( =
(






Li c
( ) ( ) ( )
2
2 2
( ) ( ) ( )( ) 2 ( ( )( )( ) a b c a a b c a a b c a b c b c ( =


( ) ( ) ( )
2 2 2
2
( )( ) 2( )( )( )( ) ( )( ) a b c a a b b c c a a b b c c a a b c a a ac ( ( = + + + = =



Do
2 2 2
1
a b c
a b b c c a
| | | | | |
+ +
| | |

\ \ \

Vy bt ng thc c chng minh xong.

Trn y l hai li gii p mt cho bi ton *. Tuy nhin cha dng li, ta mun tm mt bi ton tng qut
hn. V nh mong i, chng ta c

BI TON **. Cho , , l cc s thc khc 1 v tho mn 1, v s thc bt k . Chng minh rng ta
lun c bt ng thc
2
2 2
1
1 1 1
x m y m z m
x y z
| | + + + | | | |
+ +
| | |

\ \
\

n bi ton tng qut
This file was downloaded from VietNam Inequality Mathematic Olympiad Resource Page http://www.ddbdt.tk
LI GII (VIMF).
t , ,
1 1 1
x m y m z m
a b c
x y z
+ + +
= = =

th , ,
1 1 1
a m b m c m
x y z
a b c
+ + +
= = =


Do 1 nn . . 1
1 1 1
a m b m c m
a b c
+ + +
=

( )
2 3
( 1)( ) 1 ( ) 1 0(1) m ab bc ca m a b c m + + + + + + + + =
+ Nu 1 m = th BT hin nhin ng
+ Nu 1 m th t (1) suy ra ( )
2
( ) 1 ( ) 1 ab bc ca m a b c m m + + = + + + +

Ta c
( )
2 2 2 2 2 2
( ) 2( ) ( ) 2 1 ( ) 2 2 2 a b c a b c ab bc ca a b c m a b c m m + + = + + + + = + + + + + + +
2 2 2
( 1) 1 1 1 a b c m m m = + + + + + +
Vy bt ng thc c chng minh xong.

NHN XT. R rng bi ton ** tng qut cho bi ton * (vi trng hp ring 0, v iu kin ng thc
xy ra trong bi ton ** cng l 0). Vic nh gi
2
1 1 VT m + l mt iu rt hay. Cc bn hy gii bi
ton ** theo hng ca li gii 1 ca bi ton *, xem nh bi tp.

























K thut phn tch bnh phuong cho bt dng thc hon vj
This file was downloaded from the VietNam Inequality Mathematic Olympiad Resource Page http://ddbdt.tk
K THUT PHN TCH BNH PHNG
CHO BT NG THC HON V
(SUM OF CYCLIC)
VIMF VIMF VIMF VIMF
LTG. Bt ng thc hon v l nhng bi ton rt p bi s pht biu n gin nh nhng ca chng.
Tuy nhin, vic gii chng th ngc li, vic tm mt li gii cho chng v cng vt v v kh khn. V i vi
nhng bi ton c 2 ng thc tr ln th mi vic li cng tr nn kh khn hn. Sau mt thi gian hc hi
kinh nghim v tm ti, ti tm c mt k thut nh gi cho nhng bt ng thc hon v n gin. Do
kh ca cc bi ton nn i khi mt s li gii c i cht di, nhng b li l ta c th lm cht cho mt
s bi ton (y l mt iu bt ng m k thut ny mang li). Cng xin ni thm rng: bt ng thc hin i
rt phong ph vi rt nhiu bi tp. Tuy nhin vi bt ng thc hon v vng quanh th khc, n rt t nn c
th coi l nhng bi ton him. Vic to ra mt bt ng thc ng l kh m bt ng thc hay th
cng kh hn, nn i vi bt ng thc hon v th iu li cng kh thc hin. V th k thut ny ch l
mt cng c nh nhng li v cng hu ch cc bn c thm mt hng gii quyt cc bi ton bt ng
thc hon v vng quanh ba bin. Chng ti rt hoan nghnh nhng s ng gp, tm ti sng to thm cho k
thut ny t pha cc bn. Mi thc mc ng gp kin xin vui long lien h theo a ch email:
vif.vimf@gmail.com

I/ C s ca k thut.
S tht bt ng nu ti ni vi cc bn rng c s ca k thut ny l phng php phn tch bnh phng
. . : l a bt ng thc thun nht ba bin , , v dng
2 2 2
( ) ( ) ( ) 0
a b c
S b c S c a S a b + +
i vi bt ng thc i xng ba bin th vic quy v dng chnh tc . . nh trn l n gin gip ta
d dng gii quyt bi ton. Tuy nhin, i vi bt ng thc hon v vng quanh th cch quy trn i khi
khng thch hp v to ra cc h s ; ;
a b c
S S S rt cng knh v kh x l. Trong trng hp c mt cch khc
l quy v dng
2 2 2
( ) ( ) ( ) ( )( )( )
a b c
S b c S c a S a b S a b b c c a + +

Ti tm gi n l phn tch bnh phng hon v . .
Cch quy trn c g li?.
- Th nht. i vi cc dng hon v vng quanh th n t nhin v n gin hn cch a v . . chnh
thng.
- Th hai. i vi bt ng thc hon v th ta ch cn xt mt trong 2 kh nng sau:
+ Mt trong ba s l ln nht (gi s l , , ), th ta xt 2 trng hp c th xy ra l a b c v
a c b .
+ Mt trong ba gia 2 s kia (gi s l ), th ta xt 2 trng hp c th xy ra l a b c v c b a .
V vy, nu v tri v khng m th ta ch xt trng hp c b a m b qua trng hp a b c .
Cui cng cng xin lu lun l i vi cc bi ton sau y chng ta cng ch xt trng hp c b a
(khi
2 2 2 2 2 2
( )( )( ) 0 a b b c c a a b b c c a ab bc ca + + + + ), cn vi trng hp a b c th
( )( )( ) 0 S a b b c c a , v ta ch phi lm theo phng php truyn thng . . l chng minh bt ng thc
2 2 2
( ) ( ) ( ) 0
a b c
S b c S c a S a b + +

III/ Phn tch c s.
1.
2 2 2 2 2 2
( )( )( ) ab bc ca a b b c c a a b b c c a + + =

2.
( )
2 2 2 2 2 2 2 2 2 2 2 2 2 2 2
1
3 6
2
ab bc ca abc ab bc ca a b b c c a ab bc ca a b b c c a abc + + = + + + + + + + +
( )
2 2 2
1
( )( )( ) ( ) ( ) ( )
2
a b b c c a a b c b c a c a b = + + +
3.
( )( )( )
( )( )( )
a b b c c a a b b c c a
a b b c c a a b b c c a

+ + =
+ + + + + +

4.
1
2
a b c a b a b b c b c c a c a
a b b c c a a b b c c a
+ + + + + + | |
+ + = + +
|
+ + + + + +
\

K thut phn tch bnh phuong cho bt dng thc hon vj
This file was downloaded from the VietNam Inequality Mathematic Olympiad Resource Page http://ddbdt.tk
1 1 ( )( )( )
3 3
2 2 ( )( )( )
a b b c c a a b b c c a
a b b c c a a b b c c a
| | | |
= + + + =
| |
+ + + + + +
\
\

5.
3 3 3 3 3 3
( )( )( )( ) ab bc ca a b b c c a a b c a b b c c a + + = + +
Bn cnh cc phn tch s s ny cn rt nhiu cch phn tch khc m cc bn c th t tm thy trong qu
trnh gii ton.

IV/ Xy dng nh l.
Chng ta s xy dng nh l, a ra cc tiu chun t cch phn tch
2 2 2
( ) ( ) ( ) ( )( )( )
a b c
S b c S c a S a b S a b b c c a + +
Ch y ta ch xt n trng hp c b a
Nh th th ( )( ) 0 a b b c nn
2 2 2 2 2 2
( ) ( ) ( ) ( ) ( ) ( )
a b c a b c
S b c S c a S a b S b c S a b b c S a b + + = + + +

( ) ( ) ( )( )
2 2
( ) ( ) 2 ( )( ) 2 ( )( ) 2 ( )( )
b c a b b a b b c b
AM GM
S S a b S S b c S a b b c S S S S a b b c S a b b c

= + + + + + + +

Do bt ng thc s c chng minh nu ta chng minh c
( )( ) 2 2 0
a b b c b
S S S S S S + + +
Xy dng tng t nh trn bng cch tch ( ) a b c b c a = v ( ) b c b a c a = , ta cng c thm 2
tiu chun na.
Tip tc xy dng, ta c
2 2
( ) ( ) 4 ( )( )
b b b
AM GM
S c a S c b b a S c b b a

= +

v
2 2
( ) ( ) 2 . .( )( )
a c a c
S b c S a b S S b a c b +
Do bt ng thc s c chng minh nu ta chng minh c 4 2 . ( )
b a c
S S S S c a +

Ngoi ra ta cn c
2 2 2 2 2 2
3
( ) ( ) ( ) 3 ( ) ( ) ( )
a b c a b c
S b c S c a S a b S S S b c c a a b + +
Nn bt ng thc s c chng minh nu ta chng minh c 27


H thng cc kt qu trn ta c cc tiu chun sau
1. ( )( ) 0, 0, 2 2 ( ) 0
a b b c a b b c b
S S S S S S S S S S c a + + + + +
2. ( )( ) 0, 0, 2 2 ( ) 0
a b a c a b a c a
S S S S S S S S S S c b + + + +
3. ( )( ) 0, 0, 2 2 ( ) 0
c a c b c a c b c
S S S S S S S S S S b a + + + +
4. 0, 0, 2 . 4 ( ) 0
a c a c b
S S S S S S c a +
5. 0, 0, 0, 2 ( ) 0
a b c b c
S S S S S S c b
6. 0, 0, 0, 2 ( ) 0
a b c a b
S S S S S S b a
7.
3
0, 0, 0, 27 ( )( )( ) 0
a b c a b c
S S S S S S S a b b c c a
Cc tiu chun trn rt tin x l nhng bi ton c cc h s ; ;
a b c
S S S cng knh (c bit l tiu
chun 1 rt mnh). Tuy nhin nu ta gp nhng bi ton rt cht n ni khng th p dng c tiu ch no th
c mt cch khc l t c a x y = + + v b a x = + ( , 0) x y . Cch lm ny gip ta c th loi i a mt cch
nhanh chng nh cch phn tch trn (bi ; c a x y b a x = + = ). Hn na ta li cn c th lm cht cho bt
ng thc nh cc bin cn tha li. Cc bi ton p dng sau lm sng t thm cho iu ny.
Ngoi ra, ta cn c th chia nh nhiu trng hp na trong c b a d dng gii quyt bi ton.

V/ p dng vo gii ton.
C l mi k thut cng xut pht t mt bi ton no . V ti cng vy, ti xin bt u bng mt bi ton
khi u cho k thut ny mt cch tnh c

BI TON 1. Cho cc s thc khng m , , . Chng minh bt ng thc
2 2 2
3 3 3
2 2 2
3 . ( ) ( ) ( )
a b b c c a
a b c abc ab a b bc b c ca c a
ab bc ca
+ +
+ + + + + + + +
+ +

LI GII (VIMF).
Nu a b c th
2 2 2 2 2 2
a b b c c a ab bc ca + + + + , nn theo bt ng thc Schur th
K thut phn tch bnh phuong cho bt dng thc hon vj
This file was downloaded from the VietNam Inequality Mathematic Olympiad Resource Page http://ddbdt.tk
2 2 2
3 3 3 3 3 3
2 2 2
3 . 3 ( ) ( ) ( )
a b b c c a
a b c abc a b c abc ab a b bc b c ca c a
ab bc ca
+ +
+ + + + + + + + + + +
+ +

Nu c b a th bt ng thc c vit li nh sau
2 2 2
3 3 3
2 2 2
3 3 . 1 ( ) ( ) ( ) 6
a b b c c a
a b c abc abc ab a b bc b c ca c a abc
ab bc ca
| | + +
+ + + + + + + +
|
+ +
\

( )
2 2 2 2 2 2
2 2 2
1 3 ( )( )( )
( ) ( ) ( ) ( ) ( ) ( ) ( )
2
abc a b b c c a
a b c a b b c c a a b c b c a c a b
ab bc ca

+ + + + + +
+ +
2 2 2
2 2 2
1 1 1 3 ( )( )( )
( )( ) ( )( ) ( )( )
2 2 2
abc a b b c c a
a b c a b b c a b c c a b c a
ab bc ca

+ + + + +
+ +

Theo tiu chun 1 th ta ch cn chng minh
2 2 2
3 ( )
2 0
abc c a
ac c a b
ab bc ca

+ +
+ +

Quy ng, rt gn v nhm cc s hng li vi nhau ta c bt ng thc tng ng l
( )
2 2 2 2 2 2 2 3 2 2 2
2 ( ) ( ) 2 2 2 0 bc ac a ab c b bc c b a c a b a c ab ac ca ac a bc + + + + + + + +
Bt ng thc trn ng do c b a
Vy ta c ta c iu phi chng minh. ng thc xy ra khi v ch khi ba bin bng nhua hoc mt trong 3 bin
bng 0 v 2 bin cn li bng nhau.

BI TON 2. Chng minh rng vi mi s dng , , th
3 3 3
2 2 2 2 2 2
2 2 2 3
a b c a b c
a b b c c a
+ +
+ +
+ + +

NGUYN TRNG TH
LI GII.
cho gn ta k hiu l tng cyclic (mi tng gm 3 s hng).
Bt ng thc trn tng ng vi
3 2
3 2 2 2 2 2
2 2
0 ( )(2 )(2 ) 0
2
a ab
a ab b c c a
a b

+ +
+

3 2 2 3 4 5 2 5 2 4 2 2 4 3 4
3 2 2 4 2 2 a b c a c a b a c ab c ab c a b + + + + +

( ) ( ) ( )
5 2 3 2 2 4 2 5 2 3 2 2 4 2 3 4 3 4
2 2 2 2 a b a b c a b c a c a b c a bc a b a c + + +

( )
3 2 2 3 2 2 2 2 2
2 ( ) ( ) 2( )( )( ) a b a c a c a b a b b c c a a b a bc + +


By gi gi s ax{a,b,c} a m = . Nu th bt ng thc trn ng nn ta ch phi xt khi a c b . Ta s
chng minh
( )
3 2 2 2 3 2 3 2 2 3 2 3 3 2
2 ( ) 2 ( ) ( ) 2( )( ) a b a c a c c b a c a b a c c b a c a bc a b + + + +
Xt 2 trng hp
Trng hp 1. Xt khi c b a c , ta c
2 2 3 2 2 3 2 3 2
( ) ( ) 2 ( )( ) 4 ( )( ) b a c a c a b a b a c c b a c a c c b + +
V
( )
3
3 2 3 2 2 2
2 a b a c a c bc b + + + nn suy ra iu phi chng minh.
Trng hp 2. Xt khi c b a c > , tng t nh trn ta c
2 3 2 3 2 2 2 3 3 2
2 ( ) ( ) 2 ( )( ) 4 ( )( ) a c c b a c a b a c a c c b a c a c c b + +
V
( ) ( )
2 3 3 2 3 2 2 2 2 3 3 2 3 2
2 ( ) ( ) 0 a c a c a c bc b a c b a bc a bc b a b c b a bc c a + + + > + + = +

Nn ta cng c iu phi chng minh.
Vy, ta c iu phi chng minh. ng thc xy ra khi v ch khi


BI TON 3. Cho cc s thc khng m , , . Chng minh
2 2 2
2 2 2
4 4 4
7
a b c ab bc ca abc
a b b c c a a b b c c a abc
+ + +
+ + +
+ + + + + +

VIMF
LI GII (VIMF). Theo cch phn tch c s 4 th bt ng thc c vit li thnh
K thut phn tch bnh phuong cho bt dng thc hon vj
This file was downloaded from the VietNam Inequality Mathematic Olympiad Resource Page http://ddbdt.tk
2 2 2
2 2 2
( )( )( )
2 3 1 6
( )( )( )
a b b c c a ab bc ca abc
a b b c c a a b b c c a abc
| | | | + + +
+
| |
+ + + + + +
\ \

2 2 2
( )( )( ) 2( )( )( )
0
( )( )( )
a b b c c a a b b c c a
a b b c c a abc a b b c c a


+ + + + + +
( )
( )
2 2 2
2 2 2
( )( )( ) ( )( )( ) 2
0
( )( )( )
a b b c c a a b b c c a a b b c c a abc
a b b c c a abc a b b c c a
(
+ + + + + +


+ + + + + +
[ ]
( )
2
2 2 2
( )( )( )
0
( )( )( )
a b b c c a
a b b c c a abc a b b c c a


+ + + + + +

Bt ng thc trn hin nhin ng.
Vy ta c iu phi chng minh. ng thc xy ra khi v ch khi .

BI TON 4. Cho cc s thc dng , , sao cho 1. Chng minh rng
2 2 2
3 3 3
3
( 1) ( 1) ( 1)
a b c
a b c
+ + +
+ +
+ + +

UK TST 2005
LI GII (VIMF). Do 1 nn t , ,
y z x
a b c
x y z
= = =
Bt ng thc trn c vit li nh sau
2 2
2
2 2 2
2 2 2
3 3 3 3
3 1 1 1
( ) ( ) ( ) 4
x xy y yz z zx x y y z z x
x y y z z x x y y z z x
(
| | | | + + + | |
+ + + + + + + + (
| | |
+ + + + + +
\ ( \ \


2 2 2 2 2 2
2 2 2
1 ( ) ( ) ( ) ( ) ( ) ( )
3
4 ( ) ( ) ( )
x y x y y z y z z x z x
x y y z z x
( + + +
+ + +
(
+ + +


2 2
2
3( )( )( )
3
( )( )( )
x y y z z x x y y z z x x y y z z x
x y y z z x x y y z z x x y y z z x
| | | | | | | |
+ + + + =
| | | |
+ + + + + + + + +
\
\ \ \

Nu ( )( )( ) 0 x y y z z x th bt trn hin nhin ng.
Nu ( )( )( ) 0 x y y z z x th
2 2 2
2
3
( )( )( )
3
( )( )( )
AM GM
x y y z z x x y y z z x
x y y z z x x y y z z x

| | | | | | | |
+ +
| | | |
+ + + + + +
\
\ \ \

nn ta ch cn chng minh
( )
2 2 2
( )( )( )
1 2 0
( )( )( )
x y y z z x
x y y z z x
x y y z z x

+ +
+ + +
(lun ng).
Vy ta c iu phi chng minh. ng thc xy ra khi v ch khi 1.

C th ni hai bi ton trn khng cn phi s dng n mt tiu chun no c. nhng mt khc li cho
thy c ci li khi phn tch v ( )( )( ) S a b b c c a .
V by gi chng ta s thc hin lm cht mt s bt ng thc bng cc bin cn tha

BI TON 5. Cho cc s thc khng m , , . Chng minh bt ng thc
( )
3 2 2 2
4( ) 27 a b c ab bc ca abc + + + + +

Vasile Cirtoaje

LI GII (VIMF).
Nu a b c th
2 2 2 2 2 2
ab bc ca abc ab bc ca abc + + + + + + nn
( ) ( )
2 2 2 2 2 2 2 2 2
27
27
2
ab bc ca abc ab bc ca ab bc ca abc + + + + + + + + +
Do ta ch cn chng minh
( )
2 2 2 2 2 2 3
27 8( ) ab bc ca ab bc ca abc a b c + + + + + + + +
( ) ( )
3 3 3 2 2 2 2 2 2
8 3 6 a b c ab bc ca ab bc ca abc + + + + + + + +
Bt ng thc trn hin nhin ng theo bt ng thc
Nu c b a th ta vit bt ng thc li nh sau
K thut phn tch bnh phuong cho bt dng thc hon vj
This file was downloaded from the VietNam Inequality Mathematic Olympiad Resource Page http://ddbdt.tk
3 2 2
4 12 15 3 0 a a b ab abc +


( ) ( )
3 2 2
3 27
4 3 ( ( )) 6 0
2 2
a abc ab a b abc a b ab ( + +



( )
2 2
3 27
2( ) ( ) ( ) ( )( )( )
2 2
a b c a b a b c c b b a c a ( + +


2 2 2
(4 4 )( ) (4 4 )( ) (4 4 )( ) 27( )( )( ) b c a b c c a b c a a b c a b c b b a c a + + + + + + + +
2 2
(5 5 8 )( ) (8 5 5 )( ) 2(4 4 )( )( ) 27( )( )( ) a b c c b a b c b a a b c c b b a c b b a c a + + + + + + + +
t , c a x y b a x = + + = + . Bt ng thc c vit li nh sau
2 2
(18 8 13 ) (18 5 10 ) 2(9 5 4 ) 27 ( ) y a y x x a y x a x y xy xy x y + + + + + + + + +
Loi th ta ch cn chng minh
2 2 3 3 2 2
(8 13 ) (5 10 ) 2(5 4 ) 27 ( ) 5 4 6 3 y y x x y x x y xy xy x y x y x y xy + + + + + + + +
Ta c
( )
3 3 3 2 3 3 3 2
2 6 ; 6
AM GM AM GM
x x y x y x y y xy

+ + + +
Do ta c iu phi chng minh
By gi nh ni phn Xy dng nh l, ta s lm cht bt ng thc nh cc bin cn tha
( )
2 2 2 2
1 3 27
.18 9 . .( ) ( )
2 4 2
AM GM
a x y xy a x y a c a

+ + + =
Nh vy l ta c bt ng thc cht hn l: vi min{ , , } k a b c = v { , , } t max a b c = th
( )
3 2 2 2 2
27
4( ) 27 ( )
4
a b c ab bc ca abc k t k + + + + + + ,
Cc bn ng lo cch lm cht ny ch ng trong mt trng hp m ta ang xt, bi trong trng hp
ngc li th sau khi nh gi ( )( )( ) 0 ( )( )( ) a b b c c a a b b c c a , cng vic cn li ch
l vn tng t.

BI TON 6. Cho cc s thc khng m , , . Chng minh bt ng thc
3 3 3 2 2 2 2 2 2
2( ) 3( ) a b c a b b c c a ab bc ca + + + + + + +
LI GII (VIMF).
Nu a b c th
2 2 2 2 2 2
2( ) 2( ) a b b c c a ab bc ca + + + + v
3 3 3 2 2 2
a b c ab bc ca + + + + nn bt ng thc
hin nhin ng.
Nu c b a th bt ng thc c vit li nh sau
2 2 2
( )( ) ( )( ) ( )( ) 5( )( )( ) a b a b b c a b c a a b a b b c c a + + + + +
2 2
(2 )( ) (2 )( ) ( )( )(3 7 ) a b c b a c a b c b b a c b c a + + + + +
t , c a x y b a x = + + = + . Bt ng thc c vit li nh sau
2 2
(4 2 ) (4 3 2 ) ( 4 3 3 ) x a x y y a x y xy a x y + + + + + + +
Loi i th ta ch cn chng minh
2 2 3 3 2
(2 ) (3 2 ) (3 3 ) 2 2 2 x x y y x y xy x y x y x y + + + + +
Bt ng thc trn ng do
3 3 2
2 2
AM GM
x y x y

+
Cng nh bi ton trn, ta c th lm cht bi ton v thu c
3 3 3 2 2 2 2 2 2 2
3
2( ) 3( ) ( )
2
a b c a b b c c a ab bc ca k t k + + + + + + + +


BI TON 7. Cho cc s thc dng , , . Chng minh
( )
( )
2 2 2
2 2 2
2 2 2
2
3 5
.
2 2 2
a b b c c a abc
a b c
ab bc ca ab bc ca abc
+ +
+ +
+
+ + + +

VIMF
LI GII (VIMF).
Nu a b c th
2 2 2
2 2 2
2( )
1
2( )
a b b c c a abc
ab bc ca abc
+ +

+ +
, nn d dng suy ra iu phi chng minh
Nu c b a th bt ng thc c vit li nh sau

( )
2 2 2
2 2 2
( ) ( ) ( ) 6( )( )( )
2
a b b c c a a b b c c a
ab bc ca ab bc ca abc
+ +

+ + + +
K thut phn tch bnh phuong cho bt dng thc hon vj
This file was downloaded from the VietNam Inequality Mathematic Olympiad Resource Page http://ddbdt.tk

M
2 2
2 2 2
2 ( ) ( ) ( )( )
( ) ( ) ( ) 6( )( )
c b b a c b b a
a b b c c a c b b a
ab bc ca ab bc ca ab bc ca
( + +
+ +

=
+ + + + + +

Nn ta ch cn chng minh
( )
2 2 2
1
2
c a
ab bc ca ab bc ca abc

+ + + +

Quy ng v rt gn bt ng thc trn thnh
2 2
( )( ) 2 3 0 c b a c a ca a b + +
Bt ng thc trn hin nhin ng. Vy ta c iu phi chng minh. ng thc xy ra khi v ch khi
.

BI TON 8. Cho cc s thc khng m , , . Chng minh bt ng thc
3 3 3
3
| ( )( )( ) |
3 4
a b c
abc a b b c c a
+ +
+
LI GII (VIMF). Tht ra ta c th chng minh bt ng thc mnh hn l
3 3 3
| ( )( )( ) |
3
a b c
abc a b b c c a
+ +
+
Khng mt tnh tng qut gi s c b a . Bt ng thc trn c vit li nh sau
3 3 3
3 3| ( )( )( ) | a b c abc a b b c c a + + +
2 2 2
( ) ( ) ( ) ( ) 6| ( )( )( ) | a b c a b b c c a a b b c c a ( + + + +


p dng tiu chun 4, ta cn phi chng minh 6( ) 6( ) 0 12 6 0 a b c c a a b + + +
Bt ng thc trn hin nhin.
Vy ta c iu phi chng minh. ng thc xy ra khi v ch khi

BI TON 9. Cho cc s , , l di ba cnh ca mt tam gic. Chng minh rng
3


LI GII (VIMF). Bt ng thc ny tng ng vi
2


Ta xt 2 trng hp
Trng hp 1.
Nu th

0, nn
2


1
2


3
2


Trng hp 2. Nu th bt ng thc trn c vit li thnh

3
Hay

3
Do , , l 3 cnh ca mt tam gic nn t , , , 0. Ta c

,
Tng t ta cng c


V do ta ch cn chng minh

6
Ch rng

2 6
Nn ta cn chng minh

. 6 6

2 0
Bt ng thc ny hin nhin ng.
Php chng minh hon tt.

BI TON 10. Cho cc s thc bt k , , . Chng minh rng


K thut phn tch bnh phuong cho bt dng thc hon vj
This file was downloaded from the VietNam Inequality Mathematic Olympiad Resource Page http://ddbdt.tk
Vasile
LI GII (VIMF).
Nhn thy

| |

| |

|, nn ta ch cn chng minh bt ng thc khi , ,


0. Ta xt 2 trng hp
Trng hp 1. Nu th

, nn
3


3
2


V ta chng minh


3
2


Hay

0
Trng hp 2. Nu th bt ng thc c vit li thnh


3
2


3
2


Hay

3
V ta a v dng

,
Trong

, 3
Do

0 nn theo tiu chun 1 ta ch cn chng minh


2


Hay

2 2

2 0
Bt ng thc ny hin nhin ng v 0
Php chng minh hon tt.

VI/ Bi tp p dng.
Qua cc v d trn phn no ni ln im mnh ca k thut ny, v by gi cc bn th p dng
phng php ny gii quyt cc bi ton
BI TON 10. Cho cc s thc khng m , , sao cho
2 2 2
3 a b c + + = . Chng minh
2 2 2
2 ab bc ca abc + + +
BI TON 11 (PHAN THNH NAM). Cho cc s thc khng m , , tha
2 2 2
1 a b c + + = . Chng minh
1
( )( )( )( )
4
a b c a b b c c a + +
BI TON 12. Cho cc s thc khng m , , . chng minh bt ng thc
( )
2 2 2
3
2
2
a b c abc
b c c a a b ab bc ca
+ + +
+ + + + +















K thut cng mu s Engel ca bt dng thc Chebyshev


This file was downloaded from VietNam Inequality Mathematic Olympiad Resource Page http://www.ddbdt.tk
K THUT CNG MU S ENGEL CA
BT NG THC CHEBYSHEV
VIMF VIMF VIMF VIMF

Nh cc bn bit Bt ng thc Chebyshev l mt cng c mnh gii quyt mt lp cc bt ng
thc. Trc khi n vi bi vit ny ti xin nhc li mt cht v bt ng thc ny

I/ Bt ng thc Chebychev c in v Chebyshev dng Engel

1a. Bt ng thc Chebyshev trn 2 dy n iu cng chiu:
Cho 2 dy hu hn cc s thc

, ,

, ,

, khi
Nu c
1 2
1 2
...
...
n
n
a a a
b b b

hoc
1 2
1 2
...
...
n
n
a a a
b b b

th ta c

. . .

. . .


Du bng xy ra khi
1 2
1 2
...
...
n
n
a a a
b b b

= = =

= = =


Bt ng thc Chebyshev suy rng:
Nu
1 2 1 2
1
1 2 1 2
1
...
...
2
...
...
2
n
n
a a a a a
a
n
b b b b b
b
n

+ + + +

+ + + +

hoc
1 2 1 2
1
1 2 1 2
1
...
...
2
...
...
2
n
n
a a a a a
a
n
b b b b b
b
n

+ + + +

+ + + +

th

. . .

. . .



1b. Bt ng thc Chebyshev trn 2 dy n iu ngc chiu:
Cho 2 dy hu hn cc s thc

, ,

, ,

, khi
Nu c
1 2
1 2
...
...
n
n
a a a
b b b

hoc
1 2
1 2
...
...
n
n
a a a
b b b

th ta c

. . .

. . .


Du bng xy ra khi
1 2
1 2
...
...
n
n
a a a
b b b

= = =

= = =


Bt ng thc Chebyshev suy rng:
Nu
1 2 1 2
1
1 2 1 2
1
...
...
2
...
...
2
n
n
a a a a a
a
n
b b b b b
b
n

+ + + +

+ + + +

hoc
1 2 1 2
1
1 2 1 2
1
...
...
2
...
...
2
n
n
a a a a a
a
n
b b b b b
b
n

+ + + +

+ + + +

th

. . .

. . .



Vic chng minh cc bt ng thc trn l kh n gin. Cc bn c th tham kho t nhiu ngun ti liu khc
nhau.

By gi tr li vi ch chnh, nu ta thay dy

, ,

bi dy

, ,

th khi
2a. Nu c
1 2
1 2
...
...
n
n
a a a
x x x

hoc
1 2
1 2
...
...
n
n
a a a
x x x

th ta c


2b. Nu c
1 2
1 2
...
...
n
n
a a a
x x x

hoc
1 2
1 2
...
...
n
n
a a a
x x x

th ta c
K thut cng mu s Engel ca bt dng thc Chebyshev
This file was downloaded from VietNam Inequality Mathematic Olympiad Resource Page http://www.ddbdt.tk


Tuy nhin trong mt s trng hp,ta khng th nh gi theo 2 dy ny. Khi sso ta c mt cch khc phc,
l:
3a. Nu c
1 2
1 2
1 2
...
...
n
n
n
a a a
x x x
x x x

hoc
1 2
1 2
1 2
...
...
n
n
n
a a a
x x x
x x x

th ta c


Chng minh. theo 1a, ta c


Do



3b. Nu c
1 2
1 2
1 2
...
...
n
n
n
a a a
x x x
x x x

hoc
1 2
1 2
1 2
...
...
n
n
n
a a a
x x x
x x x

th ta c


Chng minh. theo 1b, ta c


Do


Mc d 2 bt ng thc ny c pht biu n gin v c suy ra t bt ng thc Chebychev c in.
Tuy nhin trong mt s trng hp, vic nh gi theo 2a v 2b khng my hiu qu. V th, 3a v 3b (kt hp
vi vic thm bin thch hp li tr nn hiu qu i vi cc bt ng thc i xng 3 bin c cha phn thc.
lm r iu ny chng ta cng xt n cc bi ton sau:
II/ p dng vo gii ton.
BI TON 1. Cho cc s thc dng , , sao cho
1
1

1
1

1
1
1
Chng minh rng
Italia 2007
LI GII. Ta c
1
1

1
1

1
1










Khng mt tnh tng qut, gi s th v









1
1

1
1

1
1

Bt ng thc trn lun ng.
T theo 3a th
1
1

1
1

1
1








3
2

Kt hp vi gi thit ta suy ra
3
2
1
Vy ta c iu phi chng minh. ng thc xy ra khi v ch khi 1.

K thut cng mu s Engel ca bt dng thc Chebyshev
This file was downloaded from VietNam Inequality Mathematic Olympiad Resource Page http://www.ddbdt.tk

BI TON 2. Cho tam gic nhn . Chng minh rng
1
1

1
1

1
1

3
1 23

VIMF
LI GII. Theo nh nh gi ca bi ton 1 th
1
1

1
1

1
1

3
2

T bt ng thc 3 v ng thc tan ta c
3
2

3
1
2


3
1 2
3

3
1 2
3

3
1 23

Vy, ta c iu phi chng minh. ng thc xy ra khi v ch khi tam gic u

BI TON 3. Cho cc s thc dng , , sao cho 3. Chng minh rng

9
4



9
4



9
4


13
4

VIMF
LI GII.
Khng mt tnh tng qut, gi s . Ta c 1 0 nn , tng
t ta suy ra
Li c

9
4



9
4

3
3
4
0
Bt ng thc trn lun ng
Tng t ta suy ra

9
4



9
4



9
4


T theo 3b th

9
4



9
4



9
4


3
27
4

2

3 9
6

Ta cn chng minh
3 9
6

13
4
3
Bt ng thc trn ng do

3
Vy, ta c iu phi chng minh. ng thc xy ra khi v ch khi 1.

BI TON 4. Cho cc s thc dng , , sao cho

1. Chng minh rng



1


1


1

9
2

LI GII (VIMF). Bt ng thc tng ng vi
3
2

2 2
1
0
Khng mt tnh tng qut, gi s 1 1 1
K thut cng mu s Engel ca bt dng thc Chebyshev
This file was downloaded from VietNam Inequality Mathematic Olympiad Resource Page http://www.ddbdt.tk
Ta tip tc thit lp

2 2
1

2 2
1

2

0
2 2

3 3

0
21

21

0
2

2 2

0
Bt ng thc trn lun ng. Thit lp tng t ta suy ra

2 2
1

2 2
1

2 2
1

T theo 3b ta c

2 2
1

3

2 2
3

12


3
0
Vy ta c iu phi chng minh. ng thc xy ra khi v ch khi

.
NHN XT. Nu ta mun nh gi theo kiu Chebychev c in, tc l thit lp thm

2 2

2 2 2 2 0
iu ny tht kh gii quyt bi 2 2 l m hay dng?.

BI TON 5. Cho cc s thc , , sao cho 1. Chng minh rng

1

9
10

Poland 1992
LI GII. Ta c

1

||

||

||

1

Nn ta ch cn chng minh bt ng thc vi , , 0
Khng mt tnh tng qut, gi s th

1
Tip tc ta c

1
x y1 xy 0
Do 1 2 1 nn bt ng thc trn hin nhin ng
Tng t ta suy ra

1

T theo 3a ta c

1

3

3

3
1
3

3

9
10

Vy, ta c iu phi chng minh. ng thc xy ra khi v ch khi

.

BI TON 6. Cho cc s thc dng , , sao cho
1
1 2

1
1 2

1
1 2
1
Chng minh rng 3
VIMF
LI GII. Ta c
1
1 2

1
1 2

1
1 2


2


2


2

Khng mt tnh tng qut, gi s th 2 2 2 v

2


2


2

1
1 2

1
1 2

1
1 2
ab ac bc
Bt ng thc trn lun ng vi
T theo 3a, ta c
1
1 2

1
1 2

1
1 2


2


2


2

3
6

K thut cng mu s Engel ca bt dng thc Chebyshev
This file was downloaded from VietNam Inequality Mathematic Olympiad Resource Page http://www.ddbdt.tk
Kt hp vi gi thit suy ra
3
6
1 3
Vy ta c iu phi chng minh. ng thc xy ra khi v ch khi 1

BI TON 7. Cho cc s thc dng , , sao cho. Chng minh
1
1

1
1



1
1



3
1 2

Mathlinks Contest
LI GII (VIMF). Bt ng thc trn tng ng vi
1
1 2

1
1

1
1 2

1
1

1
1 2

1
1


0
(1 ) (1 ) (1 ) 1 1 1
0 0
1 (3 ) 1 (3 ) 1 (3 ) 1 (3 ) 1 (3 ) 1 (3 )
a bc b ac c ab bc ac ab
a bc b ac c ab abc bc abc ac abc ab

+ + + +
+ + + + + +

Khng mt tnh tng qut, gi s , a b c th 1 (3 ) 1 (3 ) 1 (3 ) abc ab abc ac abc bc + + + , v
1 1 1
1 (3 ) 1 (3 ) 1 (3 )
bc ac ab
abc bc abc ac abc ab


+ + +
(do 3 3

1)
Do theo 3b ta c
( ) 3 1 1 1
1 1 1
0
1 (3 ) 1 (3 ) 1 (3 ) 1 (3 ) 1 (3 ) 1 (3 )
bc ac ab
bc ac ab
abc bc abc ac abc ab abc bc abc ac abc ab
+ +

+ + =
+ + + + + + + +

Php chng minh hon tt. ng thc xy ra khi v ch khi 1.

BI TON 8. Cho cc s thc dng , , sao cho 1. Chng minh rng
1

1


1

1


1

1


27
31

Serbian National Olympiad 2008
LI GII (VIMF). Bt ng thc cho tng ng vi

1

27
31

Khng mt tnh tng qut, gi s . Khi

1. T gi
thit ny ta cng c
2 2
1 1
x y
xyz x xyz y

+ + + +

Tht vy, bt ng thc ny tng ng vi
1

1


1

1

1 0
Ch rng t iu kin 1 2 1 nn

1 0 v theo gi thit trn


. Do bt ng thc trn l ng. Tng t ta cng suy ra c

1

Do theo 3a ta c

1
3 3
Suy ra
2 2 2 2 2 2
3
1 1 1 3 3
x y z
xyz x xyz y xyz z xyz x y z
+ +
+ + + + + + + + + +

Mt khc, t bt ng thc ta c

, suy ra
3
1
3


2
3


Nn
( ) ( )
2
2 2 2 2 2 2
1 1 10 1 31
3 3 3
3 3 3 9 9
xyz x y z x y z x y z + + + + + + + + + + + =
K thut cng mu s Engel ca bt dng thc Chebyshev
This file was downloaded from VietNam Inequality Mathematic Olympiad Resource Page http://www.ddbdt.tk
Do
2 2 2 2 2 2
3 27
1 1 1 3 3 31
x y z
xyz x xyz y xyz z xyz x y z
+ +
+ + + + + + + + + +

Php chng minh hon tt.

BI TON 9. Cho cc s thc dng , , sao cho 4. Chng minh rng
3
1

2 2 2
LI GII (VIMF). T iu kin bi ton ta c th t


V bt ng thc cn chng minh c vit li thnh
3
2



T bt ng thc

th ta ch cn chng minh
3
2



Hay
( ) 6 x y z
x y y z z x
z x y xy yz zx
+ +
+ + +
+ +
+ +

Ta c
x y y z z x x y x z y z
z x y xz yz xy yz xy zx
+ + + + + +
+ + = + +
+ + +

Khng mt tnh tng qut, gi s . Khi , v











Do theo 3b v t bt ng thc , th

6
3

6


Php chng minh hon tt. ng thc xy ra khi v ch khi 1.

III/ Bi tp p dng.

BI TON 1. Cho cc s thc dng , , sao cho 1. Chng minh

2
3
2

2


2
3
2

2


2
3
2

2
3
BI TON 2. Cho cc s thc dng , , sao cho 1. Chng minh


33
33 1

BI TON 3. Cho cc s thc dng , , sao cho 3. Chng minh


1
2

BI TON 4. Cho , , l di 3 cnh ca mt tam gic. Chng minh rng

Mt dang bt dng thc cha cn


This file was downloaded from VietNam Inequality Mathematic Olympiad Resource Page http://www.ddbdt.tk
MT DNG BT NG THC CHA CN

BI TON *. Vi , , , , , 0. Ta c bt ng thc
3 3 3 3 3 3 3 3 4 4
4 4
( ) ( ) 54( ) a x b y c z a b c x y z abc xyz + + + + + + + + + + + +
LI GII. Sau khi m 4 hay v,khai trin v rt gn ta c bt ng thc tng ng
( )( ) ( )( ) ( )( )
( )
3 3 3 3 3 3 3 3 3 3 3 3
6 a x b y c z b y a x c z + + + + + + + + +
( ) ( ) ( ) ( ) ( ) ( )
3 3 3
3 3 3 3 3 3 3 3 3 3 3 3
4 4 4
4 a x b y b y c z c z a x
| |
+ + + + + + + + +
|
\

( ) ( ) ( ) ( ) ( ) ( )
3 3 3
3 3 3 3 3 3 3 3 3 3 3 3
4 4 4
4 b y a x c z b y a x c z
| |
+ + + + + + + + +
|
\
( ) ( )( ) ( ) ( )( ) ( ) ( )( )
2 2 2
3 3 3 3 3 3 3 3 3 3 3 3 3 3 3 3 3 3
4 4 4
12 a x b y c z b y c z a x c z a x b y
| |
+ + + + + + + + + + +
|
\

( ) ( )
2 2 2 2 2 2 2 2 2 2 2 2
3 3 60( )(*) a b b a b c c b c a a c x y y x y z z y z x x z abc xyz + + + + + + + + + + + + +
Dng bt ng thc CBS suy rng cho 4 dy ta c

( )
3 3 3 3 3 3 3 3 3 3 3 3
VT(*) 6 a b x y b c y z c a z x + + + + +
( )
9 3 9 3 9 3 9 3 9 3 9 3 9 3 9 3 9 3 9 3 9 3 9 3 4 4 4 4 4 4 4 4
4 4 4 4
4 a b b a x y y x b c c b y z z y c a a c z x x z + + + + + + + + + + + + +

( )
6 3 3 6 3 3 6 3 3 6 3 3 6 3 3 6 3 3 4 4 4
4 4 4
12 a b c x y z b a c y x z c b a z x y + + + + + +
3 3 9 3 9 3 6 3 3 3 3 9 3 9 3 6 3 3 4 4 4
4 4 4
6 4 4 12 6 4 4 12 a b a b b a c b a x y x y y x z x y = + + + + + + +


Gi ta chng minh bt ng thc sau
3 3 9 3 9 3 6 3 3 2 2 4 4 4
6 4 4 12 3 3 60 (**) a b a b b a c b a a b ab abc + + + + +


Bt ng thc (**) c chng minh t cc bt ng thc sau
( ) ( )
9 3 9 3 7 5 7 5 4 4 4 4
5 5
)
2 2
a a b b a a b b a + +


( )
( )
9 3 9 3 7 5 7 5 2 2 4 4 4 4
3
) 3
2
b a b b a a b b a a b ab + + + +


7 5 7 5 4 4
) 6 c a b b a abc +


3 3
)6 18 d a b abc


6 3 3 4
)12 36 e a b c abc


Chng minh a). p dng bt ng thc ta c
9 3 9 3 7 5 4 4 4
2 3 a b b a a b +
9 3 9 3 7 5 4 4 4
2 3 b a a b b a +
Thit lp cc bt ng thc tng t v cng v theo v th chng minh c a)
Chng minh b) Ta c
9 3 7 5 2 4 4
2 a b a b a b + ;
9 3 7 5 2 4 4
2 b a b a b a +
Thit lp cc bt ng thc tng t v cng v theo v ta chng minh c b)
Bt ng thc c,d,e chng minh khng kh. Cng cc bt ng thc a,b,c,d,e v theo v ta c
3 3 9 3 9 3 6 3 3 2 2 4 4 4
6 4 4 12 3 3 60 (**) a b a b b a a b c a b ab abc + + + + +


Chng minh tng t cho cc bin , , ta cng c
3 3 9 3 9 3 6 3 3 2 2
4 4 4
6 4 4 12 3 3 60 (***) y x y x x y z x y x y xy xyz + + + + +


Cng 2 bt ng thc (**) v (***) ta c iu phi chng minh
Vy bi ton c chng minh xong. V ta i n

BI TON TNG QUT 1. Vi
1 2 1 2 1 2
; ;...; ; ; ;...; ; ; ;...;
n n n
a a a b b b c c c l cc s dng.Ta c bt ng thc
3 3 3 3 3 3 3 3 3
4 4 4
1 2 1 2 1 2
... ... ...
n n n
a a a b b b c c c + + + + + + + + + + +
( ) ( ) ( ) ( )
3 3 3
4
1 1 1 2 2 2 1 1 1 2 2 2
... 54 ...
n n n n n n
a b c a b c a b c a b c a b c a b c + + + + + + + + + + + + +
Mt dang bt dng thc cha cn
This file was downloaded from VietNam Inequality Mathematic Olympiad Resource Page http://www.ddbdt.tk
Vic chng minh bi ny tng t nh bi trc

BI TON TNG QUT 2. Vi
1 2 1 2 1 2
; ;...; ; ; ;...; ; ; ;...;
n n n
a a a b b b c c c l cc s dng; t l s t nhin nh hn 5.Ta
c bt ng thc
( )
3 3 2 2
4 4 4
4 4
1 2 1 2 1 2
4
... ... ... ( ) 27
3
t t t t t t t t t
n n n
a a a b b b c c c a a b c a b ab + + + + + + + + + + + + + + + +


BI TON TNG QUT 3. Vi
1 2 1 2 1 2
; ;...; ; ; ;...; ; ; ;...;
n n n
a a a b b b c c c l cc s dng; k l s t nhin; ; 3. k t k
Ta c
1 2 1 2 1 2
... ... ...
t t t t t t t t t
k k k
n n n
a a a b b b c c c + + + + + + + + + + +
( ) ( ) ( ) ( ) ( ) ( ) ( )
( )
3 3 3
1 1 1 2 2 2 1 1 1 2 2 2
... 3 3 ...
t t t t t t
k t
k
n n n n n n
a b c a b c a b c a b c a b c a b c + + + + + + + + + + + + +

T tng chng minh khng khc g bi u tin. y cho cng ln th thi gian chng minh cng nhiu

BI TP P DNG.

BI TON 1. Cho , , 0 tha 4.Chng minh bt ng thc
( )
2
2 4
4 3
15 1242 54 a abc + +


BI TON 2. Cho , , 0 tha
2
3 a =

.Chng minh bt ng thc


( )
3 3 2 2
4 4
4
( ) 27
3
a a b c a b ab + + + + +































Ti Liu Tham Kho
This file was downloaded from VietNam Inequality Mathematic Olympiad Resource Page http://www.ddbdt.tk
TI LIU THAM KHO


[1] Titu Andreescu, Vasile Cirtoaje. Gabriel Dospinescu, Mircea Lascu, Old and New Inequalities, Vol 1, GLI
publishing house, 2004.

[2] Phm Kim Hng, Sng to bt ng thc , nh xut bn Tr, 2006.

[3] V Quc B Cn, Bt ng thc Tm ti v Sng to
[4] Phan Thnh Vit, Chuyn ton hc: Phng php chia tr

[5] Cc ti liu Olympic online.
- www.mathlinks.ro/
- www.ddbdt.tk/
- www.truongtructuyen.vn/










































www.ddbdt.tk

You might also like